and answers sem-i/ii

108
Engineering Physics ( 17PHY12/22) MODULE WISE QUESTIONS AND ANSWERS SEM-I/II VTU ENGINEERING PHYSICS C.REDDAPPA, M.Sc; B.Ed; Prof. & HOD,CBIT,KOLAR.

Upload: others

Post on 21-May-2022

1 views

Category:

Documents


0 download

TRANSCRIPT

Page 1: AND ANSWERS SEM-I/II

Engineering Physics

( 17PHY12/22)

MODULE WISE

QUESTIONS AND

ANSWERS

SEM-I/II

VTU ENGINEERING PHYSICS

C.REDDAPPA, M.Sc; B.Ed; Prof. & HOD,CBIT,KOLAR.

Page 2: AND ANSWERS SEM-I/II

IMPORTANT IA TEST & VTU EXAMINATION QUESTIONS

MODULE-1

Modern Physics and Quantum Mechanics

1. State Planck’s quantum hypothesis(postulates) and explain black body radiation spectra

based on planck’s radiation law.

2. State Planck’s law of radiation and deduce Wein’s law and Rayleigh-Jeans law from Planck’s

law of radiation.

3. Explain Compton effect experimentally.

4. What are matter(de-Broglie) waves ? Mention the properties of matter waves.

5. Derive the relation between Phase velocity and Group velocity.

6. Define Phase velocity and Group velocity and show that Group velocity is equal to Particle

velocity.

7. State and explain Heisenberg’s uncertainty Principle and Show that electrons non exist in the

nucleus using Heisenberg’s uncertainty Principle .

8. Set up Schrodinger 1D( one dimensional) time independent wave equation.

9. Apply Schrodinger time independent wave equation to a particle in an infinite depth

potential well/Box to find eigen energies and eigen functions.

10. Mention the properties of wave function.

MODULE-2

Electrical properties of Materials.

1. Explain the terms : Drift velocity, Mean collision time, Mean free path, Relaxation time.

2. Explain the failures of CFET(Classical free electron theory)

3. State the assumptions of QFET(Quantum free electron theory) and explain the merits of

QFET.

4. Discuss the dependence of Fermi factor on temperature and energies.

5. Derive the expression for electrical conductivity based on QFET.

6. State law of mass action and Derive an expression for concentration of electrons and holes

( carrier density) in intrinsic semiconductors.

7. Write a note on Meissner’s effect and Maglev vehicles.

8. Distinguish between Type-I and Type-II superconductors.

9. Explain BCS theory qualitatively.

10. State and explain Matheissen’s rule ( Dependence of resistance on impurities and

temperature)

11. Write a note on Fermi-Dirac statistics.

Continued on inner side of Back page

Page 3: AND ANSWERS SEM-I/II

REDDAPPA C MODULE-1 INTERLINE PUBLISHING.COM

VTU ENGINEERING PHYSICS ( 17 PHY 12/22) SEM-I/II Page 1

ENGINEERING PHYSICS (15PHY12/22)

SEMESTER - I / II

[ As per CBCS Scheme with effect from the academic year 2015-2016 ]

MODULE-1 : MODERN PHYSICS AND QUANTUM MECHANICS

MODERN PHYSICS:

Q:What is meant by Black Body radiation spectra ? explain it briefly.

1. If we plot intensity of the thermal radiations emitted by a Black Body against the

corresponding wave lengths at different temperatures, we get Black body radiation

(BBR)spectra as shown in the graph.

2. From the graph it is clear that the wavelength (πœ†π‘š) corresponding to the maximum

intensity π‘¬π’Žvaries inversely as the absolute temperature(T ) of the black body. This is

called Wein’s displacement law , π’Šπ’†: π€π’Ž ∝𝟏

𝑻 or π€π’Ž 𝑻 = 𝒄𝒐𝒏𝒔𝒕𝒂𝒏𝒕 According to this law

β€²πœ†π‘šβ€² shifts towards shorter wavelength side with the increase of β€˜ T ’ of the body.

3. Wein’s law states that the energy density in the wave length interval Ξ» & Ξ» +dΞ»

is π‘ˆπœ† dΞ» = 𝐢1πœ†βˆ’5π‘’βˆ’πΆ2/πœ†π‘‡dΞ» where 𝐢1 & 𝐢2 are constants. Wein’s law explained only

the shorter wavelength region of BBR spectra below πœ†π‘š and it failed to

explain the longer wavelength region of the BBR spectra beyond πœ†π‘š.

4. Rayleigh-Jean’s law states that the energy density in the wave length interval Ξ» &

Ξ» +dΞ» is π‘ˆπœ† dΞ» = 8πœ‹π‘˜π‘‡

πœ†4 dΞ» ,where k = Boltsmann’s constant. Rayleigh-Jean’s law

explained the longer wavelength region of BBR spectra beyond πœ†π‘š and it failed

explain the shorter wavelength region of BBR spectra below πœ†π‘š.

E

Rayleigh-Jean’s law

π‘¬π’Ž Experimental B.B.R spectra

Wein’s law

𝟎 π€π’Ž 𝝺

Page 4: AND ANSWERS SEM-I/II

REDDAPPA C MODULE-1 INTERLINE PUBLISHING.COM

VTU ENGINEERING PHYSICS ( 17 PHY 12/22) SEM-I/II Page 2

5. Planck’s law states that the energy density in the wavelength interval Ξ» & Ξ» +dΞ» is

π‘ˆπœ† dΞ» = 8πœ‹β„Žπ‘

πœ†5

1

𝑒(

β„Žπ‘πœ†π‘˜π‘‡

) βˆ’1

dΞ» .

Planck’s law explained the BBR spectra completely ie: both shorter and longer

wavelength sides of the spectra.

Q: State Planck’s quantum postulates and explain Planck’s explanation of BBR specta.

The quantum postulates are :

1. The black body is made up of a large number of simple Harmonic oscillating particles,

which can oscillate in all possible frequencies.

2. An oscillating particle can have a set of energies ,which are the integral multiples of a

lowest finite quanta of energy(h𝜈) i.e: 𝐸𝑛 = n.hΞ½ ,where n = Quantum number.

3. The oscillating particles absorb or emit energy in discrete units of hΞ½, only when they

undergo transitions between any two allowed energy states.

Planck based on quantum postulates, derived an expression for energy density

in the wavelength range Ξ» and Ξ»+dΞ» called Planck’s law given by

π‘ˆπœ† dΞ» =

8πœ‹β„Žπ‘

πœ†5

1

𝑒(

β„Žπ‘πœ†π‘˜π‘‡

) βˆ’1

dΞ».

Based on this, Planck explained the BBR spectrum completely ie: both shorter and longer

wavelength side of the spectrum.

E

Rayleigh-Jean’s law

π‘¬π’Ž Experimental B.B.R spectra

Wein’s law

𝟎 π€π’Ž 𝝺

Page 5: AND ANSWERS SEM-I/II

REDDAPPA C MODULE-1 INTERLINE PUBLISHING.COM

VTU ENGINEERING PHYSICS ( 17 PHY 12/22) SEM-I/II Page 3

Q:Deduce/derive Wein’s law & Rayleigh-Jeans law from planck’s law of radiation or

Reduce Planck’s law to Wein’s law and Rayleigh-Jeans law

Planck’s law of radiation is given by π‘ˆπœ†dΞ» = 8πœ‹β„Žπ‘

πœ†5

1

𝑒(

β„Žπ‘πœ†π‘˜π‘‡

) βˆ’1

dΞ» ……………(1)

a) Wein’s law:

For shorter wave lengths, β€˜Ξ» β€˜is very small ,hence π‘ˆπœ† & 𝑒(β„Žπ‘

πœ†π‘˜π‘‡) are

Large i.e: 𝑒(β„Žπ‘

πœ†π‘˜π‘‡) ≫ 1 so that 𝑒(

β„Žπ‘

πœ†π‘˜π‘‡) βˆ’ 1 = 𝑒(

β„Žπ‘

πœ†π‘˜π‘‡) ……….(2)

∴ from eqn 1 &2,we get π‘ˆπœ†dΞ» = 8πœ‹β„Žπ‘

πœ†5

1

𝑒(

β„Žπ‘πœ†π‘˜π‘‡

)

dΞ»

∴ 𝑼𝝀 dΞ» = π‘ͺπŸπ€βˆ’πŸ“π’†βˆ’π‘ͺπŸπ€π‘» dΞ» π‘‡β„Žπ‘–π‘  𝑖𝑠 𝑀𝑒𝑖𝑛’𝑠 π‘™π‘Žπ‘€ .where 𝐢1 = 8πœ‹β„Žπ‘ & 𝐢2 =

β„Žπ‘

π‘˜

b) Rayleigh-Jeans law:

For longer wave lengths, β€˜Ξ» β€˜is very large,

hence 𝑒(β„Žπ‘

πœ†π‘˜π‘‡) 𝑖𝑠 π‘£π‘’π‘Ÿπ‘¦ π‘ π‘šπ‘Žπ‘™π‘™ ,Expanding 𝑒(

β„Žπ‘

πœ†π‘˜π‘‡) as power series ,

we get , 𝑒(β„Žπ‘

πœ†π‘˜π‘‡) = 1+ (

β„Žπ‘

πœ†π‘˜π‘‡) + (

β„Žπ‘

πœ†π‘˜π‘‡)2 +……….

= 1 + (β„Žπ‘

πœ†π‘˜π‘‡) , 𝑛𝑒𝑔𝑙𝑒𝑐𝑑𝑖𝑛𝑔 β„Žπ‘–π‘”β„Žπ‘’π‘Ÿ π‘π‘œπ‘€π‘’π‘Ÿπ‘  π‘œπ‘“ (

β„Žπ‘

πœ†π‘˜π‘‡)

𝑖𝑒: 𝑒(

β„Žπ‘

πœ†π‘˜π‘‡)

βˆ’ 1 = β„Žπ‘

πœ†π‘˜π‘‡ ………(3)

∴ from eqn 1 &3,we get π‘ˆπœ†dΞ» = 8πœ‹β„Žπ‘

πœ†5

1β„Žπ‘

πœ†π‘˜π‘‡ dΞ»

π’Šπ’†: 𝑼𝝀 dΞ» = πŸ–π…π’Œπ‘»

π€πŸ’ dΞ» , π‘‡β„Žπ‘–π‘  is Rayleigh-Jeans law

Q: What is meant by β€œultraviolet catastrophe” or failures of Rayleigh-Jeans law ?

1. As per Rayleigh-Jeans law, π‘ˆπœ† β†’ ∞ π‘Žπ‘  πœ† β†’ 0

2. 𝐡𝑒𝑑 experimentally observed that π‘ˆπœ† β†’ 0 π‘Žπ‘  πœ† β†’ 0.

3. This failure of R-J’s law beyond ultra-violet region is called β€œultraviolet catastrophe”.

Page 6: AND ANSWERS SEM-I/II

REDDAPPA C MODULE-1 INTERLINE PUBLISHING.COM

VTU ENGINEERING PHYSICS ( 17 PHY 12/22) SEM-I/II Page 4

Q:Explain Compton effect experimentally & mention it’s significance.

1. Compton experimental arrangement is as shown in the diagram.

2. X-ray photons of wave length Ξ» from X-ray tube are collimated by passing through two

slits S1 &S2.

3. The collimated X-rays are made to fall on graphite target.

4. X-rays collide with the electrons of the target at rest and transfer part of their energy

resulting in the recoil of electrons in a direction making an angle βˆ… π‘Žπ‘›π‘‘ π‘†π‘π‘Žπ‘‘π‘‘π‘’π‘Ÿπ‘’π‘‘ 𝑋 βˆ’

π‘Ÿπ‘Žπ‘¦π‘  π‘šπ‘Žπ‘˜π‘’π‘  π‘Žπ‘› π‘Žπ‘›π‘”π‘™π‘’ πœƒ π‘Ÿπ‘’π‘ π‘π‘’π‘π‘‘π‘–π‘£π‘’π‘™π‘¦ π‘€π‘–π‘‘β„Ž the incident X-rays direction.

5. The intensity of the scattered X-rays in different directions are measured using Bragg’s

spectrometer.

6. Compton calculated the wavelengths of scattered X-rays at different scattering angles and

found that scattered X-rays consists of two components namely β€˜unmodified’ component

having the same wavelength(Ξ») as incident X-rays & ’modified’ component having slightly

higher wavelength πœ†β€²

7. This scattering of X-rays due to recoil of electrons is called β€œCompton effect”.

8. The change in wavelength (πœ†β€² βˆ’ πœ†)is called β€˜Compton wavelength’ ,which depends only

πœƒ π‘Žπ‘›π‘‘ 𝑖𝑛𝑑𝑒𝑝𝑒𝑛𝑑𝑒𝑛𝑑 π‘œπ‘“ 𝞴.

9. Based on conservation of energy & momentum laws, considering the collision between X-

rays & electrons as β€˜particle-particle’ collision , Compton wavelength is given by

(πœ†β€² βˆ’ πœ†) = βˆ†πœ† = β„Ž

π‘š0𝐢 (1βˆ’πΆπ‘œπ‘ πœƒ)

where β„Ž

π‘š0𝐢= πœ†0 called β€œCompton wavelength of electron”= 0.02426 Γ…(constant)

10. βˆ†π€ π‘£π‘Žπ‘Ÿies from β€˜0’ for πœƒ = 0Β° π‘‘π‘œ 2πœ†0 for πœƒ = πŸπŸ–πŸŽΒ°

11. πΆπ‘œπ‘šπ‘π‘‘π‘œπ‘› 𝑒𝑓𝑓𝑒𝑐𝑑 𝑠𝑖𝑔𝑛𝑖𝑓𝑖𝑒𝑠 π‘‘β„Žπ‘’ π‘π‘Žπ‘Ÿπ‘‘π‘–π‘π‘™π‘’ π‘›π‘Žπ‘‘π‘’π‘Ÿπ‘’ π‘œπ‘“ 𝑋 βˆ’ π‘Ÿπ‘Žπ‘¦π‘ (π‘™π‘–π‘”β„Žπ‘‘)

Q: What is meant by Dual nature of matter ?

Matter is made up of particles like electrons, protons, neutrons, atoms etc ,also

waves are associated with these matter particles under suitable conditions.

Thus matter exhibiting both particle & wave nature is called dual nature of matter.

Bragg’s spectrometer

Slits

X-rays Collimated X-rays πœƒ

𝑆1 𝑆2 βˆ…

Graphite target Recoil electron

Page 7: AND ANSWERS SEM-I/II

REDDAPPA C MODULE-1 INTERLINE PUBLISHING.COM

VTU ENGINEERING PHYSICS ( 17 PHY 12/22) SEM-I/II Page 5

Q: What are de-Broglie (matter) waves ?

de-Broglie(matter) waves are the waves associated with material particles in

motion.

Q: Derive an expression for de-broglie wavelength(Ξ»).

According to Einstein’s mass-energy relation E=m𝐢2……(1)

where m=mass, C=speed of light.

also, from Planck’s quantum theory, E =β„ŽπΆ

πœ† …………(2) where h=planck’s contant.

Ξ½=frequency. From eqns 1&2, m𝐢2 = β„ŽπΆ

πœ†

∴ 𝝀 =𝒉

π’Žπ‘ͺ

Similarly, for a particle of mass β€˜m’ moving with a velocity β€˜π’— β€²,

De-broglie wavelength, Ξ» = 𝒉

π’Žπ“₯ or Ξ» =

𝒉

𝑷

Q: De-broglie’s wavelength of an electron accelerated in a potential difference ’V’ volt/

Show that Ξ» = 𝟏𝟐.πŸπŸ–

βˆšπ‘½ Γ… for an electron accelerated in a potential difference ’V’ volt

The K.E of an electron of mass β€˜m’,charge β€˜e’ accelerated in a potential β€˜V’ volt is given

by 1

2π‘šπ‘£2 = 𝑒𝑉 , multiplying both Nr. and Dr. of LHS by β€˜m’

we get, π‘š2𝑣2

2π‘š= 𝑒𝑉

𝑖𝑒; π‘š2𝑣2 = 2π‘šπ‘’π‘‰

𝑃2 = 2π‘šπ‘’π‘‰ ∡ π‘šπ‘£ = 𝑃

𝑷 = βˆšπŸπ’Žπ’†π‘½

But Ξ» = 𝒉

𝑷 ∴ 𝝀 =

𝒉

βˆšπŸπ’Žπ’†π‘½

= πŸ”.πŸ”πŸπŸ“π’™10βˆ’34

βˆšπŸπ’™ πŸ—.πŸπ’™πŸπŸŽβˆ’πŸ‘πŸπ’™πŸ.πŸ”π’™πŸπŸŽβˆ’πŸπŸ—π‘½

= 𝟏.πŸπŸπŸ–π’™πŸπŸŽβˆ’πŸ—

βˆšπ‘½ m

= 1.228

βˆšπ‘½π‘›π‘š =

𝟏𝟐.πŸπŸ–

βˆšπ‘½ Γ…

Q:Mention the characteristics of matter waves or Write a note of matter waves.

1. MW are non mechanical waves.

2. MW are associated with moving particles .

3. MW are non electromagnetic waves.

4. MW of microscopic particles can be measured .

5. MW are charge independent .

Page 8: AND ANSWERS SEM-I/II

REDDAPPA C MODULE-1 INTERLINE PUBLISHING.COM

VTU ENGINEERING PHYSICS ( 17 PHY 12/22) SEM-I/II Page 6

6. MW of macroscopic particles cannot be measured .

7. Different MW have different phase velocity.

8. Phase velocity of MW is greater than that of light.

9. Velocity of MW depends on the velocity of material particles.

Q:Define Phase velocity/What is meant by Phase velocity ?

β€œPhase Velocity ” is defined as the velocity with which the uni-phase particles on

the wave travels and is given by 𝑣𝑃 =πœ”

π‘˜

where πœ” = π‘Žπ‘›π‘”π‘’π‘™π‘Žπ‘Ÿ π‘£π‘’π‘™π‘œπ‘π‘–π‘‘π‘¦ π‘Žπ‘›π‘‘ π‘˜ = π‘€π‘Žπ‘£π‘’ π‘π‘œπ‘›π‘ π‘‘π‘Žπ‘›π‘‘.

Q: Define Group velocity/What is meant by Group velocity ?

β€œGroup Velocity ” is defined as the velocity with which the resultant wave packet of

the group of waves travels and is given by 𝑣𝑔 =π‘‘πœ”

π‘‘π‘˜

where dπœ” = π‘β„Žπ‘Žπ‘›π‘”π‘’ 𝑖𝑛 π‘Žπ‘›π‘”π‘’π‘™π‘Žπ‘Ÿ π‘£π‘’π‘™π‘œπ‘π‘–π‘‘π‘¦ π‘Žπ‘›π‘‘ π‘‘π‘˜ = π‘β„Žπ‘Žπ‘›π‘”π‘’ 𝑖𝑛 π‘€π‘Žπ‘£π‘’ π‘π‘œπ‘›π‘ π‘‘π‘Žπ‘›π‘‘.

Q: Derive the relation between group velocity (π’—π’ˆ) & phase velocity(𝒗𝑷)/

Show that π’—π’ˆ = 𝒗𝑷 βˆ’ 𝝀𝒅𝑉𝑃

𝒅𝝀

Consider a particle of mass β€˜m’ having phase velocity β€˜ 𝒗𝑷 β€˜ and group velocity β€˜β€²π’—π’ˆβ€².

Let the wave have the wavelength ’λ’ , frequency β€˜Ξ½, wave number k & angular velocity Ο‰

We know that 𝑣𝑃 =πœ”

π‘˜ …(1) and 𝑣𝑔 =

π‘‘πœ”

π‘‘π‘˜ ….(2) 𝑉𝑃

From eqns 1 & 2 , we get 𝑣𝑔 =𝑑(𝑉𝑃.π‘˜)

π‘‘π‘˜

= 𝑣𝑃 + π‘˜π‘‘π‘‰π‘ƒ

π‘‘π‘˜

= 𝑣𝑃 + π‘˜.𝑑𝑉𝑃

π‘‘πœ†

π‘‘πœ†

π‘‘π‘˜ ……(3)

Also, Ξ» = 2πœ‹

π‘˜ ∴

π‘‘πœ†

π‘‘π‘˜ =

𝑑

π‘‘π‘˜(

2πœ‹

π‘˜)

= βˆ’2πœ‹

π‘˜2 ……..(4)

Page 9: AND ANSWERS SEM-I/II

REDDAPPA C MODULE-1 INTERLINE PUBLISHING.COM

VTU ENGINEERING PHYSICS ( 17 PHY 12/22) SEM-I/II Page 7

From eqns 3 & 4 ,we get 𝑣𝑔 = 𝑣𝑃 + π‘˜.𝑑𝑉𝑃

π‘‘πœ†(

βˆ’2πœ‹

π‘˜2 )

= 𝑣𝑃 βˆ’ 2πœ‹

π‘˜ 𝑋

𝑑𝑉𝑃

π‘‘πœ†

ie: π’—π’ˆ = 𝒗𝑷 βˆ’ 𝝀𝒅𝑉𝑃

𝒅𝝀 ∡ Ξ» =

2πœ‹

π‘˜

Note: π’—π’ˆ < 𝒗𝑷 for a dispersive medium

and 𝑣𝑔 = 𝑣𝑃 = 𝐢 π‘“π‘œπ‘Ÿ π‘Ž π‘›π‘œπ‘› π‘‘π‘–π‘ π‘π‘’π‘Ÿπ‘ π‘–π‘£π‘’ π‘£π‘Žπ‘π‘π‘’π‘š π‘šπ‘’π‘‘π‘–π‘’π‘š

Q: Deduce the relation between group velocity(π’—π’ˆ) & particle velocity(π’—π‘·π’‚π’“π’•π’Šπ’„π’π’†)/

Show that π’—π’ˆ = π’—π‘·π’‚π’“π’•π’Šπ’„π’π’† or π’—π’ˆ = 𝒗

Consider a particle of mass β€˜m’ having particle velocity β€˜π’—π‘·π’‚π’“π’•π’Šπ’„π’π’†β€² & π‘”π‘Ÿπ‘œπ‘’π‘ π‘£π‘’π‘™π‘œπ‘π‘–π‘‘π‘¦ β€²π’—π’ˆβ€².

We know that, 𝑣𝑔 =π‘‘πœ”

π‘‘π‘˜ … … . . (1)

but πœ” = 2πœ‹πœˆ & 𝐸 = β„Žπœˆ ,we get πœ” = 2πœ‹πΈ

β„Ž

∴ dπœ” = (2πœ‹

β„Ž) 𝑑𝐸 … (2)

Also from k = 2πœ‹

πœ† & πœ† =

β„Ž

𝑃 we get k = (

2πœ‹

β„Ž)P

∴ dk = (2πœ‹

β„Ž)dP ……(3)

From eqns 1,2 &3,we get 𝑣𝑔 =π‘‘πœ”

π‘‘π‘˜=

(2πœ‹

β„Ž)𝑑𝐸

(2πœ‹

β„Ž)𝑑𝑃

=𝑑𝐸

𝑑𝑃 …….(4)

Also , from E = Β½m𝑣2 =π‘š2𝑣2

2π‘š & P = mπ“₯ ,

𝑀𝑒 𝑔𝑒𝑑 𝐸 = 𝑃2

2π‘š

∴ dE = 2𝑃

2π‘šπ‘‘π‘ƒ

𝑖𝑒; 𝑑𝐸

𝑑𝑃=

𝑃

π‘š =

π‘š π‘£π‘ƒπ‘Žπ‘Ÿπ‘‘π‘–π‘π‘™π‘’

π‘š

= π‘£π‘ƒπ‘Žπ‘Ÿπ‘‘π‘–π‘π‘™π‘’ ….(5)

From eqns 4 & 5,we get π’—π’ˆ = π’—π‘·π’‚π’“π’•π’Šπ’„π’π’† Thus group velocity =Particle velocity

Page 10: AND ANSWERS SEM-I/II

REDDAPPA C MODULE-1 INTERLINE PUBLISHING.COM

VTU ENGINEERING PHYSICS ( 17 PHY 12/22) SEM-I/II Page 8

QUANTUM MECHANICS

Q: State & explain Heisenberg’s Uncertainty Principle (HUP). Mention its significance.

HUP states that β€œthe product of the uncertainty β€˜Ξ”π‘₯’ in the position and the uncertainty

β€˜Ξ”Pπ‘₯’ in the momentum of a particle at any instant is equal to or greater than ( h/4𝛑) β€œ

i.e.: Ξ”π‘₯ Ξ”Pπ‘₯ β‰₯ β„Ž

4πœ‹ ,Where h is Planck’s constant.

The significance of HUP is that, it is impossible to determine simultaneously both the position and

momentum of the particle accurately at the same instant.

NOTE : Other HUP relations are βˆ†πΈ.βˆ†π‘‘ β‰₯ β„Ž

4πœ‹ , π‘€β„Žπ‘’π‘Ÿπ‘’ βˆ†πΈ = π‘’π‘›π‘’π‘Ÿπ‘”π‘¦, βˆ†π‘‘= time,

& βˆ†πΏ.βˆ†πœƒ β‰₯ β„Ž

4πœ‹

where , βˆ†πΏ = π‘Žπ‘›π‘”π‘’π‘™π‘Žπ‘Ÿ π‘šπ‘œπ‘šπ‘’π‘›π‘‘π‘’π‘š & βˆ†πœƒ = π‘‘π‘–π‘ π‘π‘™π‘Žπ‘π‘’π‘šπ‘’π‘›π‘‘.

Q: Show that electrons do not present in the nucleus using Heisenberg’s Uncertainty Principle.

Electron to be present in the nucleus, maximum uncertainty in position Ξ”π‘₯=10-14 m (diameter)

According to HUP,

The minimum uncertainty in momentum Ξ”Pπ‘₯ β‰₯ β„Ž

4πœ‹ π›₯π‘₯

β‰₯ 6.625 π‘₯ 10βˆ’34

4 π‘₯ 3.14 π‘₯ 10βˆ’14

Ξ”Pπ‘₯ β‰₯ 5.275 x 10-21 kg m/s = P(say)

Using E = m𝐢2 , 𝑃 = π‘šπ‘‰ π‘Žπ‘›π‘‘ π‘š = π‘š0

√1βˆ’π‘‰2

𝐢2

, 𝑀𝑒 π‘π‘Žπ‘› π‘ β„Žπ‘œπ‘€ π‘‘β„Žπ‘Žπ‘‘ the minimum energy of the

electron in the nucleus is given by 𝐸2 = P2c2 +π‘šπ‘œ2 𝑐4, but the rest mass energy π‘šπ‘œ

2 𝑐4 is

very very small compared to P2c2,neglecting π‘šπ‘œ2𝑐4 ,

we have E β‰ˆ PC = 5.275 x 10βˆ’21 x 3 x 108 J

= 5.275 π‘₯ 10βˆ’21 π‘₯ 3 π‘₯ 108

1.6 π‘₯ 10βˆ’13 MeV

= 9.89 MeV

= 10 MeV

But the maximum energy of the electrons(𝛃-particle) emitted

from the nucleus does not exceed 4MeV,hence electrons do not

present in the nucleus.

NOTE:

𝐸2 = π‘š2𝐢4 = π‘šπ‘œ

2𝐢4

(1βˆ’π‘£2

𝐢2) =

π‘šπ‘œ2𝐢6

(𝐢2βˆ’π‘£2)

𝑃2𝐢2 = π‘š2𝑣2𝐢2 = π‘šπ‘œ

2𝑣2𝐢2

(1βˆ’π‘£2

𝐢2) =

π‘šπ‘œ2𝐢4𝑣2

(𝐢2βˆ’π‘£2)

𝐸2 βˆ’ 𝑃2𝐢2 = π‘šπ‘œ

2𝐢4(𝐢2βˆ’π‘£2)

(𝐢2βˆ’π‘£2)

∴ π‘¬πŸ = π‘·πŸπ‘ͺ𝟐 + π’Žπ’πŸπ‘ͺπŸ’

Page 11: AND ANSWERS SEM-I/II

REDDAPPA C MODULE-1 INTERLINE PUBLISHING.COM

VTU ENGINEERING PHYSICS ( 17 PHY 12/22) SEM-I/II Page 9

Q: Set up 1D time independent Schrodinger wave equation for a free particle.

One dimensional wave function 𝚿 describing the de-broglie wave for a particle moving

freely in the positive direction of π‘₯-direction is given by 𝚿 = A𝑒𝑖(π‘˜π‘₯βˆ’πœ”π‘‘)

= Aπ‘’π‘–π‘˜π‘₯ π‘’βˆ’πœ”π‘‘ ,

Where Aπ‘’π‘–π‘˜π‘₯ represent the time independent part of the wave function and is

represented by πœ“ = Aπ‘’π‘–π‘˜π‘₯ ……(1) differentiating eqn (1) w.r.t β€˜π‘₯ β€˜ twice we get

π‘‘πœ“

𝑑π‘₯= 𝐴(π‘–π‘˜) π‘’π‘–π‘˜π‘₯ and

𝑑2πœ“

𝑑π‘₯2 =A(π‘–π‘˜) (π‘–πœ”) π‘’π‘–π‘˜π‘₯

= 𝑖2 π‘˜2 Aπ‘’π‘–π‘˜π‘₯ …….(2)

From eqns 1 & 2 we get

𝑑2πœ“

𝑑π‘₯2 = βˆ’

4πœ‹2

πœ†2 πœ“ ……..(3) ∡ π’ŠπŸ = βˆ’1 & π’Œ =

πŸπ…

𝝀

But, de-broglie wave length Ξ» = β„Ž

π‘šπ‘£

∴ 1

πœ†2=

π‘š2𝑣2

β„Ž2

= 2π‘š(Β½π‘šπ‘£2)

β„Ž2

= 2π‘š(𝐸𝐾)

β„Ž2 ∡ 𝐸𝐾 = Β½π‘šπ‘£2

Also , the kinetic energy (𝐸𝐾 )in terms of the total energy (E) & the potential energy( V) is

given by 𝐸𝐾=(Eβˆ’π‘½)

∴ 1

πœ†2=

2π‘š(πΈβˆ’π‘½)

β„Ž2 ……(4)

From eqns 3 & 4 ,we get 𝑑2πœ“

𝑑π‘₯2= βˆ’

4πœ‹22π‘š(πΈβˆ’π‘½)

β„Ž2 πœ“

𝑑2πœ“

𝑑π‘₯2+

8πœ‹2π‘š(πΈβˆ’π‘½)

β„Ž2πœ“ = 0 ……(5)

This is Schrodinger time independent equation for a particle

For a free particle ,V=0

∴ 𝑑2πœ“

𝑑π‘₯2+

8πœ‹2π‘šπΈ

β„Ž2πœ“ = 0

This is Schrodinger time independent equation for free particle.

Page 12: AND ANSWERS SEM-I/II

REDDAPPA C MODULE-1 INTERLINE PUBLISHING.COM

VTU ENGINEERING PHYSICS ( 17 PHY 12/22) SEM-I/II Page 10

Q: Obtain normalized wave function for a free particle in a infinite walled potential

Box/Well using Schrodinger 1D time independent equation.

a

∞ ∞

V=∞ V=0

Box/Well

particle

π‘₯=0 π‘₯=a

Consider a particle of mass β€˜m’ moving by reflection at infinitely high walls of a Box/well

of width β€˜a’ moving between π‘₯=0 & π‘₯=a Potential V=0 inside the Box/well and V=∞

outside the Box/well .one dimensional Schrodinger equation for particle is given by

𝑑2πœ“

𝑑π‘₯2+

8πœ‹2π‘š(πΈβˆ’π‘½)

β„Ž2πœ“ = 0 ……(1).

For a particle inside the Box/well V=0

∴ 𝑑2πœ“

𝑑π‘₯2+

8πœ‹2π‘šπΈ

β„Ž2πœ“ = 0 ….(2)

Putting 8πœ‹2π‘šπΈ

β„Ž2= 𝐾2 …..(3) in equation (2) ,

we get 𝑑2πœ“

𝑑π‘₯2+ 𝐾2πœ“ = 0 ……(4)

The general solution of the quadratic equation (4) is of the form ψ (π‘₯)=A sin(Kπ‘₯) +B Cos(Kπ‘₯)

……(5)

where A & B are constants determined from boundary conditions as follows :

ψ (π‘₯)=0 at π‘₯=0 from eqn (5), 0=A x 0 +B x 1

∴ B=0

also, ψ (π‘₯)=0 at π‘₯=a ∴ from eqn(4) 0=C Sin(Ka)+0xCos(Ka)

0= C Sin(Ka)

Sin(Ka) = 0 as C≠ 0

∴ Sin(Ka) =0= Sin(nπœ‹)

ie: Ka=nπœ‹ or K=π‘›πœ‹

π‘Ž …..(6)

From eqns 3 & 6 ,we get 8πœ‹2π‘šπΈ

β„Ž2=

𝑛2πœ‹2

π‘Ž2

∴ E = 𝑛2β„Ž2

8π‘Ž2π‘š or

In general En= 𝑛2β„Ž2

8π‘Ž2π‘š …(7)

Page 13: AND ANSWERS SEM-I/II

REDDAPPA C MODULE-1 INTERLINE PUBLISHING.COM

VTU ENGINEERING PHYSICS ( 17 PHY 12/22) SEM-I/II Page 11

where n=1,2,3..called quantum number.

The values of En are called Eigen energy values which satisfy Schrodinger wave equation.

n=1 gives E1 = β„Ž2

8π‘Ž2π‘š = Eo called end point /ground state energy .

n=2 gives E2 = 4β„Ž2

8π‘Ž2π‘š = 4 Eo called 1st excited state energy

n=3 gives E3 = 9β„Ž2

8π‘Ž2π‘š =9 Eo ,called 2nd excited state energy & so on

Substituting the values of B=0 & K= π‘›πœ‹

π‘Ž in eqn 5, we get

πœ“π‘›(π‘₯) = A Sin( π‘›πœ‹π‘₯

π‘Ž) …….(8) this represents the permitted solutions

To find ’A’ by normalization:

Applying the normalization condition ∫ 𝐼𝛹𝐼2𝑑π‘₯ = 1+∞

βˆ’βˆž to eqn 8 for x=0 & x=a, we get

∫ 𝐴2 𝑆𝑖𝑛2 ( π‘›πœ‹π‘₯

π‘Ž)𝑑π‘₯ = 1

π‘Ž

0

𝐴2 ∫ Β½[1 βˆ’ πΆπ‘œπ‘  ( 2 π‘›πœ‹π‘₯

π‘Ž)]𝑑π‘₯ = 1

π‘Ž

0 ∡ 𝑆𝑖𝑛2 (πœƒ)=Β½[1-Cos(2πœƒ)]

½𝐴2 [∫ 1𝑑π‘₯ βˆ’ ∫ πΆπ‘œπ‘ ( 2 π‘›πœ‹π‘₯

π‘Ž)𝑑π‘₯

π‘Ž

0 ] = 1

π‘Ž

0

½𝐴2 [π‘₯]0π‘Ž βˆ’ [

π‘Ž

2π‘›πœ‹ Sin(

2π‘›πœ‹π‘₯

π‘Ž)]0

π‘Ž = 1 ∡ ∫ πΆπ‘œπ‘ (π‘šπ‘₯) 𝑑π‘₯ = 𝑆𝑖𝑛(π‘šπ‘₯)

π‘š

½𝐴2 [π‘Ž βˆ’ 0] βˆ’ [ π‘Ž

2π‘›πœ‹ Sin(2nπœ‹) βˆ’

π‘Ž

2π‘›πœ‹ Sin(0)] =1

½𝐴2 [π‘Ž βˆ’ 0] βˆ’ [0 βˆ’ 0] =1

𝐴2 π‘Ž = 2 𝐴2 = 2/π‘Ž or A =√𝟐/𝒂 …(9)

From eqns 8 &9, the normalized Eigen functions are 𝑔𝑖𝑣𝑒𝑛 𝑏𝑦 𝝍𝒏(π‘₯)= √𝟐/𝒂 Sin( 𝒏𝝅𝒙

𝒂) …..(10)

𝑛 = 3 𝝍3 𝐼𝝍3𝐼2

.

𝑛 = 2 𝝍2 𝐼𝝍2𝐼2

𝑛 = 1 𝝍1 𝐼𝝍1𝐼2

π‘₯=0 π‘₯=a π‘₯=0 π‘₯=a

𝐸𝑖𝑔𝑒𝑛 π‘“π‘’π‘›π‘π‘‘π‘–π‘œπ‘›π‘  𝝍1 ,

𝝍2 ,𝝍3 , … . π‘Žπ‘›π‘‘ π‘‘β„Žπ‘’π‘–π‘Ÿ π‘π‘Ÿπ‘œπ‘π‘Žπ‘π‘–π‘™π‘–π‘‘π‘¦ 𝑑𝑒𝑛𝑠𝑖𝑑𝑖𝑒𝑠 𝐼𝝍1𝐼2, 𝐼𝝍2𝐼2, 𝐼𝝍3𝐼2 …

are represented as shown in the diagrams.The probability of finding the particle at the

anti-nodes is maximum and at nodes is zero.(The particle never found at nodes)

NOTE: Eigen Functions are the acceptable wave functions [πœ“π‘›(π‘₯)]

Eigen Energy values are energy values for which Schrodinger equation

can be solved.

This can be omitted

Page 14: AND ANSWERS SEM-I/II

REDDAPPA C MODULE-1 INTERLINE PUBLISHING.COM

VTU ENGINEERING PHYSICS ( 17 PHY 12/22) SEM-I/II Page 12

Q: What is a wave function and mention it’s properties/limitations

The variable quantity that characterizes the de-broglie wave of the particle is called a

β€˜Wave function’ denoted mathematically by the symbol β€˜πšΏβ€™(Psi)

Properties of the wave functions are:-

1. Wave function (𝚿) is single valued everywhere ,

2. 𝚿 is finite everywhere

3. 𝚿 is continuous every where

4. First derivatives of 𝚿 are continuous every where

5. 𝐼𝛹𝐼2 or 𝛹𝛹 * is called probability density.

6.π‘ƒπ‘Ÿπ‘œπ‘π‘Žπ‘π‘–π‘™π‘–π‘‘π‘¦ π‘œπ‘“ 𝑓𝑖𝑛𝑑𝑖𝑛𝑔 π‘π‘Žπ‘Ÿπ‘‘π‘–π‘π‘™π‘’ 𝑖𝑛 π‘ π‘π‘Žπ‘π‘’ 𝑖𝑠 𝑔𝑖𝑣𝑒𝑛 𝑏𝑦 ∫ 𝐼𝛹𝐼2𝑑𝑉 = 1+∞

βˆ’βˆž

PROBLEMS SECTION

MODULE-1 : MODERN PHYSICS AND QUANTUM MECHANICS

Formulae needed : Ξ» = β„Ž

𝑃 where P = mv = √2π‘šπΈ =√2π‘šπ‘’π‘‰ ,

E = 1

2π‘šπ‘‰2 =

𝑃2

2π‘š ;Photon energy E’ = hΞ½ =

β„ŽπΆ

πœ† ; βˆ†π‘₯. βˆ†π‘ β‰₯

β„Ž

4πœ‹ and

βˆ†π‘ = π‘š. βˆ†π‘£ ; βˆ†πΈ. βˆ†π‘‘ β‰₯β„Ž

4πœ‹ π‘Žπ‘›π‘‘ βˆ†πΈ =

β„ŽπΆβˆ†πœ†

πœ†2 ; also βˆ†πΈ = β„Ž βˆ†πœˆ ;

P = ∫ 𝐼𝛹𝐼2π‘₯2

π‘₯1dx ; 𝐸𝑛 =

𝑛2β„Ž2

8π‘šπ‘Ž2

1. Compare the energy of a photon with that of an electron when both are associated with

wavelength 0.2 nm.(Dec 2014/Jan2015)

Given: πœ†π‘ƒ = πœ†π‘’ = 0.2 π‘›π‘š = 0.2π‘₯10βˆ’9 π‘š , h=6.63x10βˆ’34 Js, m=9.1x10βˆ’31kg ; C=3x108m/s,

𝐸𝑃

𝐸𝑒= ?

Using 𝐸𝑃 =β„ŽπΆ

πœ†π‘ƒ and 𝐸𝑒 =

β„Ž2

2π‘šπœ†π‘’2

We get , 𝐸𝑃

𝐸𝑒 =

𝐢2π‘šπœ†π‘’2

β„Žπœ†π‘ƒ

= 3π‘₯108π‘₯2π‘₯9.1π‘₯10βˆ’31( 0.2π‘₯10βˆ’9)2

6.63π‘₯10βˆ’34 π‘₯ 0.2π‘₯10βˆ’9

= 1.647x 102

Page 15: AND ANSWERS SEM-I/II

REDDAPPA C MODULE-1 INTERLINE PUBLISHING.COM

VTU ENGINEERING PHYSICS ( 17 PHY 12/22) SEM-I/II Page 13

2. Calculate the kinetic energy of an electron of wavelength 18 nm.(Jun/Jul 14)

Given: πœ† = 18 π‘›π‘š = 18π‘₯10βˆ’9 π‘š , h=6.63x10βˆ’34 Js, m=9.1x10βˆ’31kg ; E= ?

Using Ξ» = β„Ž

√2π‘šπΈ

We get ,𝐸 =β„Ž2

2π‘šπœ†2

= ( 6.63π‘₯10βˆ’34 )2

2π‘₯9.1π‘₯10βˆ’31π‘₯( 18π‘₯10βˆ’9)2

= 7.454x10βˆ’22 J

=7.454π‘₯10βˆ’22

1.6π‘₯10βˆ’19

= 4.66x10βˆ’3 eV

3. An excited atom has an average life time of 10βˆ’8 seconds. During this period. it emits

photon and returns to the ground state. What is the minimum uncertainty in the frequency

of this photon ? (Jun/Jul 14)

Given: βˆ†π‘‘ = 10βˆ’8 S, β„Ž = 6.63π‘₯10βˆ’34 𝐽𝑠 ; βˆ†πœˆ = ?

Using βˆ†πΈ = β„Ž. βˆ†πœˆ and βˆ†πΈ. βˆ†π‘‘ β‰₯β„Ž

4πœ‹

We get , βˆ†πœˆ β‰₯1

4πœ‹.βˆ†π‘‘

β‰₯1

4π‘₯πœ‹π‘₯10βˆ’8

= 7.96 x 106 Hz

4. Calculate the wavelength associated with electrons whose speed is 0.01 part

of the speed of light. (Dec 2013/Jan2014)

Given: v =0.01 C = 0.01x 3x108m/s ; h=6.63x10βˆ’34 Js; π‘š = 9.1π‘₯10βˆ’31π‘˜π‘” ; πœ† = ?

Using Ξ» = β„Ž

π‘šπ‘£

= 6.63π‘₯10βˆ’34

9.1π‘₯10βˆ’31π‘₯0.01π‘₯ 3π‘₯108

= 2.43 x10βˆ’10 m

Page 16: AND ANSWERS SEM-I/II

REDDAPPA C MODULE-1 INTERLINE PUBLISHING.COM

VTU ENGINEERING PHYSICS ( 17 PHY 12/22) SEM-I/II Page 14

5. An electron is bound in one dimensional infinite well of width 0.12nm.Find the

energy value and de-Broglie wavelength in the first excited state. (Dec 2013/Jan2014)

Given: a = 0.12nm = 0.12x10βˆ’9 m ; n = 2 ; h=6.63x10βˆ’34 Js; π‘š = 9.1π‘₯10βˆ’31π‘˜π‘” ;

𝐸2 = ? & λ =?

Using 𝐸𝑛 = 𝑛2β„Ž2

8π‘šπ‘Ž2

𝐸2 = 22(6.63π‘₯10βˆ’34 )2

8π‘₯9.1π‘₯10βˆ’31π‘₯(0.12π‘₯10βˆ’9)2

= 1.68x10βˆ’17 J

Also , Ξ» = β„Ž

√2π‘šπΈ2

= 6.63π‘₯10βˆ’34

2π‘₯9.1π‘₯10βˆ’31π‘₯1.68π‘₯10βˆ’17

= 1.19 x10βˆ’10 m

6. Calculate the de-Broglie wavelength associated with an electron of energy 1.5eV. (Jun13)

Given: E=1.5 eV=1.5x1.6x10βˆ’19 𝐽 , h=6.63x10βˆ’34 Js; m=9.1x10βˆ’31kg; e=1.6x10βˆ’19 C; Ξ» =

Using Ξ» = β„Ž

√2π‘šπΈ

= 6.63π‘₯10βˆ’34

√(2π‘₯9.1π‘₯10βˆ’31π‘₯1.5π‘₯1.6π‘₯10βˆ’19 )

= 1.003 x 10βˆ’9 m

7. A spectral line of wavelength 5461Γ… has a width of 10βˆ’4Γ…. Evaluate the minimum time

spent by the electrons in the upper energy state. (Jun/Jul 2013)

Given:, β„Ž = 6.63π‘₯10βˆ’34 𝐽𝑠 ; πœ† = 5461Γ… = 5461π‘₯10βˆ’10 π‘š ; βˆ†π‘‘ = ?

Using βˆ†πΈ =β„ŽπΆ

πœ† and βˆ†πΈ. βˆ†π‘‘ β‰₯

β„Ž

4πœ‹

We get , βˆ†π‘‘ β‰₯πœ†

4πœ‹.𝐢

β‰₯5461π‘₯10βˆ’10

4π‘₯πœ‹π‘₯3π‘₯108

= 1.45 x 10βˆ’16 S

Page 17: AND ANSWERS SEM-I/II

REDDAPPA C MODULE-1 INTERLINE PUBLISHING.COM

VTU ENGINEERING PHYSICS ( 17 PHY 12/22) SEM-I/II Page 15

8. Find the de-Broglie wavelength of an electron accelerated through a potential difference

of 182 volts and object of mass 1 kg moving with a speed of 1 m/s. Compare the results

and comment. (Jun 2012)

Given:V=182V ; π‘š0 =1 kg, 𝑉0 = 1 m/s , h=6.63x10βˆ’34 Js; m=9.1x10βˆ’31kg; e=1.6x10βˆ’19 C ,

πœ†π‘’ = ? πœ†0 = ?

Using πœ†π‘’ =β„Ž

√2π‘šπ‘’π‘‰

=6.63π‘₯10βˆ’34

√2π‘₯9.1π‘₯10βˆ’31π‘₯1.6π‘₯10βˆ’19 π‘₯182

= 9.11 x10βˆ’11 m

Also,using πœ†0 = β„Ž

π‘š0𝑉0

= 6.63π‘₯10βˆ’34

1π‘₯1

= 6.63x10βˆ’34 π‘š

Comment : Waverlength is inversely proportional to the mass ,as πœ†π‘’ ≫ πœ†0

9. A quantum particle confined to one-dimensional box of width β€˜a’ is in its first excited

state. What is the probability of finding the particle over an interval of ( π‘Ž

2) marked

symmetrically at the centre of box. (𝐽𝑒𝑛

𝐽𝑒𝑙2011)

Given: 1st excited state , n=2 ;the interval ( π‘Ž

4 ,

3π‘Ž

4 )

Using P = ∫ 𝐼𝛹𝐼2π‘₯2

π‘₯1dx = ∫ (√

2

π‘Žπ‘†π‘–π‘› ( 2πœ‹π‘₯

π‘Ž))2𝑑π‘₯

3π‘Ž/4

π‘Ž/4π‘Ž/2

= ∫𝟐

𝒂𝑆𝑖𝑛2 ( 2πœ‹π‘₯

π‘Ž)𝑑π‘₯

3π‘Ž

4π‘Ž

4

0 a/4 a/2 3a/4 a

= 𝟐

𝒂 ∫ Β½[1 βˆ’ πΆπ‘œπ‘  ( 2 πœ‹π‘₯

π‘Ž)]𝑑π‘₯

3π‘Ž

4π‘Ž

4

∡ 𝑆𝑖𝑛2 (πœƒ)=Β½[1-Cos(2πœƒ)]

= ½𝟐

𝒂 ∫ 1𝑑π‘₯ βˆ’ ∫ πΆπ‘œπ‘ ( 2 πœ‹π‘₯

π‘Ž)𝑑π‘₯

3π‘Ž

4π‘Ž

4

3π‘Ž

4π‘Ž

4

= π‘Ž [π‘₯]π‘Ž

4

3π‘Ž

4 βˆ’ [ π‘Ž

2πœ‹ Sin(

2πœ‹π‘₯

π‘Ž)]π‘Ž

4

3π‘Ž

4

= π‘Ž [3π‘Ž4

βˆ’ π‘Ž4

] βˆ’ [ π‘Ž 2πœ‹

Sin( 2πœ‹

3π‘Ž

4

π‘Ž)βˆ’

π‘Ž

2πœ‹ Sin(

2πœ‹π‘Ž

4

π‘Ž)]

= π‘Ž [π‘Ž

2] βˆ’ [0 βˆ’ 0 = Β½ = 0.5 or 50%

Page 18: AND ANSWERS SEM-I/II

REDDAPPA C MODULE-1 INTERLINE PUBLISHING.COM

VTU ENGINEERING PHYSICS ( 17 PHY 12/22) SEM-I/II Page 16

10. A particle moving in one dimension box is described by the wave function 𝛹 = π‘₯[√3] π‘“π‘œπ‘Ÿ

0<π‘₯<1 and Ξ¨ =0 elsewhere. Find the probability of finding the particle within the

interval (0,Β½). (𝐽𝑒𝑛 2012)

Given : 𝛹 = π‘₯[√3] ; (π‘₯1, π‘₯2)( 0,1

2 ) ; P = ?

Using Probability, P = ∫ 𝐼𝛹𝐼2π‘₯2

π‘₯1dx

= ∫ 3π‘₯21

2

0dx = [3.

π‘₯3

3]0

1

2

= ( 1

2 )3 – (0)3 =

1

8 = 0.125 or 12.5 %

11. Calculate the energy of electron that produces Bragg’s diffraction of first order at glancing

angle of 22Β° ,when incident on crystal with inter-planar spacing of 1.8Γ…. (𝐽𝑒𝑛 2012)

Given : n =1 ; πœƒ = 22Β° ; 𝑑 = 1.8 Γ… = 1.8π‘₯ 10βˆ’10 m ; h=6.63x10βˆ’34 Js; m=9.1x10βˆ’31kg;

Ξ» = ? & E = ?

Using 1 ) Ξ» = 2𝑑 𝑠𝑖𝑛 πœƒ

𝑛

= 2π‘₯1.8π‘₯ 10βˆ’10π‘₯ 𝑠𝑖𝑛 22

1

=1.348x 10βˆ’10 m

Also from, Ξ» = β„Ž

√2π‘šπΈ

We get, E = β„Ž2

2π‘šπœ†2 J OR E =

β„Ž2

2π‘šπœ†2 𝑒 eV

= (6.63π‘₯10βˆ’34)2

2π‘₯9.1π‘₯10βˆ’31π‘₯(1.348π‘₯ 10βˆ’10 )2 =

(6.63π‘₯10βˆ’34)2

2π‘₯9.1π‘₯10βˆ’31π‘₯(1.348π‘₯ 10βˆ’10 )2π‘₯1.6 π‘₯ 10βˆ’19

= 1.329 x 10βˆ’17 J = 83.07 eV = 83.07 eV

12. Find the energy of the neutron in eV whose de-Broglie wavelength is 1Γ…. (𝐷𝑒𝑐 2011)

Given; Ξ» = 1Γ… = 1x 10βˆ’10 π‘š ;h=6.63x10βˆ’34 Js, m=1.678x10βˆ’27kg ; e=1.6x10βˆ’19 C ;E = ?

Using , Ξ» = β„Ž

√2π‘šπΈ

We get , E = β„Ž2

2π‘šπœ†2 joules

= β„Ž2

2π‘šπ‘’πœ†2 eV

= (6.63π‘₯10βˆ’34 )2

2π‘₯1.678π‘₯10βˆ’27π‘₯1.6π‘₯10βˆ’19 π‘₯(1π‘₯ 10βˆ’10 )2

= 0.082 eV

Page 19: AND ANSWERS SEM-I/II

REDDAPPA C MODULE-1 INTERLINE PUBLISHING.COM

VTU ENGINEERING PHYSICS ( 17 PHY 12/22) SEM-I/II Page 17

13. An electron is confined to a box of length 10βˆ’9 m, calculate the minimum uncertainty in it’s

velocity. (𝐷𝑒𝑐 2011)

Given: βˆ†π‘₯ = 10βˆ’9 m ; h=6.63x10βˆ’34 Js, m=9.1x10βˆ’31kg ;βˆ†π‘£ = ?

Using βˆ†π‘₯. βˆ†π‘ β‰₯β„Ž

4πœ‹ and βˆ†π‘ = π‘š. βˆ†π‘£

We get , βˆ†π‘£ β‰₯β„Ž

4πœ‹π‘šβˆ†π‘₯

β‰₯ 6.63π‘₯10βˆ’34

4πœ‹ π‘₯9.1π‘₯10βˆ’31π‘₯10βˆ’9

= 58 x103m/S

14. Compute the de-Broglie wavelength for a neutron moving with one tenth part of the

velocity of light. (𝐽𝑒𝑛

𝐽𝑒𝑙2011)

Given; V = 𝐢

10 =

3 𝑋 108

10= 0.3 π‘₯ 108 π‘š/𝑠;h=6.63x10βˆ’34 Js, m=1.678x10βˆ’27kg ;

C = 3 X 108 m/s ; Ξ» = ?

Using Ξ» = β„Ž

π‘šπ‘‰

= 6.63π‘₯10βˆ’34

1.678π‘₯10βˆ’27π‘₯0.3 π‘₯ 108 = 1.317x 10βˆ’14 m.

15. An electron has a de-Broglie wavelength 3 nm and rest mass 511keV. Determine its group

velocity and kinetic energy.

Given: Ξ» = 3nm = 3x 10βˆ’9 m ; h=6.63x10βˆ’34 Js ; 𝑉𝑔 = V = ? and 𝐸𝐾 = ?

m =E/ 𝐢2 =511x103x1.6x 10βˆ’19/(3π‘₯108)2 kg = 9.08x10βˆ’31 kg

Using Ξ» = β„Ž

π‘šπ‘£ ,we get 𝑉𝑔 = V =

β„Ž

π‘šπœ†

= 6.63π‘₯10βˆ’34

9.08π‘₯10βˆ’31π‘₯3π‘₯ 10βˆ’9

= 2.43x 106 m/s

π‘Žπ‘™π‘ π‘œ, 𝐸𝐾 =1

2π‘šπ‘‰2 =

1

2π‘₯ 9.08π‘₯10βˆ’31 π‘₯ (2.43π‘₯ 106 )2

= 2.681x 10βˆ’18 J

Page 20: AND ANSWERS SEM-I/II

REDDAPPA C MODULE-1 INTERLINE PUBLISHING.COM

VTU ENGINEERING PHYSICS ( 17 PHY 12/22) SEM-I/II Page 18

16. A spectral line of wavelength 546.1 nm has a width 10βˆ’5 nm. Estimate the minimum time

spent by electrons in the excited state during transitions. (𝐷𝑒𝑐 2010)

Given:, β„Ž = 6.63π‘₯10βˆ’34 𝐽𝑠 ; πœ† = 546.1 π‘›π‘š = 546.1π‘₯10βˆ’9 π‘š ; βˆ†π‘‘ = ?

Using βˆ†πΈ =β„ŽπΆ

πœ† and βˆ†πΈ. βˆ†π‘‘ β‰₯

β„Ž

4πœ‹

𝑀𝑒 𝑔𝑒𝑑 , βˆ†π‘‘ β‰₯πœ†

4πœ‹.𝐢

β‰₯546.1π‘₯10βˆ’9

4πœ‹π‘₯3π‘₯108 = 1.45x 10βˆ’16 S

17. Calculate the momentum of the particle and de-Broglie wavelength associated with an

electron with a kinetic energy of 1.5KeV. (𝐽𝑒𝑛 2010)

Given: E =1.5 Kev =1.5 x 103 x 1.6x10βˆ’19 J ; h = 6.625π‘₯10βˆ’34 𝐽𝑆 ;e = 1.6x10βˆ’19 J ;

m = 9.1π‘₯10βˆ’31 π‘˜π‘” P =? & Ξ» = ?

Ans: Using P = √2π‘šπΈ

= √2π‘₯9.1π‘₯10βˆ’31π‘₯1.5 π‘₯ 103 π‘₯ 1.6π‘₯10βˆ’19 = 2.09 x 10βˆ’23 kg.m/s

Also, Ξ» = β„Ž

𝑃

= 6.625π‘₯10βˆ’34

2.09 π‘₯ 10βˆ’23 = 3.17x10βˆ’11 π‘š

18. An electron is bound in one dimensional potential well of width 0.18 nm. Find the energy

value in eV of the second excited state. (𝐽𝑒𝑛 2010 & 𝐢𝐡𝐢𝑆 βˆ’ 𝐽𝑒𝑛/𝐽𝑒𝑙16)

Given: n=3(For 2nd excited state), a=0.18 nm =0.18x10βˆ’9m,h=6.63x10βˆ’34 Js;

m=9.11x10βˆ’31kg & e=1.6x10βˆ’19 C; 𝐸2 = ?

Using 𝐸𝑛 =𝑛2β„Ž2

8π‘šπ‘Ž2 J OR 𝐸𝑛 =

𝑛2β„Ž2

8π‘šπ‘Ž2𝑒 eV

𝐸2 = 32(6.63π‘₯10βˆ’34 )2

8π‘₯9.11π‘₯10βˆ’31π‘₯(0.18 π‘₯10βˆ’9)2 𝐸2 =

32(6.63π‘₯10βˆ’34 )2

8π‘₯9.11π‘₯10βˆ’31π‘₯(0.18 π‘₯10βˆ’9)2 π‘₯1.6π‘₯10βˆ’19

= 1.675 x10βˆ’17 J = 104.7 eV

=1.675 π‘₯10βˆ’17

1.6π‘₯10βˆ’19 =104.7 eV

Page 21: AND ANSWERS SEM-I/II

REDDAPPA C MODULE-1 INTERLINE PUBLISHING.COM

VTU ENGINEERING PHYSICS ( 17 PHY 12/22) SEM-I/II Page 19

19. Calculate the energy in eV, for the first excited state of an electron in an infinite potential

well of width 2 Γ… (4 marks)

Given: n=2(For 1st excited state), a=2Γ…=2x10βˆ’10m,h=6.63x10βˆ’34 Js; m=9.1x10βˆ’31kg &

e=1.6x10βˆ’19 C;

Using 𝐸𝑛 =𝑛2β„Ž2

8π‘šπ‘Ž2 J OR 𝐸𝑛 =

𝑛2β„Ž2

8π‘šπ‘Ž2𝑒 eV

=22(6.63π‘₯10βˆ’34 )2

8π‘₯9.1π‘₯10βˆ’31π‘₯(2π‘₯10βˆ’10)2 =

22(6.63π‘₯10βˆ’34 )2

8π‘₯9.1π‘₯10βˆ’31π‘₯(2π‘₯10βˆ’10)2 π‘₯1.6π‘₯10βˆ’19

= 6.038x10βˆ’18 J = 37.74 eV

=6.038π‘₯10βˆ’18

1.6π‘₯10βˆ’19 =37.74 eV

20. The wavelength of a fast neutron of mass 1.675x10βˆ’27kg is 0.02 nm.Calculate the group

velocity and phase velocity of its de-Broglie waves.( 4 marks)

Given: m=1.675x10βˆ’27kg, Ξ» = 0.02 nm=0.02x10βˆ’9m, C=3x108m/s, β„Ž = 6.63π‘₯10βˆ’34 𝐽𝑠,

𝑣𝑔 & 𝑣𝑃 =?

π‘ˆπ‘ π‘–π‘›π‘”, 𝑣𝑔 = v =β„Ž

π‘šπœ† =

6.63π‘₯10βˆ’34

1.675π‘₯10βˆ’27π‘₯0.02π‘₯10βˆ’9 = 1.979x104 m/s ,

Also 𝑣𝑃 =𝐢2

𝑣𝑔 =

(3π‘₯108)2

1.979π‘₯104 = 4.55X1012 m/s

21. Calculate the de-Broglie wavelength associated with neutron of mass 1.674 X 10βˆ’27 kg

with one tenth part of the velocity of light (4 marks - CBCS Jun/July16)

Given : v = 1

10 C =

1

10 x 3x108m/s = 3x107m/s; h=6.63x10βˆ’34 Js; π‘š = 9.11π‘₯10βˆ’31π‘˜π‘” ; πœ† = ?

Using Ξ» = β„Ž

π‘šπ‘£

= 6.63π‘₯10βˆ’34

9.11π‘₯10βˆ’31π‘₯ 3π‘₯107

= 2.43 x10βˆ’11 m

Page 22: AND ANSWERS SEM-I/II

REDDAPPA C MODULE-1 INTERLINE PUBLISHING.COM

VTU ENGINEERING PHYSICS ( 17 PHY 12/22) SEM-I/II Page 20

22. Calculate the deBroglie wavelength of an electron moving with K.E of 50KeV.

Given: E= 50keV= 5x103π‘₯ 1.6x10βˆ’19 𝐽 , h=6.63x10βˆ’34 Js; m=9.1x10βˆ’31kg;

e=1.6x10βˆ’19 C; Ξ» =?

Using Ξ» = β„Ž

√2π‘šπΈ

= 6.63π‘₯10βˆ’34

√(2π‘₯9.1π‘₯10βˆ’31π‘₯ 50π‘₯103π‘₯ 1.6π‘₯10βˆ’19 )

= 5.495 x 10βˆ’12 m

22. X-rays of wavelength 0.75Γ… are scattered from a target at an angle of 45Β°.

Calculate the wavelength of scattered X-rays.

Given: πœ† = 0.75Γ… ; πœƒ = 45Β° ; h=6.63x10βˆ’34 Js; m=9.1x10βˆ’31kg ; C=3x108m/s ;𝝺λ’ = ?

Using , Ξ»β€™βˆ’πœ† = β„Ž

π‘šπΆ (1βˆ’π‘π‘œπ‘ πœƒ)

λ’ βˆ’0.75Γ… = 6.63π‘₯10βˆ’34

9.1π‘₯10βˆ’31π‘₯ 3π‘₯108 (1βˆ’π‘π‘œπ‘ 45)

λ’ = 0.75Γ… + 7.1x10βˆ’13m

= 0.75Γ… + 0.0071Γ…

= 0.7571Γ…

223. A particle of mass 940 MeV/𝐢2 has kinetic energy 0.5 KeV. Find its de-

Broglie wavelength , C is velocity of light.

Given: m = 940 MeV/𝐢2 = 940 π‘₯ 106 𝑒𝑉

(3π‘₯108 )2 =

940 π‘₯ 106 π‘₯1.6π‘₯10βˆ’19 𝐽

(3π‘₯108 )2 = 1.671x10βˆ’27 π‘˜π‘”

E = 0.5 keV = 0.5 π‘₯103eV = 0.5 π‘₯103π‘₯1.6π‘₯10βˆ’19 𝐽 ; πœ† = ?

Using; 𝝺 = β„Ž

√(2π‘šπΈ)

= 6.63π‘₯10βˆ’34

√(2π‘₯1.671π‘₯10βˆ’27π‘₯0.5 π‘₯103π‘₯1.6π‘₯10βˆ’19)

= 1.282x10βˆ’12 m

Page 23: AND ANSWERS SEM-I/II

REDDAPPA C MODULE-1 INTERLINE PUBLISHING.COM

VTU ENGINEERING PHYSICS ( 17 PHY 12/22) SEM-I/II Page 21

24. The first excited state energy of an electron in an infinite well is 240 eV. what will be its ground state energy when the width of the potential

Well is doubled.

Given: 𝐸2 = 240 eV ; 𝐸1 = ?

Using 𝐸𝑛 =𝑛2β„Ž2

8π‘šπ‘Ž2

𝐸2 =22β„Ž2

8π‘šπ‘Ž2

240 =4β„Ž2

8π‘šπ‘Ž2

π‘Ž2 =4β„Ž2

8π‘šπ‘₯240

∴ a = √(β„Ž2

480π‘š )

π‘Žπ‘™π‘ π‘œ , 𝐸1 =12β„Ž2

8π‘š(2√(β„Ž2

480π‘š ) )2

= 480π‘šπ‘₯β„Ž2

8π‘šπ‘₯4π‘₯β„Ž2 = 15 eV

𝑂𝑅

Using ; 𝐸𝑛 =𝑛2β„Ž2

8π‘šπ‘Ž2 ,we can show that

𝐸2 π‘Ž22

22 =

𝐸1 π‘Ž12

12 =

β„Ž2

8π‘š

Given; 𝐸2 = 240𝑒𝑉 , π‘Ž1 = 2π‘Ž2

∴ 𝐸1 = 𝐸2 π‘Ž2

2

π‘Ž1222

= 240π‘Ž2

2

4π‘Ž22π‘₯4

= 15 eV

25. Calculate the deBroglie wavelength associated with neutron of mass 1.674 x10βˆ’27kg with one tenth part of the velocity of light .

Given; V = 𝐢

10 =

3 𝑋 108

10= 0.3 π‘₯ 108 π‘š/𝑠;h=6.63x10βˆ’34 Js,

m=1.674x10βˆ’27kg ; C = 3 X 108 m/s ; Ξ» = ?

Using Ξ» = β„Ž

π‘šπ‘‰ =

6.63π‘₯10βˆ’34

1.674π‘₯10βˆ’27π‘₯0.3 π‘₯ 108

= 1.320x 10βˆ’14 m.

Page 24: AND ANSWERS SEM-I/II

REDDAPPA C MODULE-1 INTERLINE PUBLISHING.COM

VTU ENGINEERING PHYSICS ( 17 PHY 12/22) SEM-I/II Page 22

26. An electron is bound in one dimensional potential well of width

0.18 nm. Find the energy value in eV of the second excited state.

Given: n=3(For 2rd excited state), a=0.18 nm =0.18 x10βˆ’9m,h=6.63x10βˆ’34 Js;

m=9.1x10βˆ’31kg & e=1.6x10βˆ’19

Using 𝐸𝑛 =𝑛2β„Ž2

8π‘šπ‘Ž2 J OR 𝐸𝑛 =

𝑛2β„Ž2

8π‘šπ‘Ž2𝑒 eV

=32(6.63π‘₯10βˆ’34 )2

8π‘₯9.1π‘₯10βˆ’31π‘₯(0.18 π‘₯10βˆ’9)2 =

32(6.63π‘₯10βˆ’34 )2

8π‘₯9.1π‘₯10βˆ’31π‘₯1.6π‘₯10βˆ’19 π‘₯(0.18 π‘₯10βˆ’9)2

= 1.677x10βˆ’17 J = 104.83 eV

=6.038π‘₯10βˆ’18

1.6π‘₯10βˆ’19

= 104.81 eV

27. Find deBroglie wavelength of a particle of mass 0.58 MeV/π‘ͺ𝟐 has a kinetic

energy 90 eV , where C is the velocity of light. ( 04 Marks) DEC2016

Given: m = 0.58 MeV/𝐢2 = 0.58 π‘₯ 106 𝑒𝑉

(3π‘₯108 )2

=0.58 π‘₯ 106 π‘₯1.6π‘₯10βˆ’19 𝐽

(3π‘₯108 )2 = 1.0324x10βˆ’30 π‘˜π‘”

E = 90eV = 90π‘₯1.6π‘₯10βˆ’19 𝐽 ; πœ† = ?

Using; 𝝺 = β„Ž

√(2π‘šπΈ)

= 6.63π‘₯10βˆ’34

√(2π‘₯1.0324π‘₯10βˆ’30 π‘₯ 90π‘₯1.6π‘₯10βˆ’19)

= 1.2142π‘₯10βˆ’10 m

28. The inherent uncertainty in the measurement of time spent by Iridium- 19

nuclei in the excited state is found to be 1.4 x πŸπŸŽβˆ’πŸπŸŽ S. Estimate the

uncertainty that results in its energy in eV in the excited State. ( 04 Marks)

Given:, β„Ž = 6.625π‘₯10βˆ’34 𝐽𝑠 ; βˆ†π‘‘ = 1.4 x πŸπŸŽβˆ’πŸπŸŽ S: βˆ†πΈ =?

Using βˆ†πΈ. βˆ†π‘‘ β‰₯β„Ž

4πœ‹ 𝑀𝑒 𝑔𝑒𝑑 , βˆ†πΈ β‰₯

β„Ž

4πœ‹.βˆ†π‘‘

β‰₯6.625π‘₯10βˆ’34

4πœ‹π‘₯𝟏.πŸ’ 𝒙 πŸπŸŽβˆ’πŸπŸŽ J

β‰₯ 6.625π‘₯10βˆ’34

4πœ‹π‘₯𝟏.πŸ’ 𝒙 πŸπŸŽβˆ’πŸπŸŽπ’™πŸ.πŸ”πŸŽπŸπ’™πŸπŸŽβˆ’πŸπŸ— eV

β‰₯ 2.351π‘₯ 10βˆ’6 eV

Page 25: AND ANSWERS SEM-I/II

REDDAPPA C MODULE-1 INTERLINE PUBLISHING.COM

VTU ENGINEERING PHYSICS ( 17 PHY 12/22) SEM-I/II Page 23

29. A spectral line of wavelength 5896 Γ… has a width of πŸπŸŽβˆ’πŸ“ Γ…. Evaluate the minimum

time spent by the electrons in the upper energy state between the excitation and

de-excitation processes. ( 04 Marks)

Given: 𝝀=5896 Γ… = 5896 xπŸπŸŽβˆ’πŸπŸŽ m ,βˆ†π€ = πŸπŸŽβˆ’πŸ“Γ… = πŸπŸŽβˆ’πŸ“ xπŸπŸŽβˆ’πŸπŸŽm ,

h = 6.625 x πŸπŸŽβˆ’πŸ‘πŸ’ JS , C = 3 x πŸπŸŽπŸ–π’Žπ’”βˆ’πŸ , βˆ†π’• = ?

Using βˆ†π‘¬. βˆ†π’• β‰₯ 𝒉

πŸ’π… and βˆ†π‘¬ =

𝒉𝒗 βˆ†π€

π€πŸ ∡ E =

𝒉𝒄

𝝀

We get , βˆ†π’• = π€πŸ

πŸ’π…π’„βˆ†π€

∴ βˆ†π’• = (πŸ“πŸ–πŸ—πŸ” π’™πŸπŸŽβˆ’πŸπŸŽ)𝟐

πŸ’π…π’™πŸ‘ 𝒙 πŸπŸŽπŸ–π’™πŸπŸŽβˆ’πŸ“ π’™πŸπŸŽβˆ’πŸπŸŽ

= 9.221 x πŸπŸŽβˆ’πŸ– S

29. Compare the energy of a photon with that of a neutron when both are associate

with a wavelength 0.25 nm,mass of neutron is 1.675 X πŸπŸŽβˆ’πŸπŸ• kg. ( 04 Marks)

Given : 𝝀𝒑 = 𝝀𝒏 = 𝟎. πŸπŸ“ π’π’Ž = 0.25 x πŸπŸŽβˆ’πŸ— m , π’Žπ’ = 1.675 X πŸπŸŽβˆ’πŸπŸ• kg

h = 6.625 x πŸπŸŽβˆ’πŸ‘πŸ’ JS , C = 3 x πŸπŸŽπŸ–π’Žπ’”βˆ’πŸ , 𝝀𝒑

𝝀𝒏 = ?

Using , 𝑬𝒑 =𝒉𝒄

𝝀 =

πŸ”.πŸ”πŸπŸ“ 𝒙 πŸπŸŽβˆ’πŸ‘πŸ’π’™πŸ‘ 𝒙 πŸπŸŽπŸ–

𝟎.πŸπŸ“ 𝒙 πŸπŸŽβˆ’πŸ— = 7.95 x πŸπŸŽβˆ’πŸπŸ” J

Also , 𝑬𝒏 =π’‰πŸ

πŸπ’Žπ€πŸ =

(πŸ”.πŸ”πŸπŸ“ 𝒙 πŸπŸŽβˆ’πŸ‘πŸ’)𝟐

πŸπ’™πŸ.πŸ”πŸ•πŸ“ 𝑿 πŸπŸŽβˆ’πŸπŸ•π’™ (𝟎.πŸπŸ“ 𝒙 πŸπŸŽβˆ’πŸ—)𝟐 = 2.096 xπŸπŸŽβˆ’πŸπŸ J

∴ 𝑬𝒑

𝑬𝒏=

πŸ•.πŸ—πŸ“ 𝒙 πŸπŸŽβˆ’πŸπŸ”

𝟐.πŸŽπŸ—πŸ” π’™πŸπŸŽβˆ’πŸπŸ = 3.793 x πŸπŸŽπŸ“

**end**

Page 26: AND ANSWERS SEM-I/II

REDDAPPA C MODULE-2 INTERLINE PUBLISHING.COM

VTU ENGINEERING PHYSICS (17 PHY12/22) SEM-I/II Page 1

MODULE-2:

ELECTRICAL PROPERTIES OF MATERIALS:

Q:Explain the terms: Drift velocity, Mean collision time, Mean free path & relaxation time.

a) Drift velocity(𝒗𝒅) is the average velocity with which the free electrons drift in a direction opposite to that of the applied electric field

b) Mean collision time(𝝉) is the average time elapsed between two consecutive collisions.

c) Mean free path(Ξ»)is the average distance travelled by the electron between two consecutive collisions.

d) Relaxation time(𝝉𝒓) is time during which the drift velocity is reduced to 1

𝑒 of its

velocity when the field is cut off.

Q:Obtain an expression for Drift velocity.

Let an electron of mass β€˜m’ ,charge β€˜e’ moves with drift velocity ’𝒗𝒅 β€˜ when an

electric field β€˜E β€˜ is applied to it. Then the driving force acting on the electron

F = βˆ’π‘’πΈ ……(1)

Also the retarding force acting on the electron F’= π‘šπ‘£π‘‘

𝜏 …..(2)

where 𝜏 = π‘šπ‘’π‘Žπ‘› π‘π‘œπ‘™π‘™π‘–π‘ π‘–π‘œπ‘› π‘‘π‘–π‘šπ‘’

For a system in steady state F’ = βˆ’πΉ

ie: π‘šπ‘£π‘‘

𝜏= 𝑒𝐸

∴ 𝒗𝒅 =𝒆𝝉𝑬

π’Ž

Q:Derive an expression for electrical conductivity and hence electrical resistivity.

The drift velocity of electron 𝒗𝒅 =𝒆𝝉𝑬

π’Ž ……(1)

Also from Ohm’s law electrical conductivity , 𝜎 =𝒋

𝑬 ..(2)where j=current density &

E=electric field But j = ne𝒗𝒅 … (πŸ‘)

∴ eqns 1&3 ,we get j = ne. 𝒆𝝉𝑬

π’Ž

∴ 𝒋

𝑬 =

π’π’†πŸπ‰

π’Ž ….(4)

From eqns 2 &4,we get 𝝈 = π’π’†πŸπ‰

π’Ž …..(5)

By definition electrical resistivity, 𝜌 =1

𝜎 …(6)

∴ from eqns 5&6, we get 𝝆 = π’Ž

π’π’†πŸπ‰

Page 27: AND ANSWERS SEM-I/II

REDDAPPA C MODULE-2 INTERLINE PUBLISHING.COM

VTU ENGINEERING PHYSICS (17 PHY12/22) SEM-I/II Page 2

Q:Write the assumptions of classical free electron theory(CFET) or Drude-Lorentz theory.

The assumptions of CFET are:-

1. All metals contains a large number of free electrons called conduction electrons .

2. The free electrons are called electron gas which have 3 degrees of freedom as they

are treated as gas molecules.

3. Free electrons obey laws of kinetic theory of gases and hence they have mean free

path, mean collision time, drift velocity.

4. Free electrons move in a constant electric or potential field due to positive

ion cores.

5. Electro-static forces between electron-ions and electron-electrons are negligible.

6. Free electrons move with drift velocity in a direction opposite to the direction

of the applied field.

7. Average kinetic energy of free electrons at temperature T is 3

2π‘˜π‘‡,

where k = Boltzmann constant.

Q:Explain the failures of classical free electron theory(CFET).

Failures of CFET are:-

1.Specific heat :According to CFET the molar specific heat of electron gas at constant

volume is given by 𝐢𝑣 =3

2𝑅 ,where R=universal gas constant. But experimentally

determined 𝐢𝑣 𝑖𝑠 given by 𝐢𝑣 = 10βˆ’4𝑅𝑇 ,where T=absolute temperature. Thus CFET fail to

explain the the dependence of 𝐢𝑣 π‘œπ‘› T and numerical constant.

2.Dependance of electrical conductivity (𝝈) on T:

According to CFET kinetic energy 1

2π‘šπ‘£2 =

3

2π‘˜π‘‡

∴ 𝑣 ∝ βˆšπ‘‡ …(1)

But mean collision time 𝜏 ∝ 1

𝑣

∴ 𝜏 ∝ 1

βˆšπ‘‡ … . (2) ∡ 𝑣 ∝ βˆšπ‘‡

Also from 𝜎 =𝑛𝑒2𝜏

π‘š ,we get 𝜎 ∝ 𝜏 … . (3)

From eqns 2 & 3 we get 𝝈 ∝ 𝟏

βˆšπ‘‡ ,but experimentally it has been observed that 𝝈 ∝

𝟏

𝑻 ,

thus CFET fail to explain the dependence of 𝜎 π‘œπ‘› 𝑇.

3.Dependance of β€²πˆβ€²π’π’ 𝒆𝒍𝒆𝒄𝒕𝒓𝒐𝒏 π’„π’π’π’„π’†π’π’•π’“π’‚π’•π’Šπ’π’ β€˜n’ :

πΉπ‘Ÿπ‘œπ‘š 𝜎 =𝑛𝑒2𝜏

π‘š , we get 𝜎 ∝ 𝑛 , according to CFET 𝝈 π‘œπ‘“ tri-atomic metals must be more

than that of di-atomic and mono-atomic metals but from experimental results, it has been

found that 𝝈 of mono-atomic metals of low n are more than that of di and tri-atomic metals

of large n .Thus CFET fail to explain dependence of 𝝈 on concentration n.

Page 28: AND ANSWERS SEM-I/II

REDDAPPA C MODULE-2 INTERLINE PUBLISHING.COM

VTU ENGINEERING PHYSICS (17 PHY12/22) SEM-I/II Page 3

4. Mean free path’ λ’:

Mean free path of Cu from CFET is given by Ξ»= π‘£πœ=2.85nm. But the experimentally

determined value of Ξ»=28.5 nm, which is 10 times more than that Ξ» of CFET.

Q:State/Mention the assumptions of (Sommerfeld’s)quantum free electron theory(QFET)

The assumptions of QFET are:

1.The energy of free electrons are quantized.

2. Free electrons obey Pauli’s exclusion principle.

3. The distribution of free electrons in energy levels is governed by Fermi-Dirac

statistics.

4. Free electrons move in uniform potential field due to ionic cores in a metal .

5. The electrostatic electron-ion attractions and electron-electron repulsions

are negligible.

6. Electrons are considered as wave like particles.

Q:Explain the success of QFET.

QFET explained the following experimental facts which were not explained by CFET.

1.Specific heat: According to QFET the molar specific heat is given by 𝐢𝑣 =2π‘˜

𝐸𝐹𝑅𝑇

where k = Boltzmann constant and 𝐸𝐹=Fermi energy. But 2π‘˜

𝐸𝐹 = 10βˆ’4 ∴ 𝐢𝑣 = 10βˆ’4 𝑅𝑇

which agrees with experimental value.

2.Dependance of electrical resistivity β€²πˆβ€² on β€˜T’(temperature):

We know that(WKT) 𝝈 = π’π’†πŸπ‰

π’Žβˆ— and 𝜏 =

πœ†

𝑣𝐹 where 𝑣𝐹=Fermi velocity, Ξ»=wavelength

∴ 𝝈 = π’π’†πŸ

π’Žβˆ—

𝝀

𝒗𝑭 ∴ 𝝈 ∝ 𝝀 … . . (1)

Also it has been shown that λ ∝1

𝐴∝

1

π‘Ÿ2∝

1

𝑇 Ξ» ∝

1

𝑇 …(2)

where A=area, r=radius & T=temperature.

∴ π‘“π‘Ÿπ‘œπ‘š π‘’π‘žπ‘›π‘  1&2,we get 𝝈 ∝𝟏

𝑻 which is the experimentally determined relation.

3.Dependance of β€²πˆβ€²π’π’β€²π’β€²:

Page 29: AND ANSWERS SEM-I/II

REDDAPPA C MODULE-2 INTERLINE PUBLISHING.COM

VTU ENGINEERING PHYSICS (17 PHY12/22) SEM-I/II Page 4

WKT conductivity 𝝈 = π’π’†πŸ

π’Žβˆ—

𝝀

𝒗𝑭 , thus 𝝈 𝑑𝑒𝑝𝑒𝑛𝑑𝑠 π‘π‘œπ‘‘β„Ž π‘œπ‘› 𝒏 π‘Žπ‘›π‘‘

𝝀

𝒗𝑭 .With the decreases of

atomicity, n decreases and 𝝀

𝒗𝑭 increases so that n .

.𝝀

𝒗𝑭 increases ,so that 𝝈 is more for

monoatomic metals than that of di&tri-atomic metals.This explains the conductivity of

mono-atomic metals is more than that of di & tri-atomic metals.

4.Mean free path obtained from QFET is about 28.5 nm which is experimentally

determined value.

Q:Distinguish /(Write the differences )between CFET and QFET. CFET QFET

1.Free electron energy levels are

continuous

1.Free electron energy levels are dis- continuous .

2.Free electrons may possess

same energy

2.No two electrons can possess same energy, as

they obey Pauli’s exclusion principle.

3.Distribution of electrons in

energy levels obeys Maxwell-

Boltzmann statistics

3.Distribution of electrons in energy levels

obeys Fermi-Dirac statistics

Q: Derive an expression for conductivity and hence resistivity based on QFET.

According to de-broglie hypothesis Ξ» = β„Ž

π‘šπ‘£ ,also Ξ» =

2πœ‹

π‘˜ where symbols have usual

significance. ∴ π‘šπ‘£ =β„Žπ‘˜

2πœ‹ , where v & k are vectors

differentiating this w.r.t β€˜t’,

We get m 𝒅𝒗

𝒅𝒕 =

𝒉

πŸπ…

π’…π’Œ

𝒅𝒕 ….(1)

If an electric field E is applied ,the electron experience a force m 𝒅𝒗

𝒅𝒕 = βˆ’π‘’πΈ …(2)

∴ π‘“π‘Ÿπ‘œπ‘š π‘’π‘žπ‘’π‘›π‘ 1&2, 𝑀𝑒𝑔𝑒𝑑 , 𝒉

πŸπ…

π’…π’Œ

𝒅𝒕 = βˆ’π‘’πΈ

∴ π‘‘π‘˜ = βˆ’2πœ‹

β„Žπ‘’πΈ 𝑑𝑑

𝑂𝑛 Integration between the limits 0 to t ,we get

∫ π‘‘π‘˜π‘‘

0 = βˆ’

2πœ‹

β„Žπ‘’πΈ ∫ 𝑑𝑑

𝑑0

k(𝑑) βˆ’ π‘˜(0) = βˆ’2πœ‹

β„Žπ‘’πΈ (𝑑 βˆ’ 0)

πœ•π‘˜ = βˆ’2πœ‹

β„Žπ‘’πΈ 𝑑

The Fermi sphere displace through πœ•π‘˜ in a direction opposite to the direction of

applied electric field E in a time β€˜t’ as shown in the diagram.

Page 30: AND ANSWERS SEM-I/II

REDDAPPA C MODULE-2 INTERLINE PUBLISHING.COM

VTU ENGINEERING PHYSICS (17 PHY12/22) SEM-I/II Page 5

If t = 𝜏 , π‘‘β„Žπ‘’π‘› 𝑑𝑒𝑒 π‘‘π‘œ π‘π‘œπ‘™π‘™π‘–π‘ π‘–π‘œπ‘›π‘  π‘‘β„Žπ‘’ π‘šπ‘’π‘Žπ‘› π‘‘π‘–π‘ π‘π‘™π‘Žπ‘π‘’π‘šπ‘’π‘›π‘‘ πœ•π‘˜π‘Žπ‘£ π‘œπ‘“ π‘“π‘’π‘Ÿπ‘šπ‘– π‘ π‘β„Žπ‘’π‘Ÿπ‘’ is given

by πœ•π‘˜π‘Žπ‘£ = βˆ’2πœ‹

β„Žπ‘’πΈ 𝜏 ….(3),

Using mv = β„Ž

2πœ‹π‘˜

we get π‘š πœ•π‘£π‘Žπ‘£ = β„Ž

2πœ‹πœ•π‘˜π‘Žπ‘£

∴ πœ•π‘£π‘Žπ‘£ = β„Ž

2πœ‹π‘šπœ•π‘˜π‘Žπ‘£ ….(4)

From eqns 3&4,we get πœ•π‘£π‘Žπ‘£ = βˆ’π‘’πΈ

π‘š 𝜏

but πœ•π‘£π‘Žπ‘£ = 𝑣𝑑 drift velocity as initial average velocity of electrons is zero.

ie: 𝑣𝑑 = βˆ’π‘’πΈ

π‘š 𝜏 … (5)

Also current density J = βˆ’π‘›π‘’π‘£π‘‘ ….(6)

∴ π‘“π‘Ÿπ‘œπ‘š π‘’π‘žπ‘›π‘  5&6, we get J = 𝑛𝑒2𝜏

π‘š E

∴ 𝐽

𝐸 =

𝑛𝑒2𝜏

π‘š ….(7)

But from Ohm’s law conductivity 𝜎 = 𝐽

𝐸 …..(8)

∴ π‘“π‘Ÿπ‘œπ‘š π‘’π‘žπ‘›π‘  7&8 ,we get 𝝈 = π’π’†πŸπ‰

π’Ž …(7),

where m is called effective electron mass usually denoted by π’Žβˆ—

Also resistivity 𝜌 =1

𝜎 …(8)

∴ π‘“π‘Ÿπ‘œπ‘š π‘’π‘žπ‘›π‘  7&8 ,we get 𝝆 =π’Ž

π’π’†πŸπ‰

Q:Explain the terms: Mobility, Fermi velocity, Fermi temperature, Fermi Level &Fermienergy.

a) Mobility(ΞΌ) of electrons is defined as the drift velocity (𝒗𝒅)acquired by the electrons per unit

electric field(E). ie: ΞΌ = 𝒗𝒅

𝑬 ΞΌ =

𝝈

𝒏𝒆 ΞΌ =

𝒆𝝉

π’Ž

b) Fermi velocity(𝒗𝑭) of an electron is defined as it’s velocity when it’s energy is equal to the

Fermi energy. ie: 1

2π‘šπ‘£πΉ

2 = 𝐸𝐹 𝑣𝐹 = (√2πΈπΉπ‘š

)

c) Fermi temperature(𝑻𝑭) is defined as the temperature at which the average thermal

energy of the electron is equal to the Fermi energy at 0K. ie: 𝑻𝑭 =𝑬𝑭

π’Œ where k=Boltzmann

constant.

d) Fermi level is defined as the highest filled energy level in a metal at 0K.

e) Fermi energy(𝑬𝑭)is defined as the energy of the highest occupied level in a metal at 0K.

E

0

πœ•π‘˜

Page 31: AND ANSWERS SEM-I/II

REDDAPPA C MODULE-2 INTERLINE PUBLISHING.COM

VTU ENGINEERING PHYSICS (17 PHY12/22) SEM-I/II Page 6

Q:What is Fermi-Dirac statistics ? Explain.

Fermi –Dirac statistics is the statistical rule applied to the distribution of identical,

indistinguishable particles of spin 1

2 called fermions like electrons, which obey Pauli’s

exclusion principle. The probability of occupation of a state by an electron is given

by Fermi factor/Fermi- Dirac distribution function given by

f(E)= 1

1+𝑒(πΈβˆ’πΈπΉ)/π‘˜π‘‡

where 𝐸𝐹 = πΉπ‘’π‘Ÿπ‘šπ‘– π‘’π‘›π‘’π‘Ÿπ‘”π‘¦, E=energy of State, k=Boltzmann constant and

T= temperature.

Case1. At T=0K & E < 𝐸𝐹 ,f(E)= 1

Case2. At T=0k & E > 𝐸𝐹 ,f(E)= 0 and

Case3. At T> 0𝐾 & E = 𝐸𝐹 ,f(E)= 0.5

Variation of f(E) with E is as shown in the graph.

Q:What is meant by Fermi factor ?Explain the variation of Fermi factor with temperature & energy.

Fermi factor is the probability of occupation of a given energy state by an electron in a metal

at thermal equilibrium. It is given by the relation f(E) = 𝟏

𝟏+𝒆(π‘¬βˆ’π‘¬π‘­)/π’Œπ‘» ,where πΈπΉπΉπ‘’π‘Ÿπ‘šπ‘– π‘’π‘›π‘’π‘Ÿπ‘”π‘¦,

E = energy of State, k = Boltzmann constant and T = temperature

The variation of f(E) with temperature and energy is discussed below:

When T = 0K

Case-1: If E < 𝐸𝐹 (𝑬 βˆ’ 𝑬𝑭)𝑖𝑠 βˆ’ 𝑣𝑒 ,then 𝒆(π‘¬βˆ’π‘¬π‘­)/π’Œπ‘» = π’†βˆ’βˆž = 0

∴ f(E) = 𝟏

𝟏+𝒆(π‘¬βˆ’π‘¬π‘­)/π’Œπ‘» = 𝟏

𝟏+𝟎 =1,

Thus the probability of occupation up to Fermi level is 100%.

Case-2: If E > 𝐸𝐹 (𝑬 βˆ’ 𝑬𝑭)𝑖𝑠 + 𝑣𝑒 ,then 𝒆(π‘¬βˆ’π‘¬π‘­)/π’Œπ‘» = π’†βˆž = ∞

∴ f(E) = 𝟏

𝟏+𝒆(π‘¬βˆ’π‘¬π‘­)/π’Œπ‘» = 𝟏

𝟏+∞=

1

∞ =0,

Thus the probability of occupation above Fermi level is 0%.

Case-3: If E = 𝐸𝐹 (𝑬 βˆ’ 𝑬𝑭) = 0 ,then 𝒆(π‘¬βˆ’π‘¬π‘­)/π’Œπ‘» = π’†πŸŽ

𝟎 indeterminate

f(E)

1 T = 0K

0.5 T> π‘œπΎ

0 E

𝐸𝐹

Page 32: AND ANSWERS SEM-I/II

REDDAPPA C MODULE-2 INTERLINE PUBLISHING.COM

VTU ENGINEERING PHYSICS (17 PHY12/22) SEM-I/II Page 7

When T > 0K

∴ f(E)= 𝟏

𝟏+𝒆(π‘¬βˆ’π‘¬π‘­)/π’Œπ‘»

𝑰𝒇 𝑬 = 𝑬𝑭 (𝑬 βˆ’ 𝑬𝑭) = 𝟎 π‘‘β„Žπ‘’π‘› 𝒆(π‘¬βˆ’π‘¬π‘­)/π’Œπ‘» = π’†πŸŽ = 1

∴ f(E) = 𝟏

𝟏+𝒆(π‘¬βˆ’π‘¬π‘­)/π’Œπ‘» = 𝟏

𝟏+𝟏 =

𝟏

𝟐 Thus the probability

of occupation of Fermi level is 50% above 0K.

The variation of f(E) with temperature(T) and

energy(E)is shown in the graph .

Q:What is meant by Density of states ? Explain.

Density of states g(E) is defined as the number of electronic states present in a unit

energy range. Mathematically g(E) 𝑑𝐸 = 8√2πœ‹π‘š3/2

β„Ž3 𝐸1/2 𝑑𝐸 is a continuous function and

the product g(E)dE=dN gives the number of states per unit volume in an energy range

(dE)between E and E+dE The number of electrons per unit volume ,n=∫ 𝑔(𝐸)𝑓(𝐸)𝑑𝐸 and

theVariation of g(E) with E is shown in the graph.

Q:What is meant by effective mass? Explain.

When an electric field is applied to a metal, electrons in the K-shell

are not at all accelerated as they are tightly bound to the nucleus.

These electrons possess infinite mass called effective mass denoted by π’Žβˆ—

Effective mass is equal to the true mass if the electron is in vacuum.

g(E)

0

E

f(E) Note: 𝑇1 = 500π‘˜ < 𝑇2=1000k

1 T = 0K

0.5 𝑇1 < 𝑇2

0 E

𝐸𝐹

Page 33: AND ANSWERS SEM-I/II

REDDAPPA C MODULE-2 INTERLINE PUBLISHING.COM

VTU ENGINEERING PHYSICS (17 PHY12/22) SEM-I/II Page 8

Q:Explain the dependence of resistivity on temperature and

impurity Or State and explain Matthiessen’s rule.

1. Variation of resistivity 𝝆 of metals with temperature T is given by

Matthiessen’s rule 𝝆 = π†π’Š + 𝝆𝑷𝒉 (T)

Where πœŒπ‘– = π‘‘π‘’π‘šπ‘π‘’π‘Ÿπ‘Žπ‘‘π‘’π‘Ÿπ‘’ 𝑖𝑛𝑑𝑒𝑝𝑒𝑛𝑑𝑒𝑛𝑑 π‘Ÿπ‘’π‘ π‘–π‘ π‘‘π‘–π‘£π‘–π‘‘π‘¦ 𝑑𝑒𝑒 π‘‘π‘œ π‘–π‘šπ‘π‘’π‘Ÿπ‘–π‘‘π‘–π‘’π‘  & π‘–π‘šπ‘π‘’π‘Ÿπ‘“π‘’π‘π‘‘π‘–π‘œπ‘›π‘ 

πœŒπ‘ƒβ„Ž (T)= π‘‘π‘’π‘šπ‘π‘’π‘Ÿπ‘Žπ‘‘π‘’π‘Ÿπ‘’ 𝑑𝑒𝑝𝑒𝑛𝑑𝑒𝑛𝑑 π‘Ÿπ‘’π‘ π‘–π‘ π‘‘π‘–π‘£π‘–π‘‘π‘¦ 𝑑𝑒𝑒 π‘‘π‘œπ‘™π‘Žπ‘‘π‘‘π‘–π‘π‘’

π‘β„Žπ‘œπ‘›π‘œπ‘›π‘£π‘–π‘π‘Ÿπ‘Žπ‘‘π‘–π‘œπ‘›π‘ .

2. Variation of resistivity 𝝆 with temperature T is shown graphically.

At 0K, 𝝆𝑷𝒉 (T) = 0 = π†π’Š ,

3. For pure metals πœŒπ‘– = 0 ,but π‘π‘Ÿπ‘Žπ‘π‘‘π‘–π‘π‘Žπ‘™π‘™π‘¦ π‘›π‘œπ‘‘ π‘π‘œπ‘ π‘ π‘–π‘π‘™π‘’ π‘‘π‘œ π‘π‘Ÿπ‘œπ‘‘π‘’π‘π‘’ π‘π‘’π‘Ÿπ‘’ π‘šπ‘’π‘‘π‘Žπ‘™π‘ , ∴ πœŒπ‘– β‰ 

0

4. and at large T , πœŒπ‘– is negligible compared to π‘™π‘Žπ‘Ÿπ‘”π‘’ πœŒπ‘ƒβ„Ž (T) ∴ 𝜌 = πœŒπ‘ƒβ„Ž (T)

Q:Derive an expression for electrical conductivity of a semi-conductor.

In a semiconductor the net current is due to both electrons and holes .

The current due to electrons is 𝐼𝑒 = π‘›π‘’π‘’π‘Žπ‘£π‘’ ,where 𝑛𝑒 = π‘›π‘’π‘šπ‘π‘’π‘Ÿ 𝑑𝑒𝑛𝑠𝑖𝑑𝑦, π‘Ž = π‘Žπ‘Ÿπ‘’π‘Ž ,

𝑣𝑒 = π‘‘π‘Ÿπ‘–π‘“π‘‘ π‘£π‘’π‘™π‘œπ‘π‘–π‘‘π‘¦ & 𝑒 = π‘β„Žπ‘Žπ‘Ÿπ‘”π‘’

Also current due to holes is πΌβ„Ž = π‘›β„Žπ‘’π‘Žπ‘£β„Ž

But the total current I= 𝐼𝑒 + πΌβ„Ž

= ea(𝑛𝑒𝑣𝑒 + π‘›β„Žπ‘£β„Ž)

𝐼

π‘Ž = e (𝑛𝑒𝑣𝑒 + π‘›β„Žπ‘£β„Ž)

π‘ŠπΎπ‘‡, π‘π‘’π‘Ÿπ‘Ÿπ‘’π‘›π‘‘ 𝑑𝑒𝑛𝑠𝑖𝑑𝑦 𝐽 =𝐼

π‘Ž

∴ J= e (𝑛𝑒𝑣𝑒 + π‘›β„Žπ‘£β„Ž)

but from Ohm’s law J=𝜎𝐸

ie: 𝜎𝐸 = 𝑒 (𝑛𝑒𝑣𝑒 + π‘›β„Žπ‘£β„Ž)

𝜎 = 𝑒 (𝑛𝑒𝑣𝑒

𝐸+ π‘›β„Ž

π‘£β„Ž

𝐸)

𝝈 = e(𝒏𝒆𝝁𝒆+𝒏𝒉𝝁𝒉)

where πœ‡π‘’&πœ‡β„Ž mobilities of electrons and holes respectively.

For an intrinsic semiconductors 𝑛𝑒 = π‘›β„Ž = 𝑛𝑖 called the density of intrinsic charge

carriers.

For n-type semiconductor 𝑛𝑒 ≫ π‘›β„Ž 𝝈 = e𝒏𝒆𝝁𝒆

𝜌

πœŒπ‘–

0 T

Page 34: AND ANSWERS SEM-I/II

REDDAPPA C MODULE-2 INTERLINE PUBLISHING.COM

VTU ENGINEERING PHYSICS (17 PHY12/22) SEM-I/II Page 9

For p-type semiconductor, 𝑛𝑒 β‰ͺ π‘›β„Ž 𝝈 = e𝒏𝒉𝝁𝒉

Q: State law of mass action. Obtain an expression for the intrinsic carrier density.

Law of mass action states that the product of the concentration of charge carriers

electrons ( 𝑛𝑒)and holes( π‘›β„Ž) in an intrinsic semiconductor is equal to the

square of the intrinsic charge carriers( 𝑛𝑖2) at any temperature.

ie: 𝒏𝒆. 𝒏𝒉 = π’π’ŠπŸ

Q: Derive an expression for electron concentration in an intrinsic semiconductor.

or Derive an expression for electron charge carrier density in an intrinsic

semiconductor.

We know that the number of energy levels between the energy interval E and E+dE

is given by g(E)dE = (8√2πœ‹(π‘šπ‘’

βˆ—)3/2

β„Ž3 ) 𝐸1/2𝑑𝐸 ,where π‘šπ‘’βˆ— = effective mass of

electron.

Also, the probability of occupation of an energy level is given by Fermi factor

f(E) =1

𝑒(πΈβˆ’πΈπΉ

π‘˜π‘‡ )

+1

where 𝐸𝐹 = πΉπ‘’π‘Ÿπ‘šπ‘– π‘’π‘›π‘’π‘Ÿπ‘”π‘¦ & π‘˜ = π΅π‘œπ‘™π‘‘π‘§π‘šπ‘Žπ‘›π‘› π‘π‘œπ‘›π‘ π‘‘π‘Žπ‘›π‘‘

Then ,the concentration of electron charge carriers is given by

𝑛𝑒 = ∫ 𝑓(𝐸)∞

𝐸𝑔 g(E)dE

= ∫1

𝑒(πΈβˆ’πΈπΉ

π‘˜π‘‡ )

+1

∞

𝐸𝑔 (

8√2πœ‹(π‘šπ‘’βˆ— )3/2

β„Ž3 ) 𝐸1/2𝑑𝐸

where π‘šπ‘’βˆ— = effective mass of electron.

On integration we can show that

𝑛𝑒 = 4√2 ( πœ‹π‘šπ‘’

βˆ— π‘˜π‘‡

β„Ž2)

3/2 𝑒

(πΈπΉβˆ’πΈπ‘”

π‘˜π‘‡)

Similarly, the concentration of hole charge carriers is given by

π‘›β„Ž = ∫ 𝑓(𝐸)0

βˆ’βˆž g(E)dE

= ∫1

𝑒(πΈβˆ’πΈπΉ

π‘˜π‘‡ )

+1

0

βˆ’βˆž (

8√2πœ‹(π‘šβ„Žβˆ— )

3/2

β„Ž3 ) 𝐸1/2𝑑𝐸

where π‘šβ„Žβˆ— = effective mass of electron.

On integration we can show that

π‘›β„Ž = 4√2 ( πœ‹π‘šβ„Ž

βˆ— π‘˜π‘‡

β„Ž2 )3/2

𝑒(

βˆ’πΈπΉπ‘˜π‘‡

)

Page 35: AND ANSWERS SEM-I/II

REDDAPPA C MODULE-2 INTERLINE PUBLISHING.COM

VTU ENGINEERING PHYSICS (17 PHY12/22) SEM-I/II Page 10

Q: Based on law of mass action derive an expression for intrinsic charge density

(concentration) in an intrinsic semiconductor.

Law of mass action states that the product of the concentration of charge carriers

electrons ( 𝑛𝑒)and holes( π‘›β„Ž) in an intrinsic semiconductor is equal to the square

of the intrinsic charge carriers( 𝑛𝑖2) at any temperature.

ie: π’π’ŠπŸ = 𝒏𝒆. 𝒏𝒉

But , WKT, π‘π‘œπ‘›π‘π‘’π‘›π‘‘π‘Ÿπ‘Žπ‘‘π‘–π‘œπ‘› π‘œπ‘“ π‘’π‘™π‘’π‘π‘‘π‘Ÿπ‘œπ‘›π‘ , 𝑛𝑒 = 4√2 ( πœ‹π‘šπ‘’

βˆ—π‘˜π‘‡

β„Ž2 )3/2

𝑒(

πΈπΉβˆ’πΈπ‘”π‘˜π‘‡

) and

Concentration of holes, π‘›β„Ž = 4√2 ( πœ‹π‘šβ„Ž

βˆ— π‘˜π‘‡

β„Ž2)

3/2 𝑒

(βˆ’πΈπΉπ‘˜π‘‡

)

From law of mass action,

𝑛𝑖2 = 𝑛𝑒 . π‘›β„Ž

∴ 𝑛𝑖2 = 4√2 (

πœ‹π‘šπ‘’βˆ—π‘˜π‘‡

β„Ž2 )3/2 𝑒(πΈπΉβˆ’πΈπ‘”

π‘˜π‘‡) 𝑋 4√2 (

πœ‹π‘šβ„Žβˆ—π‘˜π‘‡

β„Ž2 )3/2 𝑒(

βˆ’πΈπΉπ‘˜π‘‡

)

𝑛𝑖 = √4√2 ( πœ‹π‘šπ‘’

βˆ—π‘˜π‘‡

β„Ž2 )3/2 𝑒(πΈπΉβˆ’πΈπ‘”

π‘˜π‘‡) 𝑋 4√2 (

πœ‹π‘šβ„Žβˆ— π‘˜π‘‡

β„Ž2 )3/2 𝑒(βˆ’πΈπΉπ‘˜π‘‡

)

on simplification we can we can show that,

π’π’Š = 4√𝟐 ( π…π’Žπ’†

βˆ—π’Œπ‘»

π’‰πŸ )πŸ‘/πŸ’( π…π’Žπ’‰

βˆ— π’Œπ‘»

π’‰πŸ )πŸ‘/πŸ’ 𝒆(

βˆ’π‘¬π’ˆ

πŸπ’Œπ‘»)

Q:Explain the terms: Fermi velocity & Fermi temperature.

Fermi velocity (𝑣𝐹)is defined as the velocity of the electron having the energy

equal to Fermi energy(𝐸𝐹) ,mathematically ,1

2π‘šπ‘£πΉ

2 = 𝐸𝐹 ∴ 𝑣𝐹 = √2𝐸𝐹

π‘š

Fermi temperature (𝑇𝐹)is defined as the ratio of Fermi energy(𝐸𝐹) to

the Boltzmann constant(k). Mathematically,𝑇𝐹 =𝐸𝐹

π‘˜

Page 36: AND ANSWERS SEM-I/II

REDDAPPA C MODULE-2 INTERLINE PUBLISHING.COM

VTU ENGINEERING PHYSICS (17 PHY12/22) SEM-I/II Page 11

SUPERCONDUCTIVITY:

Q: Explain the terms :-

a. Superconductivity is the phenomenon in which the resistivity/resistance of a substance

becomes zero.

b. Superconducting state is the zero resistivity/resistance state of the substance.

c. Critical/Transition temperature is the minimum temperature at which the substance

changes from normal state to superconducting state.

d. Critical magnetic field is the minimum magnetic field at which the substance changes to

normal state from superconducting state.

e. Flux quantization is the quantization of the magnetic flux through a superconducting ring

and is given by βˆ… =𝒏𝒉

πŸπ’† ,where n =1,2,3…..h = Planck’s constant and e = electron charge.

f. Isotopic effect states that the transition temperature 𝑇𝐢varies inversely

as the square root of the atomic mass M ie: 𝑇𝐢 ∝1

βˆšπ‘€

Q: temperature dependence of resistivity in metals and superconducting materials.

1. The variation of the resistivity of the metals and superconductor is as shown in the graph.

2. When metals are cooled their resistivity decreases and reaches the minimum value

πœŒπ‘– called the residual resistivity at 0K , πœŒπ‘– is due to impurities and imperfections in

π‘‘β„Žπ‘’ metals.

3. The resistivity of the metals increases with temperature due to ion vibrations .

4. The resistivity of the metals is given by Mathessian’s rule 𝜌 = πœŒπ‘– + πœŒπ‘ƒβ„Ž(T).

5. But the resistivity of superconductors decreases with the decrease of temperature and

suddenly becomes zero at the temperature 𝑇𝐢 called critical/transition temperature.

6. The transition temperature depends on the impurities present in the metals.

Magnetic materials reduce 𝑇𝐢 to large extent.

𝜌 Normal conductor

πœŒπ‘– Super conductor

0 𝑇𝐢 T

Page 37: AND ANSWERS SEM-I/II

REDDAPPA C MODULE-2 INTERLINE PUBLISHING.COM

VTU ENGINEERING PHYSICS (17 PHY12/22) SEM-I/II Page 12

Q:Explain Meissner’s effect.

1) Meissner effect is the phenomenon in which the magnetic flux in the material

Is completely expelled out of the material at temperature called Critical

temperature.

2) Below Critical temperature the material behave as perfect dia-magnet.

3) Consider a superconducting material placed in a magnetic field H, the magnetic

lines penetrate through the conductor.

4) When the temperature of the conductor is cooled below the critical temperature

𝑇𝐢 ,the magnetic flux inside the conductor is completely expelled out of the

conductor.

5) Above 𝑇𝐢 ,Bβ‰  0 and below 𝑇𝐢 ,B= 0.

6) Meissner effect is the principle of Maglev vehicles, Squid etc.

Q:Write a note on Type-I and Type-II superconductors.

Type-I

Type-I

1) Type-I Superconductors(SC’s) are also called soft SCs.

2) The magnetization of Type-I SC’s increases with the increase of the applied magnetic field

and suddenly drops to zero at the magnetic field 𝐻𝐢 called the critical magnetic field as

shown graphically.

H H

T> 𝑻π‘ͺ π‘†π‘’π‘π‘’π‘Ÿ π‘π‘œπ‘›π‘‘π‘’π‘π‘‘π‘œπ‘Ÿ T< 𝑻π‘ͺ

βˆ’π‘€

Super

Conducting Normal

State State

O 𝐻𝐢 H

Page 38: AND ANSWERS SEM-I/II

REDDAPPA C MODULE-2 INTERLINE PUBLISHING.COM

VTU ENGINEERING PHYSICS (17 PHY12/22) SEM-I/II Page 13

3) Below 𝐻𝐢 SC becomes perfectly diamagnetic obeying Meissner effect and beyond 𝐻𝐢 SC

changes to normal state.

4) The Type-I SC’s exists only in two states namely superconducting and normal

Type-II

1) Type-II Superconductors(SC’s) are also called hard SC’s.

2) The magnetization of Type-II SC’s increases with the increase of the applied magnetic

field and begin to decrease at the magnetic field 𝐻𝐢1 called the lower critical field and

finally becomes zero at the upper critical magnetic field 𝐻𝐢2.

3) Up to 𝐻𝐢1 Type-II SC’s exhibit Meissner effect with perfect diamagnetism.

4) Between 𝐻𝐢1 & 𝐻𝐢2 they exhibit meissner effect partially.ie: Magnetic flux begins to

penetrate at 𝐻𝐢1 π‘Žπ‘›π‘‘ π‘π‘œπ‘šπ‘π‘™π‘’π‘‘π‘™π‘¦ penetrates at 𝐻𝐢2.

5) Thus Type-II SC’s exists in three states namely SC state up to 𝐻𝐢1, intermediate state

between 𝐻𝐢1 & 𝐻𝐢2 and normal state beyond 𝐻𝐢2.

6) Type-II SC’s exhibit superconductivity electrically even in the intermediate state .They can

carry large currents.

7) These SC’s are used as coils for producing very large magnetic fields.

8) Type-I can be changed in Type-II , Type-I SC Lead can be changed to Type-II by adding

indium as impurity to it.

βˆ’π‘€

Super Intermediate Normal

conducting state state

state

O 𝐻𝐢1 𝐻𝐢 𝐻𝐢2 H

Page 39: AND ANSWERS SEM-I/II

REDDAPPA C MODULE-2 INTERLINE PUBLISHING.COM

VTU ENGINEERING PHYSICS (17 PHY12/22) SEM-I/II Page 14

Q; Explain the temperature dependence of critical field.

1) Critical magnetic field(𝐻𝐢) is the minimum magnetic field applied to the superconductor

to change from superconducting state to normal state.

2) The variation of the critical field with temperature is given by the relation

𝐻𝐢 = 𝐻0 [1 βˆ’π‘‡2

𝑇𝐢2],

where 𝐻0 is called the critical field at 0K , T = temperature and 𝑇𝐢 =Critical temperature.

3) Thus critical field decreases with the increase of temperature.

Q:Explain BCS theory of superconductivity qualitatively.

1) BCS(Bardeen-Cooper-Schrieffer) theory successfully explained superconductivity

quantum mechanically.

2) BCS theory is based on electrons interactions through phonons/lattice as

mediators.

3) An electron distorts the lattice due to electrostatic attraction when it approach a

positive ion lattice, Smaller the mass of the ion distortion is more.

4) The oscillatory distorted lattice is quantized in terms of phonons .

5) This distorted lattice attract another electron by reducing its energy.

6) Thus the second electron attracts the first electron through lattice. This results in the

pairing of two electrons called β€œCooper Pair”. The attraction between the electrons is

maximum when the two electrons have equal and opposite spin and momentum.

( π‘˜1 βˆ’ π‘ž ) ( π‘˜2 +q )

q

π‘˜1 π‘˜2

e e

H

𝐻𝑂

Superconducting normal

State state

O 𝑇𝐢 T

Page 40: AND ANSWERS SEM-I/II

REDDAPPA C MODULE-2 INTERLINE PUBLISHING.COM

VTU ENGINEERING PHYSICS (17 PHY12/22) SEM-I/II Page 15

7) The formation of cooper pair is considered as emission and absorption of the phonon by

the two electrons as shown in the diagram.

8) The cooper pair formation begins at critical temperature 𝑇𝐢 and is completed at 0K.

9) Thus at 0K all the electrons are paired in to ordered cooper pairs, which can pass through

lattice without collisions when a potential difference is applied. This explains the zero

resistivity of superconductors.

10) When the temperature increases the breaking of the cooper pairs begin beyond

0K and completed π‘Žπ‘‘ 𝑇𝐢 as a result the superconductor attains normal state.

11) BCS theory explained the existence of energy gap in superconductors, Meissner

effect ,Coherence length and Flux quantization, etc.

Q: Write a note on High temperature superconductors(HTSC).

1. Generally Superconductors of transition temperature more than 10K are called

High temperature superconductors(HTSCs).

2. First HTSC found was ceramic material (oxide of lanthanium, barium &

copper)whose 𝑇𝐢 =30~35K, by Bednorz and Muller

3. Later many HTSCs are found of 𝑇𝐢 more than 150K using liquid Nitrogen.

In 2015 HTSC found was Hydrogen Sulphide of 𝑇𝐢 = 203K at very high pressure.

4. Also 1-2-3 compound HTSCs are found which are alloys of 1 rare Earth element

atom, 2 barium atoms,3 Copper atoms and 7 Oxygen atoms. Ex:Yπ΅π‘Ž2𝐢𝑒3𝑂7 etc

5. HTSCs are all Type-II superconductors having complicated structures called

Perovskite , Tetragonal and Orthorhombic.

6. In all the HTSCs 1 to 4 Cu-O layers are placed between other layers.

7. HTSCs are used in Squids, MagLev vehicles ,MRI, NMR, LHC, and other applications

in various fields like Industry, Medicine, Physics, Chemistry, Engineering ,Scientific

research etc

8. HTSCs are not perfectly diamagnetic since they exhibit partial Meissner’s effect.

Q;Explain the Principle,Construction and working of Maglev vehicles.

1) Maglev vehicles works on the principle of Meissner effect & magnetic levitation.

RW RW

Vehicle

Superconducting magnet

Aluminium guide way

Page 41: AND ANSWERS SEM-I/II

REDDAPPA C MODULE-2 INTERLINE PUBLISHING.COM

VTU ENGINEERING PHYSICS (17 PHY12/22) SEM-I/II Page 16

2) Floating of a magnet placed over a superconducting magnet is called magnetic levitation.

3) The schematic diagram of Maglev vehicle is as shown in the diagram. Superconducting

magnet is fitted in to the base of the vehicle, which is placed over aluminium guide way.

4) Magnetic field produced due to current in the guide way lifts/levitate the vehicle and also

propel it.

5) The vehicle is provided with retractable wheels which can be pulled out when the vehicle

is brought to rest and can be pulled in when levitated and propelled.

6) Maglev vehicle trains have attained very high speeds of 603 km/hr.

PROBLEMS SECTION

MODULE-2 : ELECTRICAL PROPERTIES OF MATERIALS

Formulae needed : f(E) = 1

1+π‘’πΈβˆ’πΈπΉ

π‘˜π‘‡

; 𝜌 = π‘š

𝑛𝑒2𝜏 ; 𝜎 =

1

𝜌 ; 𝜎 =

𝑛𝑒2𝜏

π‘š ;

n = π‘₯𝐷𝑁𝐴

𝑀 ; πœ‡ =

1

π‘›π‘’πœŒ ; Β½ m𝑉𝐹

2 = 𝐸𝐹 ; 𝑉𝑑 = π‘’πΈπœ

π‘š ; Ξ» = 𝑉. 𝜏 ; n =

𝜎

πœ‡π‘’=

1

πœŒπœ‡π‘’ ; 𝐻𝐢 =

𝐻0 [1 βˆ’π‘‡2

𝑇𝐢2]

1. Find the relaxation time of conduction electrons in a metal of resistivity 1.54 x10βˆ’8 Ξ©-m, if

the metal has 5.8 x1028 Conduction electrons per π‘š3.( JAN2015)

Given: 𝜌 = 1.54 x10βˆ’8 Ξ©-m ; n = 5.8 x1028 /π‘š3 ; π‘š = 9.1π‘₯10βˆ’31π‘˜π‘” ;

𝑒 = 1.6π‘₯10βˆ’19𝐢 ; πœπ‘Ÿ= 𝜏 =?

Using 𝜌 = π‘š

𝑛𝑒2𝜏 ,we get 𝜏 =

π‘š

πœŒπ‘›π‘’2

= 9.1π‘₯10βˆ’31

1.54 π‘₯10βˆ’8π‘₯5.8 π‘₯1028π‘₯(1.6π‘₯10βˆ’19)2

= 3.98 π‘₯10 βˆ’14 S

Page 42: AND ANSWERS SEM-I/II

REDDAPPA C MODULE-2 INTERLINE PUBLISHING.COM

VTU ENGINEERING PHYSICS (17 PHY12/22) SEM-I/II Page 17

2. Calculate the mobility and relaxation time of an electron in copper assuming that each

atom contributes one free electron ,whose density is 8.92 x 103kg/π‘š3 and 𝑁𝐴=6.02 x

1026/kg mole. (Jun/Jul 14) Given: 𝜌 = 1.73 x 10βˆ’8 Ξ©-m ; M = 63.5 ; D = 8.92 x 103kg/π‘š3 ; 𝑁𝐴=6.02 x 1026/kg mole ; π‘š =

9.1π‘₯10βˆ’31π‘˜π‘” ; 𝑒 = 1.6π‘₯10βˆ’19𝐢 ; π‘₯ = 1 π‘’π‘™π‘’π‘π‘‘π‘Ÿπ‘œπ‘›/π‘Žπ‘‘π‘œπ‘š, 𝑛 = ? ; πœ‡ = ? & 𝜏 = ?

π‘ˆπ‘ π‘–π‘›π‘” n = π‘₯𝐷.𝑁𝐴

𝑀

= 1π‘₯8.92 π‘₯ 103π‘₯.6.02 π‘₯ 1026

63.5

= 8.456x1028 atoms.

πœ‡ = 1

π‘›π‘’πœŒ

πœ‡ = 1

8.456π‘₯1028π‘₯1.6π‘₯10βˆ’19π‘₯1.73 π‘₯ 10βˆ’8 = 4.27x10βˆ’3

also πœ‡ = π‘’πœ

π‘š , we get 𝜏 =

πœ‡π‘š

𝑒

= 4.27π‘₯10βˆ’3π‘₯9.1π‘₯10βˆ’31

1.6π‘₯10βˆ’19

= 2.43 x10βˆ’14 S

3. Find the temperature at which there is 1.0% probability that a state with an energy 0.5eV

above Fermi energy will be occupied. (Dec 2013/Jan2014)

Given: (𝐸 βˆ’ 𝐸𝐹) = 0.5 𝑒𝑉 = 0.5π‘₯1.6π‘₯10βˆ’19 J , k=1.38x10βˆ’23 J/K ; f(E) = 1 % = 1/100 =0.01 ; T= ?

Using f(E)= 1

1+𝑒(πΈβˆ’πΈπΉ)/π‘˜π‘‡ ,we get T = (πΈβˆ’πΈπΉ)

π‘˜.𝐼𝑛(1

𝑓(𝐸) –1)

= 0.5π‘₯1.6π‘₯10βˆ’19

1.38π‘₯10βˆ’23.𝐼𝑛(1

0.01 –1)

= 0.5π‘₯1.6π‘₯10βˆ’19

1.38π‘₯10βˆ’23.𝐼𝑛(100 –1)

= 1262 K =1.262x103K

4. Calculate the conductivity of sodium given πœπ‘š =2 x 10βˆ’14 S. Density of sodium is 971 kg/π‘š3,its

atomic weight is 23 and has one conduction electron/atom. (𝐽𝑒𝑛 2012)

Given: M = 23 ; D = 971 kg/π‘š3 ; 𝑁𝐴=6.02 x 1026/kg mole ; π‘š = 9.1π‘₯10βˆ’31π‘˜π‘” ; 𝜏 = πœπ‘š =2 x 10βˆ’14 S

𝑒 = 1.6π‘₯10βˆ’19𝐢 ; π‘₯ = 1π‘’π‘™π‘’π‘π‘‘π‘Ÿπ‘œπ‘›/π‘Žπ‘‘π‘œπ‘š, 𝑛 = ? ; 𝜎 = ?

π‘ˆπ‘ π‘–π‘›π‘” n =π‘₯𝐷.𝑁𝐴

𝑀

= 1π‘₯971π‘₯.6.02 π‘₯ 1026

23

= 2.54x1028 atoms.

Page 43: AND ANSWERS SEM-I/II

REDDAPPA C MODULE-2 INTERLINE PUBLISHING.COM

VTU ENGINEERING PHYSICS (17 PHY12/22) SEM-I/II Page 18

Also, 𝜎 = 𝑛𝑒2𝜏

π‘š

= 2.54π‘₯1028π‘₯( 1.6π‘₯10βˆ’19)2π‘₯2 π‘₯ 10βˆ’14

9.1π‘₯10βˆ’31

= 1.42x107 Ξ©m.

5. Calculate the Fermi velocity and the mean free path for the conduction electrons in silver,

given that its Fermi energy is 5.5 eV and relaxation time for electrons is 3.97

X10βˆ’14S. (𝐷𝑒𝑐 2011)

Given: 𝐸𝐹 = 5.5 𝑒𝑉 = 5.5 π‘₯ 1.6π‘₯10βˆ’19 𝐽 ; 𝜏 = πœπ‘Ÿ = 3.97 X10βˆ’14S ; π‘š = 9.1π‘₯10βˆ’31π‘˜π‘” 𝑉𝐹 = ? :

Ξ» = ?

Using Β½ m𝑉𝐹2 = 𝐸𝐹 ,we get 𝑉𝐹 = √

2𝐸𝐹

π‘š

= √2π‘₯5.5 π‘₯ 1.6π‘₯10βˆ’19

9.1π‘₯10βˆ’31

= 1.391x 106 m/s and

Also, Ξ» =𝑉𝐹. πœπ‘Ÿ = 1.391x 105 x 3.97 X10βˆ’14 = 5.522π‘₯ 10βˆ’9 m

6. The Fermi level of potassium is 2.1eV. What are the energies for which the probabilities of

occupancy at 300 K are 0.99 and 0.5 ? (𝐽𝑒𝑛/𝐽𝑒𝑙2011)

Given: 𝐸𝐹 =2.1 eV = 2.1x1.6x10βˆ’19 𝐽 ;T = 300K ; f(𝐸1) =0.99 ; f(𝐸2) =0.5 ;

K =1.38x10βˆ’23 J/K ; 𝐸1 =? ; 𝐸2 =?

Using f(E)= 1

1+𝑒(πΈβˆ’πΈπΉ)/π‘˜π‘‡ ,

we get E = 𝐸𝐹 + π‘˜π‘‡ 𝐼𝑛 [ 1

𝑓(𝐸) – 1]

∴ 𝐸1 = 2.1π‘₯1.6π‘₯10βˆ’19 + 1.38x10βˆ’23π‘₯300 𝐼𝑛 [ 1

0.99 – 1]

= 3.36 π‘₯10βˆ’19 βˆ’ 0.1902 π‘₯10βˆ’19

= 3.17x10βˆ’19 𝐽

= 3.17π‘₯10βˆ’19

1.6π‘₯10βˆ’19 eV

= 1.98eV

Also, 𝐸2 = 2.1π‘₯1.6π‘₯10βˆ’19 + 1.38x10βˆ’23π‘₯300 𝐼𝑛 [ 1

0.5 – 1]

= 3.36 π‘₯10βˆ’19 βˆ’ 0

= 3.36 π‘₯10βˆ’19 𝐽

= 3.36 π‘₯10βˆ’19

1.6 π‘₯10βˆ’19 eV

= 2.1 eV

Page 44: AND ANSWERS SEM-I/II

REDDAPPA C MODULE-2 INTERLINE PUBLISHING.COM

VTU ENGINEERING PHYSICS (17 PHY12/22) SEM-I/II Page 19

7. The Fermi level for a metal is 2.1eV, calculate the energies for which the probability of

occupancy at 300 K are 98% and 50%.(𝐷𝑒𝑐 2010)

Given: 𝐸𝐹 =2.1 eV = 2.1x1.6x10βˆ’19 𝐽 ;T = 300K ; f(𝐸1) =98%=0.98 ; f(𝐸2)=50%=0.5

k =1.38x10βˆ’23 J/K ; 𝐸1? , 𝐸2 =?

Using f(E)= 1

1+𝑒(πΈβˆ’πΈπΉ)/π‘˜π‘‡ ,

we get E = 𝐸𝐹 + π‘˜π‘‡ 𝐼𝑛 [ 1

𝑓(𝐸) – 1]

∴ 𝐸1 = 2.1π‘₯1.6π‘₯10βˆ’19 + 1.38x10βˆ’23π‘₯300 𝐼𝑛 [ 1

0.98 – 1]

= 3.36 π‘₯10βˆ’19 βˆ’ 0.16 π‘₯10βˆ’19

= 3.2x10βˆ’19 𝐽

= 3.2π‘₯10βˆ’19

1.6π‘₯10βˆ’19 eV

=2eV

Also , 𝐸2 = 2.1π‘₯1.6π‘₯10βˆ’19 + 1.38x10βˆ’23π‘₯300 𝐼𝑛 [ 1

0.5 – 1]

= 2.1π‘₯1.6π‘₯10βˆ’19 βˆ’ 0

= 2.1π‘₯1.6π‘₯10βˆ’19

1.6π‘₯10βˆ’19 eV

= 2.1 eV

8. A uniform silver wire has resistivity 1.54 x10βˆ’8π›Ίπ‘š at room temperature for an electric

field 2 V/m. Calculate relaxation time and drift velocity of the electrons, assuming that

there are 5.8 X 1022 conduction electrons per π‘π‘š3 of the material. (𝐽𝑒𝑛 2010)

Given: 𝜌 = 1.54 x 10βˆ’8 Ξ©-m ; E = 2 V/m ; 𝑛 = 5.8 𝑋 1022/π‘π‘š3 = 5.8 𝑋 1022π‘₯10βˆ’6/

π‘š3 ; π‘š = 9.1π‘₯10βˆ’31π‘˜π‘” ; 𝑒 = 1.6π‘₯10βˆ’19𝐢 ; 𝜏 = ? & 𝑉𝑑 = ?

π‘ˆπ‘ π‘–π‘›π‘”: 𝜏 = πœŒπ‘š

𝑛𝑒2

= 1.54 π‘₯ 10βˆ’8 π‘₯9.1π‘₯10βˆ’31

5.8 𝑋 1022π‘₯10βˆ’6( 1.6π‘₯10βˆ’19)2

= 9.44 x10βˆ’18 S

𝑉𝑑 = π‘’πΈπœ

π‘š

= 1.6π‘₯10βˆ’19π‘₯2π‘₯9.44 π‘₯10βˆ’18

9.1π‘₯10βˆ’31

= 3.32 x 10βˆ’6 m/s

Page 45: AND ANSWERS SEM-I/II

REDDAPPA C MODULE-2 INTERLINE PUBLISHING.COM

VTU ENGINEERING PHYSICS (17 PHY12/22) SEM-I/II Page 20

9. Calculate the probability of an electron occupying an energy level 0.02 eV above the Fermi

level at 300K.( 4 marks)

Given: (𝐸 βˆ’ 𝐸𝐹) = 0.02 𝑒𝑉 = 0.02π‘₯1.6π‘₯10βˆ’19 J , k=1.38x10βˆ’23 J/K and T=300K

Using f(E) = 1

1+𝑒(πΈβˆ’πΈπΉ)/π‘˜π‘‡

= 1

1+𝑒(0.02π‘₯1.6π‘₯10βˆ’19)/1.38π‘₯10βˆ’23 π‘₯ 300

= 1

1+𝑒0.773

= 1

1+2.166

=1

3.166

=0.32 or 32 %

10. Calculate the concentration at which the acceptor atoms must be added to a germanium

sample to get a P-type semiconductor with conductivity 0.15 per Ohm-metre. Given the

mobility of holes=0.17 π‘š2/Vs. ( 4 marks)

Given: 𝜎 =0.15 /Ξ©m ,πœ‡ = 0.17 π‘š2/𝑉𝑠 , e=1.6x10βˆ’19 C, n= ?

Using, n =𝜎

πœ‡π‘’

=0.15

0.17π‘₯1.6π‘₯10βˆ’19

= 5.515 x1018 electrons

13 . A superconducting tin has a critical field of 306 gauss at 0K and 217 gauss at 2K.

Find the critical temperature of superconducting tin.

Given : π»π‘œ = 306 π‘”π‘Žπ‘’π‘ π‘ , 𝐻𝐢 = 217 π‘”π‘Žπ‘’π‘ π‘ , 𝑇 = 2𝐾 , 𝑇𝐢 =?

Using: 𝐻𝐢 = π»π‘œ [1 βˆ’π‘‡2

𝑇𝐢2]

We get , 𝑇𝐢 = βˆšπ‘‡2

[1βˆ’π»πΆπ»π‘œ

]

= √22

[1βˆ’217

306]

=3.71K

Page 46: AND ANSWERS SEM-I/II

REDDAPPA C MODULE-2 INTERLINE PUBLISHING.COM

VTU ENGINEERING PHYSICS (17 PHY12/22) SEM-I/II Page 21

14. Calculate the mobility of electrons in copper assuming that each atom

Contribute one free electron for conduction. Resistivity of copper=1.7 X 10βˆ’8

Ξ©m, atomic weight =63.54,density =8.96 X 103kg/π‘š3( CBCS-Jun/Jul16)

Given: 𝜌 = 1.7 x 10βˆ’8 Ξ©-m ; M = 63.54 ; D = 8.96 x 103kg/π‘š3 ; 𝑁𝐴=6.025 x 1026/kg mole

; π‘š = 9.11π‘₯10βˆ’31π‘˜π‘” ; 𝑒 = 1.6π‘₯10βˆ’19𝐢 ; π‘₯ = 1π‘’π‘™π‘’π‘π‘‘π‘Ÿπ‘œπ‘›/π‘Žπ‘‘π‘œπ‘š, 𝑛 = ? ; πœ‡ = ?

π‘ˆπ‘ π‘–π‘›π‘” n =π‘₯𝐷.𝑁𝐴

𝑀

= 1π‘₯8.96 π‘₯ 103π‘₯.6.025 π‘₯ 1026

63.54

= 8.5x1028 atoms.

πœ‡ = 1

π‘›π‘’πœŒ

= 1

8.5π‘₯1028π‘₯1.6π‘₯10βˆ’19π‘₯1.7 π‘₯ 10βˆ’8 = 4.325x10βˆ’3 π‘š2/vs

15. For intrinsic gallium arsenide, the room temperature electrical conductivity is

10βˆ’6 /Ξ©m.The electron and hole mobilities are respectively 0.85 π‘š2/𝑉𝑠 and

0.04 π‘š2/𝑉𝑠. 𝐢alculate the intrinsic carrier concentration at room temperature.

Given: πœŽπ‘– = 10βˆ’6 /Ξ©m ; πœ‡π‘’ = 0.85 π‘š2/𝑉𝑠 πœ‡π‘’ = 0.04 π‘š2/𝑉𝑠; 𝑛𝑖 =?

π‘ˆπ‘ π‘–π‘›π‘”, πœŽπ‘– = 𝑛𝑖e (πœ‡π‘’ + πœ‡β„Ž),

𝑀𝑒 𝑔𝑒𝑑 , 𝑛𝑖 =πœŽπ‘–

𝑒 (πœ‡π‘’ +πœ‡β„Ž)

𝑛𝑖 =10βˆ’6

1.6π‘₯10βˆ’19 (0.85+0.04)

= 7.023π‘₯1012/π‘š3

16. The effective mass for the electron in germanium is 0.55 π‘šπ‘œ,where π‘šπ‘œπ‘–π‘  π‘‘β„Žπ‘’

free electron mass. Find the electron concentration in Germanium at 300 K,

assuming that the Fermi level lies exactly in the middle of the energy gap,

given that the energy gap for Germaium is 0.66 eV.

Given: π‘šπ‘œ = 9.1 x 10βˆ’31kg ; π‘šπ‘’βˆ— = 0.55x 9.1 x 10βˆ’31kg ; T= 300K ; 𝐸𝑔 = 0.66𝑒𝑉;

𝐸𝐹 =𝐸𝑔

2 =

0.66

2 = 0.33 𝑒𝑉

Using ; 𝑛𝑒 = 4√2 ( πœ‹π‘šπ‘’ π‘˜π‘‡

βˆ—

β„Ž2 )

3/2

𝑒(

πΈπΉβˆ’πΈπ‘”π‘˜π‘‡

)

= 4√2 ( πœ‹π‘₯0.55π‘₯ 9.1 π‘₯ 10βˆ’31π‘₯1.38π‘₯10βˆ’23π‘₯300

(6.625π‘₯10βˆ’34)2)3/2 x 𝑒

(0.33βˆ’0.66)π‘₯1.6π‘₯10βˆ’19)

1.38π‘₯10βˆ’23π‘₯300

= 5.657x1.807x1024 x2.892x10βˆ’6

𝑛𝑒 = 2.956x1019 /π‘š3

Page 47: AND ANSWERS SEM-I/II

REDDAPPA C MODULE-2 INTERLINE PUBLISHING.COM

VTU ENGINEERING PHYSICS (17 PHY12/22) SEM-I/II Page 22

17. For intrinsic Gallium Arsenide , the electric conductivity at room temperature

is πŸπŸŽβˆ’πŸ”π’π’‰π’Žβˆ’πŸπ’Žβˆ’πŸ. The electron and hole mobilities are respectively 0.85 π’ŽπŸ/𝑽𝑺 𝒂𝒏𝒅

𝟎. πŸŽπŸ’ π’ŽπŸ/𝑽𝑺. Calculate the intrinsic carrier concentration at room temperature.

(04 Marks)

Given: πœŽπ‘– = 10βˆ’6 /Ξ©m ; πœ‡π‘’ = 0.85 π‘š2/𝑉𝑠 πœ‡π‘’ = 0.04 π‘š2/𝑉𝑠; 𝑛𝑖 =?

π‘ˆπ‘ π‘–π‘›π‘”, πœŽπ‘– = 𝑛𝑖e (πœ‡π‘’ + πœ‡β„Ž),

𝑀𝑒 𝑔𝑒𝑑 , 𝑛𝑖 =πœŽπ‘–

𝑒 (πœ‡π‘’ +πœ‡β„Ž)

𝑛𝑖 =10βˆ’6

1.6π‘₯10βˆ’19 (0.85+0.04)

= 7.023π‘₯1012/π‘š3

18. Calculate the probability of finding an electron at an energy level 0.02 eV above

Fermi level at 200 K . ( 04 Marks)

Given: (𝐸 βˆ’ 𝐸𝐹) = 0.02 𝑒𝑉 = 0.02π‘₯1.602π‘₯10βˆ’19 J , k=1.38x10βˆ’23 J/K and T=200K

Using f(E) = 1

1+𝑒(πΈβˆ’πΈπΉ)/π‘˜π‘‡

= 1

1+𝑒(0.02π‘₯1.602π‘₯10βˆ’19)/1.38π‘₯10βˆ’23 π‘₯ 200

= 1

1+𝑒1.161

= 1

1+3.193

= 1

4.193

= 0.2385 or 23.85 %

19. Gold has one free electron/atom.Its density,atomic weight and resistivity are

19300 kg/π‘š3 ,197 and 2.21 x 10βˆ’8 Ξ©π‘š . Calculate the free electron concentration

and mobility of conduction electron. ( 04 Marks)

Given : D=19300 kg/π’ŽπŸ‘ , M =197 , 𝝆 = 2.21 x 10βˆ’8 Ξ©π‘š , 𝒙 = 1 electron/atom,

𝑡𝑨 = 6.02 x πŸπŸŽπŸπŸ–/Kmol , e=1.6 x πŸπŸŽβˆ’πŸπŸ— C , n =? , 𝝁 = ?

Using, n =𝒙 𝑫 𝑡𝑨

𝑴

∴ 𝒏 =πŸπ’™ πŸπŸ—πŸ‘πŸŽπŸŽπ’™ πŸ”.πŸŽπŸπ’™ πŸπŸŽπŸπŸ–

πŸπŸ—πŸ• = 5.898 x πŸπŸŽπŸ‘πŸŽ electrons/π’ŽπŸ‘

𝑨𝒍𝒔𝒐 , using 𝝁 =𝟏

𝝆𝒏𝒆

∴ 𝝁 =1

2.21 π‘₯ 10βˆ’8π‘₯5.898 π‘₯ 1030π‘₯1.6 π‘₯ 10βˆ’19 = 4.795x10βˆ’5 π‘š2/Vs

Page 48: AND ANSWERS SEM-I/II

REDDAPPA C MODULE-2 INTERLINE PUBLISHING.COM

VTU ENGINEERING PHYSICS (17 PHY12/22) SEM-I/II Page 23

20. Calculate the drift velocity and thermal velocity of conduction electrons in copper

at a temperature of 300K,when a copper wire of length 2 mand resistance 0.02 Ξ© carries

a current of 15 A. Given the mobility of free electrons in copper is 4.3xπŸπŸŽβˆ’πŸ‘π’ŽπŸ/Vs.

( 04 Marks)

Given: T = πŸ‘πŸŽπŸŽπ‘² ,L = 𝟐 π’Ž , 𝑹 = 0.02 Ξ© , I = 15 A , 𝝁 = 4.3xπŸπŸŽβˆ’πŸ‘ π’ŽπŸ/Vs,

m= π’Žπ’† = 9.1 x πŸπŸŽβˆ’πŸ‘πŸ’kg , k = 1.38 x πŸπŸŽβˆ’πŸπŸ‘ J/K , 𝒗𝒅 =? , 𝒗𝑭 =?

Using 𝒗𝒅 = 𝝁E but E = 𝑽

𝑳 and V= IR

we get 𝒗𝒅 =𝝁𝑰𝑹

𝑳 =

πŸ’.πŸ‘π’™πŸπŸŽβˆ’πŸ‘π’™ πŸπŸ“ 𝒙 𝟎.𝟎𝟐

𝟐 = 6.45 x πŸπŸŽβˆ’πŸ’ m/s

Also using πŸ‘

πŸπ’Œπ‘» =

𝟏

πŸπ’Žπ’—π‘­

𝟐

We get , 𝑽𝑭 = βˆšπŸ‘π’Œπ‘»

π’Ž = √

πŸ‘π’™πŸ.πŸ‘πŸ– 𝒙 πŸπŸŽβˆ’πŸπŸ‘π’™πŸ‘πŸŽπŸŽ

πŸ—.𝟏 𝒙 πŸπŸŽβˆ’πŸ‘πŸ’ =1.168xπŸπŸŽπŸ“ m/s

@@ end@@

Page 49: AND ANSWERS SEM-I/II

C.REDDAPPA MODULE-3 INTERLINE PUBLISHING.COM

VTU ENGINEERING PHYSICS ( 17 PHY 12/22 ) SEM-I/II Page 1

MODULE-3 : LASERS AND OPTICAL FIBRES

LASERS

Q: Explain Einstein’s explanation of interaction of radiation with matter (or)

Explain Induced absorption , Spontaneous emission &Stimulated emission.

Let N1 & N2 be the population of the energy states E1 & E2 respectively

so that (E2 βˆ’E1) = hΞ½ & E2 >E1

According to Einstein radiation interacts with matter in 3 ways namely:

1) Induced absorption:

Induced absorption is the phenomenon in which an atom(A) in the lower energy state E1

absorb the incident photon of energy β€˜hν’ & excite to the higher energy state E2

if (E2 βˆ’E1) = hΞ½ .

Mathematically it(induced absorption) is represented as

hΞ½ +A β†’ π‘¨βˆ— or photon +atomβ†’ π‘Žπ‘‘π‘œπ‘šβˆ—

Also, Rate of induced absorption = 𝐡12 𝑁1π‘ˆπœˆ ,where π‘ˆπœˆ = π‘’π‘›π‘’π‘Ÿπ‘”π‘¦ 𝑑𝑒𝑛𝑠𝑖𝑑𝑦 &

𝐡12 = 𝐸𝑖𝑛𝑠𝑑𝑒𝑖𝑛′𝑠 𝑖𝑛𝑑𝑒𝑐𝑒𝑑 π‘Žπ‘π‘ π‘œπ‘Ÿπ‘π‘‘π‘–π‘œπ‘› π‘π‘œπ‘’π‘“π‘“π‘–π‘π‘–π‘’π‘›π‘‘.

2.Spontaneous emission:

Spontaneous emission is the phenomenon in which an atom (A) in the excited state of

energy E2 de-excite to the lower energy state E1 without any external influence by

emitting a photon of energy hΞ½ =(E2 βˆ’E1).

Mathematically, it is represented as π‘¨βˆ— β†’ A + hΞ½

Also, Rate of Spontaneous emission = 𝐴21 𝑁2

π‘€β„Žπ‘’π‘Ÿπ‘’ 𝐴21 = 𝐸𝑖𝑛𝑠𝑑𝑒𝑖𝑛′𝑠 π‘ π‘π‘œπ‘›π‘‘π‘Žπ‘›π‘’π‘œπ‘’π‘  π‘’π‘šπ‘–π‘ π‘ π‘–π‘œπ‘› π‘π‘œπ‘’π‘“π‘“π‘–π‘π‘–π‘’π‘›π‘‘.

𝐸2 𝑁2 π‘¨βˆ— β€’

hΞ½ β‡’

𝐸1 β€’ 𝑨 𝑁2

𝐸2 π‘¨βˆ— β€’ 𝑁2

β‡’ hΞ½

𝐸1 𝑁1 𝑨 β€’

Page 50: AND ANSWERS SEM-I/II

C.REDDAPPA MODULE-3 INTERLINE PUBLISHING.COM

VTU ENGINEERING PHYSICS ( 17 PHY 12/22 ) SEM-I/II Page 2

3.Stimulated emission:

Stimulated emission is the phenomenon in which an atom (π‘¨βˆ—) in the excited state of

energy E2 de-excite to the lower energy state E1 under the influence of an external photon

(hΞ½) by emitting an identical photon of energy hΞ½ =(E2 βˆ’E1).

Mathematically,𝑖𝑑 𝑖𝑠 π‘Ÿπ‘’π‘π‘Ÿπ‘’π‘ π‘’π‘›π‘‘π‘’π‘‘ π‘Žπ‘  𝒉𝝂 + π‘¨βˆ— β†’ A +2 hΞ½

Also, Rate of Stimulated emission= 𝐡21 𝑁2π‘ˆπœˆ ,where π‘ˆπœˆ = π‘’π‘›π‘’π‘Ÿπ‘”π‘¦ 𝑑𝑒𝑛𝑠𝑖𝑑𝑦 &

𝐡21 = 𝐸𝑖𝑛𝑠𝑑𝑒𝑖𝑛′𝑠 π‘†π‘π‘œπ‘›π‘‘π‘Žπ‘›π‘’π‘œπ‘’π‘  π‘’π‘šπ‘–π‘ π‘ π‘–π‘œπ‘› π‘π‘œπ‘’π‘“π‘“π‘–π‘π‘–π‘’π‘›π‘‘.

Q: Explain the terms : Active medium ; Population Inversion ; Pumping ; Meta stable state

and Laser Cavity( Optical resonator) Or Explain the requisites of a laser system.

The requisites of a laser system are :

1. Active medium is a Solid/Liquid/Gas medium in which stimulated emission and

amplification of the radiations can be achieved.

2. Pumping is the supply of energy to the atoms in the lower states in order to excite them to

higher states. The methods of pumping are Optical pumping, Electrical pumping, Forward

bias pumping ,Chemical pumping, Elastic one-one collisions.

3. Population Inversion is condition of system in which the population of higher energy states

exceed the population of lower states.

4. Meta stable state is an intermediate state in which the average life of the atoms is of the

order of 10βˆ’3s ie: their life is 105 times more than that of normal states.

5. Laser Cavity( Optical resonator) is a pair of parallel/con-focal/concentric mirrors between

which active medium is placed so that stimulated emitting photons are used to cause

further Stimulated emissions and to amplify the beam.

One mirror is highly silvered and the other partially silvered. The distance between the

mirrors is given by L = π‘›πœ†

2 ,where Ξ» = wavelength and n = number of stationary waves

produced.

𝐸2 π‘¨βˆ— β€’ 𝑁2

hΞ½ hΞ½ + hΞ½

𝐸1 𝑁1 β€’A

𝐸2 𝑁2 β€’

Page 51: AND ANSWERS SEM-I/II

C.REDDAPPA MODULE-3 INTERLINE PUBLISHING.COM

VTU ENGINEERING PHYSICS ( 17 PHY 12/22 ) SEM-I/II Page 3

𝑸:Derive an expression for energy density in terms of Einstein’s coefficients (or)

Derive the relation between Einstein’s coefficients

.

Consider a system of atoms in thermal equilibrium with radiation of energy density

π‘ˆπœˆ & frequency β€˜Ξ½β€™. Let N1 & N2 be the population of the energy states E1 & E2

respectively, where (E2 >E1 )

We know that,

Rate of induced absorption = 𝐡12 𝑁1π‘ˆπœˆ ,

where 𝐡12 = 𝐸𝑖𝑛𝑠𝑑𝑒𝑖𝑛′𝑠 π‘π‘œπ‘’π‘“π‘“π‘–π‘π‘–π‘’π‘›π‘‘ π‘œπ‘“ 𝑖𝑛𝑑𝑒𝑐𝑒𝑑 π‘Žπ‘π‘ π‘œπ‘Ÿπ‘π‘‘π‘–π‘œπ‘› .

Rate of Spontaneous emission = 𝐴21 𝑁2 π‘€β„Žπ‘’π‘Ÿπ‘’ 𝐴21 =

𝐸𝑖𝑛𝑠𝑑𝑒𝑖𝑛′𝑠 π‘π‘œπ‘’π‘“π‘“π‘–π‘π‘–π‘’π‘›π‘‘ π‘œπ‘“ π‘ π‘π‘œπ‘›π‘‘π‘Žπ‘›π‘’π‘œπ‘’π‘  π‘’π‘šπ‘–π‘ π‘ π‘–π‘œπ‘› π‘Žπ‘›π‘‘

Rate of Stimulated emission= 𝐡21 𝑁2π‘ˆπœˆ ,

where 𝐡21 = 𝐸𝑖𝑛𝑠𝑑𝑒𝑖𝑛′𝑠 π‘π‘œπ‘’π‘“π‘“π‘–π‘π‘–π‘’π‘›π‘‘ π‘œπ‘“ π‘†π‘‘π‘–π‘šπ‘’π‘™π‘Žπ‘‘π‘’π‘‘ π‘’π‘šπ‘–π‘ π‘ π‘–π‘œπ‘› .

At thermal equilibrium,

Rate of induced absorption = π‘…π‘Žπ‘‘π‘’ π‘œπ‘“ π‘†π‘π‘œπ‘›π‘‘π‘Žπ‘›π‘’π‘œπ‘’π‘ 

π‘’π‘šπ‘–π‘ π‘ π‘–π‘œπ‘› +

π‘…π‘Žπ‘‘π‘’ π‘œπ‘“ π‘†π‘‘π‘–π‘šπ‘’π‘™π‘Žπ‘‘π‘’π‘‘ π‘’π‘šπ‘–π‘ π‘ π‘–π‘œπ‘›

ie: 𝐡12 𝑁1π‘ˆπœˆ = 𝐴21 𝑁2 + 𝐡21 𝑁2π‘ˆπœˆ

∴ π‘ˆπœˆ( 𝐡12 𝑁1 βˆ’ 𝐡21 𝑁2) = 𝐴21 𝑁2

𝑖𝑒: π‘ˆπœˆ = 𝐴21 𝑁2

( 𝐡12 𝑁1βˆ’ 𝐡21 𝑁2) dividing both Nr & Dr by 𝐡21 𝑁2 ,we get

= 𝐴21

𝐡21

1

[ 𝐡12 𝐡21

𝑁1𝑁2

βˆ’1] ……….(1)

According to Boltzmann’s law, 𝑁1

𝑁2 = 𝑒(𝐸2 βˆ’πΈ1)/𝐾𝑇 = 𝑒 β„Žπœˆ/𝐾𝑇 …….(2)

From eqns 1 &2 , we get. π‘ˆπœˆ = 𝐴21

𝐡21

1

[ 𝐡12

𝐡21 π‘’β„Žπœˆ/𝐾𝑇 βˆ’1]

……..(3)

But .the energy density given by Planck’s law is π‘ˆπœˆ = 8πœ‹β„Žπœˆ3

𝐢3

1

( π‘’β„Žπœˆ/𝐾𝑇 βˆ’1) ……(4)

Comparing eqns 3 & 4 ,we get π‘¨πŸπŸ

π‘©πŸπŸ =

πŸ–π…π’‰π‚πŸ‘

π‘ͺπŸ‘ or

𝐴21

𝐡21 = π‘ˆπœˆ ( 𝑒

β„Žπœˆ

𝐾𝑇 βˆ’ 1) and π‘©πŸπŸ = π‘©πŸπŸ

Thus coefficient of stimulated absorption = coefficient of stimulated emission.

Thus energy density 𝑼𝝂 = π‘¨πŸπŸ

π‘©πŸπŸ

𝟏

[𝒆𝒉𝝂/𝑲𝑻 β€“πŸ]

Induced absorption Stimulated emission

𝐸2 𝑁2 β€’ β€’

hΞ½ hΞ½ hΞ½ 2hΞ½

𝐸1 β€’ 𝑁1 β€’ β€’

Spontaneous emission

Page 52: AND ANSWERS SEM-I/II

C.REDDAPPA MODULE-3 INTERLINE PUBLISHING.COM

VTU ENGINEERING PHYSICS ( 17 PHY 12/22 ) SEM-I/II Page 4

Q:Explain the fundamental mode of vibration in CO2 molecule.

In CO2 molecule there are three fundamental modes of vibrations, namely

1. Symmetric mode :

Symmetric mode is the mode in which both the oxygen atoms oscillate simultaneously to

& fro about the stationary carbon atom along the molecular axis.

2. Asymmetric mode:

Asymmetric mode is the mode in which both the oxygen atoms move in one direction and

the carbon atom move in the opposite direction along the molecular axis.

3. Bending mode:

Bending mode is the mode in which both oxygen atoms and carbon atom move in opposite

directions perpendicular to the molecular axis.

The internal vibrations of CO2 molecule are the combination of the above three modes.

Molecular axis

O O C

Molecular axis

O O C

Molecular axis

C

O O

Page 53: AND ANSWERS SEM-I/II

C.REDDAPPA MODULE-3 INTERLINE PUBLISHING.COM

VTU ENGINEERING PHYSICS ( 17 PHY 12/22 ) SEM-I/II Page 5

Q:Explain the principal, construction and working of Cπ‘ΆπŸ laser.

Q:Explain the construction & working of CO2 Laser.

Principle : CO2 laser works on the principle of stimulated emission.

Construction:

1) The schematic diagram CO2 Laser is as shown in the diagram invented by CKN Patel an

Indian engineer

2) It consists of a (glass)discharge tube of length 5 m & diameter 2.5 cm filled with a

mixture of gases CO2 ,N2 ,He in the ratio 1:2:3

3) High DC voltage can be applied to the gas between the electrodes A&C .

4) Ends of the tube is fitted with ( NaCl ) Brewster windows to get polarized laser beam

5) Two con-focal silicon mirrors coated with aluminum are provided at the ends of the tube

which act as optical resonators.

6) Cold water is circulated through a tube surrounding the discharge tube

π‘ͺπ‘ΆπŸ laser diagram

C𝑂2 𝑁2 𝐻𝑒 gases outlet

Water outlet

𝑀1 -optical cavity(oc) 𝑀2 (oc)

5 m

A electrodes C 2.5 cm Laser

BW BW=Brewster window

Water inlet Ba

Energy level diagram

𝑁2 Resonant transfer of energy C𝑂2

𝜈 = 1 𝐢5 (meta stable state)

𝐢4 10.6 πœ‡π‘š laser

Excitations by collission 𝐢3 9.6 πœ‡π‘š laser

𝐢2

𝜈 = 0 Ground state 𝐢1

Page 54: AND ANSWERS SEM-I/II

C.REDDAPPA MODULE-3 INTERLINE PUBLISHING.COM

VTU ENGINEERING PHYSICS ( 17 PHY 12/22 ) SEM-I/II Page 6

Working:

1) CO2 Laser is a four level molecular gas laser which produce continuous or pulsed laser

beam.

2) It works on the principle of stimulated emission between the rotational sublevels of an

upper & lower vibrational levels of CO2 molecules.

3) Ionisation takes place due to electric discharge when high DC voltage is applied between

electrodes producing electrons.

4) The accelerated electrons excite both N2& CO2 atoms to their higher energy levels β€˜ π“₯ =1 &

C5 from their ground states 0 & C1 due to collision as follows:

e+π‘΅πŸ β†’ π‘΅πŸβˆ—

+e’ and e+π‘ͺπ‘ΆπŸ β†’ π‘ͺπ‘ΆπŸβˆ—

+e’

where e& e ’are the energies of electron before and after collision.

5) π‘΅πŸβˆ— molecule in excited level collide with CO2 molecules in their ground state C1 &

excite it to metastable state C5 by resonant energy transfer as level C5 of CO2 is same as

level π“₯=1 of 𝑁2 given by π‘΅πŸβˆ— +π‘ͺπ‘ΆπŸ β†’ π‘ͺπ‘ΆπŸ

βˆ—+π‘΅πŸ

6) As this process continues due to electric discharge pumping , population inversion takes

place betweenC5 &C4 and C5 & C3.

7) The transitions/de-excitations takes place as follows:

C5 β†’ C4 producing laser 10.6πœ‡π‘š (IR region)

C5 β†’ C3 producing laser 9.6πœ‡π‘š (IR region)

C4 β†’ C2

C3 β†’ C2 Radiation less transitions

C2 β†’ C1

8) Due to high thermal conductivity of He, it removes heat from mixture and de-populate the

lower states C3 &C2 quickly .

9) Laser beam is amplified by using optical resonators.

10) The laser output is 100kW for continuous mode and 10 kW in pulsed mode.

Q: Explain the construction and working of Semi-conductor laser.

Semiconductor laser Energy level giagram Metalic coated(MC) surface P-type pn-Jn n-type RS 𝐸𝐹𝑒 PS CB Ba p P Laser 𝐸𝐹𝑝 h+e

n-type VB Polished surfac(PS) Laser Roughened surface(RS) MC

P-type

Pn-jn

n-type

Page 55: AND ANSWERS SEM-I/II

C.REDDAPPA MODULE-3 INTERLINE PUBLISHING.COM

VTU ENGINEERING PHYSICS ( 17 PHY 12/22 ) SEM-I/II Page 7

Principle: SC laser works on the principle of stimulated emission.

Construction:

1. The schematic diagram of GaAs semi-conductor device is as shown in the diagram.

2. It consists of heavily doped n-region of GaAs doped with tellurium and p-region of GaAs

doped with zinc.

3. The upper and lower surfaces are metalized so that pn-junction is forward biased .

4. Two surfaces perpendicular to the Jn are polished so that they act as optical resonators

and the other two surfaces roughened to prevent lasing in that direction.

Working:

1. Semi-conductor laser are made up of highly de-generate semi-conductors having direct

band gap like Gallium Arsenide (GaAs).

2. When GaAS diode is forward biased with voltage nearly equal to the energy gap voltage,

electrons from n-region & holes from p-region flow across the junction creating

population inversion in the active jn region.

3. As the voltage is gradually increased due to forward biasing population inversion is

achieved between the valence band and conduction band which in turn result in

stimulated emission.

4. Photons produced are amplified between polished optical resonator surfaces producing

laser beam.

5. GaAs laser produce laser beam of wavelength 8870Γ… in IR region , GaAsP produce laser

beam of 6500Γ… in visible region etc.

Q:Mention the characteristics of laser beam/light.

The laser beam characteristics are:

1. They are highly monochromatic.

2. They are highly coherent.

3. They are highly directional.

4. They are highly focusable.

5. They are least divergent.

Q:Mention the uses of laser beam.

Laser beam is used in Holography, Laser welding, Laser cutting laser drilling and

measurement of atmospheric pollutants.

Page 56: AND ANSWERS SEM-I/II

C.REDDAPPA MODULE-3 INTERLINE PUBLISHING.COM

VTU ENGINEERING PHYSICS ( 17 PHY 12/22 ) SEM-I/II Page 8

Q: What is holography ?

Holography is the process in which the details of an object in 3-dimensions can

be recorded on a 2-dimensional aid based on the principle of interference of light.

Q: Construction/Recording of Hologram.

1. The schematic diagram for the construction of a Hologram is as shown in the diagram.

2. A laser beam of wavelength (Ξ») is made to fall on beam splitter, which split the beam in to

two beams. One beam which passes through splitter is called β€˜reference beam’. The other

reflected beam is made to fall on the object whose hologram is to be produced.

3. The beam reflected by the object is called β€˜spherical object beam’.

4. The reference and object beams produce concentric circular rings called β€œGaber zones

β€œwhere both intensity and phase are recorded due to interference on the photographic

plate.

5. On developing photographic plate we obtain β€˜hologram’.

6. At each and every point on the hologram, complete information/details of the object are

recorded.

7. If the hologram is cut in to any number of pieces, each piece produce the complete image

of the object with less resolution.

Q: Re-construction of Hologram.

Object

Reflected beam Object spherical beam

Laser beam (Ξ») Hologram

Beam splitter reference beam

Hologram Observation direction

Laser beam (Ξ»)

Object Virtual image Object Real image

Page 57: AND ANSWERS SEM-I/II

C.REDDAPPA MODULE-3 INTERLINE PUBLISHING.COM

VTU ENGINEERING PHYSICS ( 17 PHY 12/22 ) SEM-I/II Page 9

1. The schematic diagram for the reconstruction of the image from Hologram is as shown in

the diagram.

2. The hologram is illuminated with the same laser beam of wavelength(Ξ») which used for

construction of hologram.

3. The hologram act as diffraction grating. Due to diffraction and interference two images of

the object are produced.

4. One which is formed on the side of incident laser is virtual image and the other formed on

the other side is the real image of the object.

5. By changing the direction of the observation ,we can see all the details of the object

originally hidden from view.

Q:Explain laser welding

1. The schematic arrangement of laser welding is as shown in the diagram.

2. High intensity and high focussability of lasers is used for laser welding.

3. Laser beam is focused on to the spot to be welded. Due to generation of high heat the

material melts in a short period & the impurities float to the surface.

4. On cooling the joint, it becomes homogeneous stronger joint.

5. As laser welding is contactless welding ,no foreign materials enter in to the joint.

6. Laser welding can be done more precisely by using pre-programmed computer assisted

welding.

7. CO2 laser can be used to weld both metallic and non-metallic substances.

Q:Explain laser cutting:

𝑂2/𝑁2 inlet

Condenser lens

material

Laser beam

Condenser lens

Materials to be weld

Page 58: AND ANSWERS SEM-I/II

C.REDDAPPA MODULE-3 INTERLINE PUBLISHING.COM

VTU ENGINEERING PHYSICS ( 17 PHY 12/22 ) SEM-I/II Page 10

1. The schematic arrangement of laser cutting is as shown in the diagram.

2. High intensity and high focussability of lasers is used for laser cutting.

3. Laser cutting involve melting and gas assisted blowing out the material.

4. Laser beam focused on to the surface to be cut ,melts it and the high speed

gas O2 /N2 passed through the nozzle blows away the molten material. O2

gas need low power laser than for N2.

5. This process continues till the material is cut.

6. Laser cutting can be done more precisely by using pre-programmed computer

assisted cutting.

7. CO2 laser can be used to cut both metallic and non-metallic substances

Q:Explain laser drilling.

1..The schematic arrangement of laser cutting is as shown in the diagram.

2.High intensity and high focussability of lasers is used for laser cutting.

3.The pulsed laser beam is focused to the spot where the hole to be drilled.

High heat generated melts the material.

4.Melted material absorb the heat further and vaporizes.

5.with continues heating by the laser vapours ionize into plasma state.

6. The plasma state material further absorb laser and emit black body radiations,

which in turn generate detonation waves. These waves and high power pressure

expel the material out of hole.

7.. Laser drilling can be done more precisely by using pre-programmed computer

assisted drilling.

8. CO2 laser can be used to drill both metallic and non-metallic substances.

Laser beam

Condenser lens

material

Page 59: AND ANSWERS SEM-I/II

C.REDDAPPA MODULE-3 INTERLINE PUBLISHING.COM

VTU ENGINEERING PHYSICS ( 17 PHY 12/22 ) SEM-I/II Page 11

Q:Measurement of atmospheric pollutants using LIDAR(Light Detection And Ranging).

1. The schematic experimental arrangement to measure atmospheric pollutants is as shown

in the diagram.

2. LIDAR (LIght Detection And Ranging)method is used to find the concentration of

atmospheric pollutants using laser.

3. Laser beam is sent in to the atmosphere in a particular direction and the scattered laser

beam by the pollutants are detected using detector.

4. The received scattered laser beam is analysed using analyser.

5. The measured intensity gives the concentration of the pollutants in a particular distance

and direction.

6. This method does not give information regarding the nature of the pollutants.

7. In order to know the nature of pollutants we have to adopt Raman spectroscopy method .n

this comparing the spectra of atmospheric pollutants and comparing them with standard

spectra of pollutants, we can identify pollutants.

OPTICAL FIBRES:

Q:What is an Optical Fibre?

Optical Fibre is a transparent di-electric material (like glass/plastic) which guides/

carry) light along it based on the principle of total reflection of light.

Optical fibre consists of a cylindrical transparent di-electric material of high refractive

index called core. It is surrounded by another di-electric transparent material of low

refractive index called cladding Cladding in turn is surrounded by cylindrical insulator

called Sheath, which gives mechanical strength & protect the fibre from absorption,

scattering etc

Atmosphere

β€’ β€’ β‹… β€’ β€’ β€’ β‹… β€’

pollutants

β€’ β€’ β‹… β€’ β€’ β€’ β‹… β€’

laser Detector

Analyser

Page 60: AND ANSWERS SEM-I/II

C.REDDAPPA MODULE-3 INTERLINE PUBLISHING.COM

VTU ENGINEERING PHYSICS ( 17 PHY 12/22 ) SEM-I/II Page 12

Q:What is an acceptance angle of an optical fibre ? Find the expression for acceptance

angle or Numerical aperture.

Acceptance angle is the maximum angle submitted by the ray with the axis of the fibre so

that light can be accepted and guided along the fibre.

Let 𝑛1 , 𝑛2 & π‘›π‘œ be the RI of core, cladding and launch medium respectively. Also OA incident

ray,AB refracted ray,BC totally reflected ray, πœƒπ‘–, πœƒπ‘Ÿ & βˆ… be the angles of incidence, refraction at A & angle of

incidence at B respectively.

By snell’s law at A, π‘›π‘œ 𝑠𝑖𝑛 πœƒπ‘– = 𝑛1 𝑠𝑖𝑛 πœƒπ‘Ÿ 𝑠𝑖𝑛 πœƒπ‘– = 𝑛1

π‘›π‘œπ‘ π‘–π‘› πœƒπ‘Ÿ ……(1)

But, from βˆ†π‘™π‘’ 𝐴𝐷𝐡, πœƒπ‘Ÿ =(90 βˆ’βˆ…) 𝑠𝑖𝑛 πœƒπ‘Ÿ = 𝑠𝑖𝑛(90 βˆ’ βˆ…)=cos(βˆ…) … . (2)

when πœƒπ‘– 𝑖𝑠 π‘šπ‘Žπ‘₯π‘–π‘šπ‘’π‘š = πœƒπ‘œ , π‘Žπ‘π‘π‘’π‘π‘‘π‘Žπ‘›π‘π‘’ π‘Žπ‘›π‘”π‘™π‘’, π‘‘β„Žπ‘’π‘› βˆ… = βˆ…πΆ Critical angle ……..(3)

From eqns 1,2 &3 ,we get 𝑠𝑖𝑛 πœƒπ‘œ = 𝑛1

π‘›π‘œπ‘π‘œπ‘ (βˆ…πΆ) …….(4)

Also, 𝑠𝑖𝑛(βˆ…πΆ) =𝑛2

𝑛1 and π‘π‘œπ‘ (βˆ…πΆ) = √(1 βˆ’ 𝑠𝑖𝑛2βˆ…πΆ) =

βˆšπ‘›12βˆ’π‘›2

2

𝑛1 …(5)

From eqns 4 &5,we get 𝑠𝑖𝑛 πœƒπ‘œ =βˆšπ‘›1

2βˆ’π‘›22

π‘›π‘œ or πœƒπ‘œ = π‘ π‘–π‘›βˆ’1

βˆšπ‘›12βˆ’π‘›2

2

π‘›π‘œ

for air π‘›π‘œ = 1

Q: What is meant by numerical aperture(NA) and mention the expression for it.

Numerical aperture is the ability of the optical fibre to accept the light and guide

along the fibre and is numerically equal to sine of the acceptance angle.

ie: NA = 𝑠𝑖𝑛 πœƒπ‘œ = βˆšπ‘›1

2βˆ’π‘›22

π‘›π‘œ

Launch medium (π‘›π‘œ ) Fibre axis

A

πœƒπ‘–

O

𝛷 Core (𝑛1)

πœƒπ‘Ÿ

D

B Cladding ( π’πŸ )

Page 61: AND ANSWERS SEM-I/II

C.REDDAPPA MODULE-3 INTERLINE PUBLISHING.COM

VTU ENGINEERING PHYSICS ( 17 PHY 12/22 ) SEM-I/II Page 13

Q:What is meant by fractional refractive index change(βˆ†).

The ratio of the difference between refractive indices of core and cladding to that

of core is called fractional refractive index change. ie: βˆ† =(𝑛1βˆ’π‘›2 )

𝑛1

Q:What is attenuation and explain types of attenuation in optical fibres.

Loss of power of light signal as it is guided along the fibre is called attenuation.

Attenuation is measured in terms of dB/km.

There are three types of attenuations in the fibre namely:

1. Absorption losses are the losses due to impurities & material itself and they are

two types namely

a) Impurity losses are the losses due to the impurities(Cu, Fe, etc) present in the

fibre, which can be minimised by taking care during manufacture of the fibre.

b) Intrinsic losses are the losses due to the material itself, these losses decreases with the

increase of wavelength.

2. Scattering losses are the losses due to imperfections of the fibre called Rayleigh

scattering losses which varies inversely as the πœ†4.

3. Radiation losses are the losses are two types namely:-

a) Microscopic losses are the losses due to non-linearity of the fibre axis ,which can be

minimized by providing compressible jacket & taking care during manufacture of

the fibre.

a) Macroscopic losses are the losses due to large curvature/bending of the fibre when it is

wound over a spool/bent at corners. These losses increase exponentially up to

threshold radius and there afterwards losses becomes large.

Q: Obtain expression for attenuation coefficient in an optical fiber of length L

Lambert’s law states that the rate of decrease of intensity of light with distance(βˆ’π‘‘π‘ƒ

𝑑𝐿 )

in a medium is directly proportional to the original intensity(P)

ie: βˆ’π‘‘π‘ƒ

𝑑𝐿 ∝ 𝑃

𝑑𝑃

𝑑𝐿 = βˆ’π›Όπ‘ƒ …(1) ,where 𝛼 is called the attenuation coefficient.

Rewriting(1 ) as 𝑑𝑃

𝑃 = βˆ’π›Ό. 𝑑𝐿 ,

Integrating this between the limits (𝑃𝑖 , π‘ƒπ‘œ) π‘“π‘œπ‘Ÿ 𝑃 π‘Žπ‘›π‘‘ (0, 𝐿)π‘“π‘œπ‘Ÿ 𝐿,

βˆ«π‘‘π‘ƒ

𝑃= βˆ’π›Ό

,π‘ƒπ‘œ

π‘ƒπ‘–βˆ« 𝑑𝐿

𝐿

0

[π‘™π‘œπ‘”10P]𝑃𝑖

,π‘ƒπ‘œ = βˆ’π›Ό [L]0𝐿

π‘™π‘œπ‘”10 (,π‘ƒπ‘œ

𝑃𝑖) = βˆ’π›ΌL……(2) where 𝑃𝑖 , π‘ƒπ‘œ are 𝑖𝑛 & out put powers.

Equation (2),can be written as 𝜢 = βˆ’πŸπŸŽ

𝑳 π’π’π’ˆπŸπŸŽ (

,𝑷𝒐

π‘·π’Š) dB/km.

or 𝑷𝒐

π‘·π’Š= 𝒆

βˆ’πœΆπ‘³

𝟏𝟎

Page 62: AND ANSWERS SEM-I/II

C.REDDAPPA MODULE-3 INTERLINE PUBLISHING.COM

VTU ENGINEERING PHYSICS ( 17 PHY 12/22 ) SEM-I/II Page 14

Q:What is meant by modes of propagation.

The paths along which the light is guided in the fibre are called modes of

propagation and the number of modes of the fibre is given by N = 𝑉2

2

Q: What is V-Parameter/number ?

It is the quantity which represent the number of modes of the fibre given by

V = πœ‹π‘‘

πœ† . NA =

πœ‹π‘‘

πœ† .

βˆšπ‘›12βˆ’π‘›2

2

𝑛0

where d = diameter , Ξ» = wavelength,

𝑛1, 𝑛2 &𝑛0 RI’s of Core, Cladding & medium

Q: Explain the construction and working of Types of optical Fibres.

There are three types of optical fibres namely:-

1) Single mode step index fibre(SMF

1. SMF has a core diameter 8 βˆ’ 10πœ‡π‘š of uniform RI and cladding diameter

60-70πœ‡π‘š has π‘’π‘›π‘–π‘“π‘œπ‘Ÿπ‘š 𝑅𝐼

2. The step index, cross section, modes and pulse profiles of SMF are as shown in the

diagram.

3. SMF guides light in a single mode as shown.

4. The V-number is < 2.4.

5. Numerical aperture is < 0.12 .

6. Attenuation is in the range 0.25-0.5 dB/km.

7. Information carrying capacity is very large. They are long haul carriers.

8. The output and input pulses are almost same.

9. Laser source is used. Connectors are costly

Cladding

Core

Input output

Index profile Cross section profile Modes profile Pulse profile

Page 63: AND ANSWERS SEM-I/II

C.REDDAPPA MODULE-3 INTERLINE PUBLISHING.COM

VTU ENGINEERING PHYSICS ( 17 PHY 12/22 ) SEM-I/II Page 15

2) Multimode step index fibre(MMF)

Input output

1) MMF has a core diameter 50 βˆ’ 200πœ‡π‘š of uniform RI and cladding diameter

100-250πœ‡π‘š has π‘’π‘›π‘–π‘“π‘œπ‘Ÿπ‘š 𝑅𝐼.

2) The step index, cross section,modes and pulse profiles are as shown in the diagram.

3) MMF guides light in multi-modes as shown .

4) The V-number is > 2.4.

5) Numerical aperture is 0.2 π‘‘π‘œ 0.3 .

6) Attenuation is in the range 0.5- 4 dB/km.

7) Information carrying capacity is small to medium and short haul carriers.

8) The output pulse is widened.

9) LED source is used & Connectors are cheap.

3) Graded index multimode fibre(GRIN

1) GRIN has a core diameter 50 βˆ’ 200πœ‡π‘š of variable RI and cladding diameter 100-250πœ‡π‘š

has π‘’π‘›π‘–π‘“π‘œπ‘Ÿπ‘š 𝑅𝐼.

2) The graded index, cross section ,modes and pulse profiles are as shown in the diagram.

3) MMF guides light in multi-modes as shown in the diagram.

4) The V-number is > 2.4.

5) Numerical aperture is 0.2 π‘‘π‘œ 0.3 .

Cladding

Core

Input output

Index profile Cross section profile Modes profile Pulse profile

Cladding

Core

Input output

Index profile Cross section profile Modes profile Pulse profile

Page 64: AND ANSWERS SEM-I/II

C.REDDAPPA MODULE-3 INTERLINE PUBLISHING.COM

VTU ENGINEERING PHYSICS ( 17 PHY 12/22 ) SEM-I/II Page 16

6) Attenuation is in the range 0.5- 4 dB/km.

7) Information carrying capacity is large and efficient and are short haul carriers.

8) The output pulse and input pulse are as shown.

9) Laser/LED source is used. Connectors are cheap.

10) Easy to splice and interconnect but expensive.

Q:Explain point to point communication system using optical fibre.

The schematic block diagram of point to point communication system using optical fibre is

as shown in the diagram.

Note: AVS =audio/video signal. AES=analog electrical signal,

BES=binary electrical signal & OS = optical signal.

1. Information receiver-receives , convert input AVS in to AES & fed to coder.

2. Coder- receives, convert AES in to BES and fed in to optical transmitter after

modulating it with carrier signal.

3. Optical transmitter-receives, convert BES in to OS and fed in to carrier optical fibre.

4. Carrier optical fibre-receive OS and guide it along the fibre. Weakened OS is fed

in to repeater.

4. Re-peater( Receiver cum transmitter)-receives the Weakened OS, restore to original

strength and fed back in to carrier optical fibre again, which in turn guide OS and fed in

to optical receiver.

6. Optical receiver- receive ,convert OS in to BES & fed in to de-coder.

7. De-coder-receive, de-modulate & convert BES in to AES & fed in to information

transmitter.

8. Information transmitter-finally receive, convert AES in to AVS as output

Input

Audio-video

Signal

Output

Audio-video

Signal

Information

receiver

AVS AES

Coder

AES BES

Modulator

Optical

transmitter

BES OS

Optical fibre

carrier REPEATER

[ Receiver

Cum

Transmitter ]

Information

transmitter

AES AVS

De-Coder

BES AES

De- modulator

Optical

receiver

OS BES

Optical fibre

carrier

Page 65: AND ANSWERS SEM-I/II

C.REDDAPPA MODULE-3 INTERLINE PUBLISHING.COM

VTU ENGINEERING PHYSICS ( 17 PHY 12/22 ) SEM-I/II Page 17

PROBLEMS SECTION

MODULE-3 : LASERS AND OPTICAL FIBRES

Formulae needed: L = π‘›πœ†

2 ; 𝛼 = βˆ’

10

𝐿 log[

π‘ƒπ‘œ

𝑃𝑖 ] ;

𝑁2

𝑁1 = π‘’βˆ’(β„ŽπΆ/πœ†π‘˜π‘‡)

; E= P.t and E =n.β„ŽπΆ

πœ†

;

NA = βˆšπ‘›1

2βˆ’π‘›22

π‘›π‘œ ; Sinπœƒπ‘œ = NA ; V =

πœ‹π‘‘

πœ† X NA ; N =

𝑉2

2 ; βˆ† =

(𝑛1βˆ’π‘›2)

𝑛1 ;

1. Find the number of modes of standing waves in the resonator cavity of length 1 m in He-

Ne temperature of laser operating at wavelength 632.8nm.( JAN2015)

Given: L= 1 m ; Ξ» = 632.8 nm = 632.8π‘₯10βˆ’9 m. n = ?

Using L = π‘›πœ†

2 ,we get n =

2𝐿

πœ†

= 2π‘₯1

632.8π‘₯10βˆ’9

= 3.161 x 106 π‘šπ‘œπ‘‘π‘’π‘ 

2. A fiber with an input power of 9ΞΌW has a loss of 1.5 dB/km. If the fiber is 3000 m long,

calculate the output power.(JAN2015)

Given: 𝑃𝑖 = 9πœ‡π‘€ = 9π‘₯10βˆ’6𝑀 ,𝛼 = 1.5 𝑑𝐡/π‘˜π‘š ,L = 3000m = 3 km , π‘ƒπ‘œ = ?

Using 𝛼=βˆ’10

𝐿 log ⟦

π‘ƒπ‘œ

π‘ƒπ‘–βŸ§ we get π‘ƒπ‘œ = 𝑃𝑖 𝑒

βˆ’π›ΌπΏ

10

= 9π‘₯10βˆ’6π‘’βˆ’1.5π‘₯3

10

= 5.74 π‘₯10βˆ’6𝑀

3. Find the ratio of populations of two energy levels in a laser if the transition between

them produces light of wavelength 6493Γ…,assuming the ambient temperature as

27.(Jun/Jul 14)

Given: Ξ» = 6493Γ… = 6493x10βˆ’10 m, t=27 T = 273+27 =300K; h=6.63x10βˆ’34 Js;

k=1.38x10βˆ’23 J/K ;C=3x108m/s ;𝑁2

𝑁1 = ?

Using 𝑁2

𝑁1 = 𝑒

βˆ’(β„ŽπΆ

πœ†π‘˜π‘‡)

= π‘’βˆ’(

6.63π‘₯10βˆ’34π‘₯3π‘₯108

6493π‘₯10βˆ’10π‘₯1.38π‘₯10βˆ’23π‘₯300)

= π‘’βˆ’79.99

= 7.33x10βˆ’33

Page 66: AND ANSWERS SEM-I/II

C.REDDAPPA MODULE-3 INTERLINE PUBLISHING.COM

VTU ENGINEERING PHYSICS ( 17 PHY 12/22 ) SEM-I/II Page 18

4. The numerical aperture of an optical fibre is 0.2 when surrounded by air. Determine the

R.I of its core, given the R.I of the cladding is 1.59.Also find the acceptance angle when the

fibre is in water of R.I 1.33(Jun/Jul 14)

Given: π‘›π‘œπ‘₯ NA = 0.2 , 𝑛2 = 1.59 , π‘›β€²π‘œ = 1.33 , 𝑛1 =? a𝑛𝑑 πœƒβ€²π‘œ = ?

For air , Using NA = βˆšπ‘›1

2βˆ’π‘›22

π‘›π‘œ , we get 𝑛1 = √(π‘›π‘œπ‘₯𝑁𝐴)2 + 𝑛2

2

= √( 0.22 + 1.592 )

= 1.603

Also πœƒβ€²π‘œ = π‘†π‘–π‘›βˆ’1(

βˆšπ‘›12βˆ’π‘›2

2

π‘›β€²π‘œ

) = π‘†π‘–π‘›βˆ’1( √1.6032βˆ’1,592

1.33 ) = 8.65Β°

5. A pulse laser has an average power output 1.5mW per pulse and pulse duration is

20ns.The number of photons emitted per pulse is estimated to be 1.047 x108.Find the

wavelength of the emitted laser. (Dec 2013/Jan2014)

Given: P=1.5mW = 1.5x10βˆ’3 𝑀 ,t=20 ns =20x10βˆ’9 s , C=3x108m/s ; h=6.63x10βˆ’34 Js,

n = 1.047 x108 ,Ξ» = ?

Using E= Pxt and E = n.β„ŽπΆ

πœ† , 𝑀𝑒 𝑔𝑒𝑑 , Ξ» =

π‘›β„ŽπΆ

𝑃𝑑

= 1.047 π‘₯108π‘₯6.63π‘₯10βˆ’34π‘₯3π‘₯108

1.5π‘₯10βˆ’3 π‘₯20π‘₯10βˆ’9

= 6.942x10βˆ’7 m.

6. The angle of acceptance of an optical fibre is 30Β° π‘€β„Žπ‘’π‘› kept in air. Find the angle of

acceptance when it is in a medium of refractive index 1.33. (Dec 2013/Jan2014)&(Jun/Jul

2013) (𝐽𝑒𝑛

𝐽𝑒𝑙2011)

Given: πœƒπ‘œ =30Β° , π‘›β€²π‘œ=4/3=1.333 , π‘›π‘œ = 1 π‘“π‘œπ‘Ÿ π‘Žπ‘–π‘Ÿ , πœƒβ€²π‘œ =?

Using π‘›β€²π‘œπ‘†π‘–π‘› πœƒβ€²π‘œ = π‘›π‘œSinπœƒπ‘œ = βˆšπ‘›12 βˆ’ 𝑛2

2 = constsnt

We get , πœƒβ€²π‘œ = π‘†π‘–π‘›βˆ’1 (π‘›π‘œπ‘†π‘–π‘›πœƒπ‘œ π‘›β€²π‘œβ„ )

= π‘†π‘–π‘›βˆ’1 (1 𝑆𝑖𝑛30Β° 1.333⁄ )

=22.03Β°

7. Calculate the NA,V-number and number of modes in an optical fibre of core diameter

50ΞΌm,core and cladding refractive indices 1.41 and 1.4 at wavelength 820 nm. (𝐽𝑒𝑛 2012)

Given: 𝑛1 = 1.41 ; 𝑛2 = 1.4 ; Ξ» = 820 nm= 820 x 10βˆ’9 m ; d = 50ΞΌm =50x 10βˆ’6 m ;

π‘›π‘œ =1 for air NA = ? ; V = ? & N = ?

a. NA = βˆšπ‘›1

2βˆ’π‘›22

π‘›π‘œ =

√(1.41)2βˆ’(1.4)2

1 = 0 .168

b. V = πœ‹π‘‘

πœ† X NA =

πœ‹π‘₯50π‘₯ 10βˆ’6π‘₯0 .168

820 π‘₯ 10βˆ’9 = 32.18

c. N = 𝑉2

2 =

(32.17)2

2= 517.5 = 518

Page 67: AND ANSWERS SEM-I/II

C.REDDAPPA MODULE-3 INTERLINE PUBLISHING.COM

VTU ENGINEERING PHYSICS ( 17 PHY 12/22 ) SEM-I/II Page 19

8. The refractive indices of the core and cladding of a step index fibre are 1.45 and 1.40

respectively and its core diameter is 45ΞΌm.Calculate its relative refractive index

difference, V- number at wavelength 1000 nm and number of modes. (𝐷𝑒𝑐 2011) Given: 𝑛1 = 1.45 ; 𝑛2 = 1.4 ; Ξ» = 1000 nm= 1000 x 10βˆ’9 m ; d = 45ΞΌm =45x 10βˆ’6 m ; π‘›π‘œ = 1; βˆ† = ? ; V = ?

& N = ?

1) βˆ† =(𝑛1βˆ’π‘›2)

𝑛1 =

(1.45βˆ’1.4)

1.45= 0.0345

2) V = πœ‹π‘‘

πœ† 𝑋

βˆšπ’πŸπŸβˆ’π’πŸ

𝟐

𝒏𝒐=

3.14π‘₯45π‘₯ 10βˆ’6π‘₯√1.452βˆ’1.42

1000 π‘₯ 10βˆ’9 π‘₯1= 53.34

3) N = 𝑉2

2 =

(53.34)2

2= 1422.6 =1423

9. A He-Ne gas laser is emitting a laser beam with an average power of 4.5 mW. Find the

number of photons emitted per second by the laser. The wavelength of the emitted

radiation is 6328Γ…. (𝐷𝑒𝑐 11)

Given: P=4.5mW = 4.5x10βˆ’3 𝑀 ,t= 1 S , C=3x108m/s ; h=6.63x10βˆ’34 Js,

Ξ» = 6328Γ… = 6328x10βˆ’10 m ; n = ?

Using E= Pxt and E =n.β„ŽπΆ

πœ† 𝑛 =

πœ†π‘ƒπ‘‘

β„ŽπΆ

= 6328π‘₯10βˆ’10

π‘₯4.5π‘₯10βˆ’3π‘₯1

6.63π‘₯10βˆ’34 π‘₯3π‘₯108

= 1.432x1016 photons.

10. Find the number of modes of standing waves and their frequency separation in the

resonant cavity of 1 m length of He-Ne operating at wavelength of 632.8 nm. (𝐽𝑒𝑛/

𝐽𝑒𝑙2011)

Given: L= 1 m ; Ξ» = 632.8 nm = 632.8π‘₯10βˆ’9 m. n = ? and

Using L = π‘›πœ†

2 , we get , n =

2𝐿

πœ†

= 2π‘₯1

632.8π‘₯10βˆ’9

= 3.161 π‘₯ 106 π‘šπ‘œπ‘‘π‘’π‘ 

11. The ratio of population of two energy states in a laser 1.059 x10βˆ’30.If the temperature of

the system is 57°𝐢, π‘€β„Žπ‘Žπ‘‘ 𝑖𝑠 π‘‘β„Žπ‘’ π‘€π‘Žπ‘£π‘’π‘™π‘’π‘›π‘”π‘‘β„Ž π‘œπ‘“ π‘‘β„Žπ‘’ π‘™π‘Žπ‘ π‘’π‘Ÿ ?

Given: 𝑁2

𝑁1 = 1.059x10βˆ’30, t = 57°𝐢, T =273+57=330K; h=6.63x10βˆ’34 Js; k=1.38x10βˆ’23

J/K ;C=3x108m/s ; Ξ» = ?

Using 𝑁2

𝑁1 = π‘’βˆ’(β„ŽπΆ/πœ†π‘˜π‘‡) ,we get , In(

𝑁2

𝑁1) = βˆ’(β„ŽπΆ/πœ†π‘˜π‘‡) 𝑆𝑖𝑛𝑐𝑒: π‘™π‘œπ‘”π‘’ (

𝑁2

𝑁1) = In(

𝑁2

𝑁1)

Ξ» = βˆ’(β„ŽπΆ/𝐼𝑛 (𝑁2

𝑁1) π‘˜π‘‡)

= βˆ’6.63π‘₯10βˆ’34 π‘₯3π‘₯108

𝐼𝑛(1.059π‘₯10βˆ’30) π‘₯1.38π‘₯10βˆ’23π‘₯330

= 6.328x10βˆ’7 π‘š

Page 68: AND ANSWERS SEM-I/II

C.REDDAPPA MODULE-3 INTERLINE PUBLISHING.COM

VTU ENGINEERING PHYSICS ( 17 PHY 12/22 ) SEM-I/II Page 20

12. A signal with input power 200mW loses 10% of its power after travelling a distance of

3000 m. Find the attenuation coefficient of the fiber. (𝐷𝑒𝑐 2010)

Given: 𝑃𝑖 = 200π‘šπ‘Š = 200π‘₯10βˆ’3𝑀 , π‘ƒπ‘œ = 90% 𝑃𝑖 = 0.9π‘₯200π‘₯10βˆ’3𝑀 ;

L = 3000 m = 3 km ; 𝛼 =? ,

Using 𝛼= βˆ’10

𝐿 log[

π‘ƒπ‘œ

𝑃𝑖] = βˆ’

10

3 log [

0.9π‘₯200π‘₯10βˆ’3

200π‘₯10βˆ’3] = 0.1525 𝑑𝐡/π‘˜π‘š

13. Calculate on the basis of Einstein’s theory the number of photons emitted per second by

He-Ne laser source emitting light of wavelength 6328Γ… with an optical power

10mW. (𝐽𝑒𝑛 2010) Given: P=10mW = 10x10βˆ’3 𝑀 ,t=1 s , C=3x108m/s ; h=6.63x10βˆ’34 Js, ,

Ξ» = 6328 Γ… = 6328 x 10βˆ’10 m ; n = ?

Using E= Pxt and E =n.β„ŽπΆ

πœ† , 𝑀𝑒 𝑔𝑒𝑑 𝑛 =

πœ†π‘ƒπ‘‘

β„ŽπΆ

= 6328 π‘₯ 10βˆ’10 π‘₯10π‘₯10βˆ’3 π‘₯1

6.63π‘₯10βˆ’34 π‘₯3π‘₯108

= 3.18 x1016 /π‘š3

14. An optical fiber has core R.I 1.5 and R.I of cladding is 3% less than the core index. Calculate

the numerical aperture, angle of acceptance and internal critical acceptance

angle. (𝐽𝑒𝑛 2010)

Given: 𝑛1 = 1.5 ; 𝑛2 =97% 𝑛1 = 0.97 x1.5 =1.455 ; 𝑛0 = 1 ; 𝑁𝐴 = ? ; πœƒπ‘œ = ? & πœƒπΆ = ?

Using 1) NA = βˆšπ‘›1

2βˆ’π‘›22

π‘›π‘œ

= √1.52βˆ’1.4552

1 = 0.365

2) 𝑠𝑖𝑛 πœƒ0= 𝑁𝐴 πœƒπ‘œ = π‘†π‘–π‘›βˆ’1(𝑁𝐴)

= π‘†π‘–π‘›βˆ’1(0.365) = 21.41Β°

3) 𝑠𝑖𝑛 πœƒπΆ =𝑛2

𝑛1 πœƒπΆ = π‘†π‘–π‘›βˆ’1 (

𝑛2

𝑛1) = 𝑆𝑖𝑛

βˆ’1(1.455

1.5)

= 75.93Β°

A 5W pulsed laser emits light of wavelength 694 nm. If the duration of each pulse is

20 ns, calculate the number of photons emitted per pulse.( 4 marks

Given: P=5W , Ξ» =694 nm=694x10βˆ’9 m ,t=20 ns =20x10βˆ’9 s , C=3x108m/s ;

h=6.63x10βˆ’34 Js, n =?

Using E=P.t and E =n.β„ŽπΆ

πœ† ,

𝑀𝑒 𝑔𝑒𝑑 n = π‘ƒπ‘‘πœ†

β„ŽπΆ

= 5π‘₯20π‘₯10βˆ’9 π‘₯694π‘₯10βˆ’9

6.63π‘₯10βˆ’34 π‘₯3π‘₯108

=3.489x1011 photons/pulse.

Page 69: AND ANSWERS SEM-I/II

C.REDDAPPA MODULE-3 INTERLINE PUBLISHING.COM

VTU ENGINEERING PHYSICS ( 17 PHY 12/22 ) SEM-I/II Page 21

15. The angle of acceptance of an optical fiber kept in air is 30Β°.Find the angle of acceptance

when the fiber is in a medium of refractive index 4/3.( 4 marks)

Given: πœƒπ‘œ =30Β° , π‘›β€²π‘œ=4/3=1.333 , π‘›π‘œ = 1 π‘“π‘œπ‘Ÿ π‘Žπ‘–π‘Ÿ , πœƒβ€²π‘œ =?

Using π‘›β€²π‘œπ‘†π‘–π‘› πœƒβ€²π‘œ = π‘›π‘œSinπœƒπ‘œ =constant (βˆšπ’πŸπŸ βˆ’ π’πŸ

𝟐 ) for a fibre

We get πœƒβ€²π‘œ = π‘†π‘–π‘›βˆ’1 (

π‘›π‘œπ‘†π‘–π‘›πœƒπ‘œ

π‘›β€²π‘œ

)

= π‘†π‘–π‘›βˆ’1 (1 𝑆𝑖𝑛30Β°

1.333)

= 22.03Β°

16. The ratio of population of two energy levels out of which one corresponds to metastable

state is 1.059 x10βˆ’30.Find π‘‘β„Žπ‘’ π‘€π‘Žπ‘£π‘’π‘™π‘’π‘›π‘”π‘‘β„Ž π‘œπ‘“ π‘™π‘–π‘”β„Žπ‘‘ π‘’π‘šπ‘–π‘‘π‘‘π‘’π‘‘ π‘Žπ‘‘ 330𝐾.(CBCS-Jun/Jul16)

Given: 𝑁2

𝑁1 = 1.059x10βˆ’30, T =330K; h=6.63x10βˆ’34 Js; k=1.38x10βˆ’23 J/K ;

C=3x108m/s ; Ξ» = ?

Using 𝑁2

𝑁1 = π‘’βˆ’(β„ŽπΆ/πœ†π‘˜π‘‡)

we get , In(𝑁2

𝑁1) = βˆ’(β„ŽπΆ/πœ†π‘˜π‘‡) 𝑆𝑖𝑛𝑐𝑒: π‘™π‘œπ‘”π‘’ (

𝑁2

𝑁1) = In(

𝑁2

𝑁1)

Ξ» = βˆ’(β„ŽπΆ/𝐼𝑛 (𝑁2

𝑁1) π‘˜π‘‡)

= βˆ’6.63π‘₯10βˆ’34 π‘₯3π‘₯108

𝐼𝑛(1.059π‘₯10βˆ’30) π‘₯1.38π‘₯10βˆ’23π‘₯330

= 6.328x10βˆ’7 π‘š

17. The refractive indices of the core and cladding of a step index fibre are 1.45 and 1.40

respectively and its core diameter is 45ΞΌm.Calculate the refractive index change and

numerical Aperture.(CBCS-Jun/Jul16)

Given: 𝑛1 = 1.45 ; 𝑛2 = 1.4 ; ; d = 45ΞΌm =45x 10βˆ’6 m ; π‘›π‘œ = 1, βˆ† = ? ; & NA = ?

1) βˆ† = (𝑛1βˆ’π‘›2)

𝑛1

= (1.45βˆ’1.4)

1.45 = 0.0345

2) NA = βˆšπ’πŸ

πŸβˆ’π’πŸπŸ

𝒏𝒐

= √1.452βˆ’1.42

1 = 0.3775

Page 70: AND ANSWERS SEM-I/II

C.REDDAPPA MODULE-3 INTERLINE PUBLISHING.COM

VTU ENGINEERING PHYSICS ( 17 PHY 12/22 ) SEM-I/II Page 22

19.The average power output of a laser beam of wavelength 6500Γ… is 10 mW.Find

the number of photon emitted per second by the laser source.

Given: P =10mW = 10x10βˆ’3 𝑀 ; t=1 s ; C=3x108m/s ; h=6.63x10βˆ’34 Js ;

Ξ» = 6500 Γ… = 6500 x 10βˆ’10 m ; n = ?

Using E= Pxt and E =n.β„ŽπΆ

πœ†

We get, 𝑛 = πœ†π‘ƒπ‘‘

β„ŽπΆ

= 6500 π‘₯ 10βˆ’10 π‘₯10π‘₯10βˆ’3 π‘₯1

6.63π‘₯10βˆ’34 π‘₯3π‘₯108

=3.268 x 1016 π‘β„Žπ‘œπ‘‘π‘œπ‘›π‘ /π‘š3

20. An optical signal propagating in a fiber retains 85% of input power after

travelling a distance of 500 m in the fiber. Calculate the attenuation coefficient.

Given: π‘ƒπ‘œ = 85% 𝑃𝑖 = 0.85 𝑃𝑖 ; L = 500m = 0.5 km; 𝛼 =? ,

Using 𝛼 = βˆ’10

𝐿 log[

π‘ƒπ‘œ

𝑃𝑖]

= βˆ’10

0.5 log [

0.85 𝑃𝑖

𝑃𝑖] = βˆ’

10

0.5 log [0.85]

= 1.412 dB/km

21. The attenuation of light in an optical fiber is 2 dB/km. What fraction of its

intensity remains after (i) 2 km, (ii) 5 km ?

Given: 𝛼 = 2𝑑𝐡

π‘˜π‘š, (

π‘ƒπ‘œ

𝑃𝑖)

𝐿=2π‘˜π‘š=?, (

π‘ƒπ‘œ

𝑃𝑖)

𝐿=5π‘˜π‘š=?

Using (π‘ƒπ‘œ

𝑃𝑖)

𝐿 = 10

βˆ’π›ΌπΏ

10

(π‘ƒπ‘œ

𝑃𝑖)

𝐿=2π‘˜π‘š= 10

βˆ’π›ΌπΏ

10 = 10βˆ’2π‘₯2

10 = 0.3981

(π‘ƒπ‘œ

𝑃𝑖)

𝐿=5π‘˜π‘š= 10

βˆ’π›ΌπΏ

10 = 10βˆ’2π‘₯5

10 = 0.1

22. The refractive indices of the core and cladding of a step-index optical fibre

are 1.45 and 1.40 respectively and its core diameter is 45ΞΌm. Calculate its

fractional refractive index change and numerical aperture.

Given: 𝑛1 = 1.45 ; 𝑛2 = 1.4 ; ; d = 45ΞΌm =45x 10βˆ’6 m ; βˆ† = ? ; & NA = ?

1) βˆ† = (𝑛1βˆ’π‘›2)

𝑛1 =

(1.45βˆ’1.4)

1.45 = 0.0345

2) NA = βˆšπ’πŸ

πŸβˆ’π’πŸπŸ

𝒏𝒐

= √1.452βˆ’1.42

1

= 0.3775

Page 71: AND ANSWERS SEM-I/II

C.REDDAPPA MODULE-3 INTERLINE PUBLISHING.COM

VTU ENGINEERING PHYSICS ( 17 PHY 12/22 ) SEM-I/II Page 23

23. Find the ratio of population of two energy levels in a medium at thermal equilibrium ,

if the wavelength of light emitted at 291 K is 6928 Γ… . ( 04 Marks)

Given: Ξ» = 6928Γ… = 6928x10βˆ’10 m, t=27 T =291K; h=6.625x10βˆ’34 Js;

k=1.38x10βˆ’23 J/K ;C=3x108m/s ;𝑁2

𝑁1 = ?

Using 𝑁2

𝑁1 = 𝑒

βˆ’(β„ŽπΆ

πœ†π‘˜π‘‡)

= π‘’βˆ’(

6.625π‘₯10βˆ’34π‘₯3π‘₯108

6928π‘₯10βˆ’10π‘₯1.38π‘₯10βˆ’23π‘₯291)

= π‘’βˆ’71.44

= 9.419x10βˆ’32 or = 9.441x10βˆ’32

24. Calculate the numerical aperture and angle of acceptance for an optical fibe

having refractive indices 1.563 and 1.498 for core and cladding respectively. ( 04 Marks)

Given: 𝑛1 = 1.563; 𝑛2 = 1.498 ,π‘›π‘œ = 1 ; 𝑁𝐴 =? a𝑛𝑑 πœƒπ‘œ = ?

Using NA = βˆšπ‘›1

2βˆ’π‘›22

π‘›π‘œ

= √1.5632βˆ’1.4982

1

= 0.4461

Also πœƒπ‘œ = π‘†π‘–π‘›βˆ’1(𝑁𝐴 ) = π‘†π‘–π‘›βˆ’1(0.4461) = 26.49Β°

25. A pulsed laser emits photons of wavelength 820 nm with 22 nW average

power/pulse. Calculate the number of photons contained in each pulse,if the

pulse duration is 12 ns. ( 04 Marks)

Given: 𝝀 = 820 nm = 820 x 10βˆ’9 m , P = 22 mW = 22 x 10βˆ’3W ,

t = 12 ns = 12 x 10βˆ’9s , h = 6.625 x 10βˆ’34 JS , C = 3 x 108π‘šπ‘ βˆ’1 , n = ?

Using E = Pt and E = 𝑛 π‘₯ β„ŽπΆ

πœ†

We get , n =πœ†π‘ƒπ‘‘

β„ŽπΆ =

820 π‘₯ 10βˆ’9π‘₯ 22 π‘₯ 10βˆ’3π‘₯ 12 π‘₯ 10βˆ’9

6.625 π‘₯ 10βˆ’34π‘₯ 3 π‘₯ 108 = 1.089 x 109 electrons/π‘š3

Page 72: AND ANSWERS SEM-I/II

C.REDDAPPA MODULE-3 INTERLINE PUBLISHING.COM

VTU ENGINEERING PHYSICS ( 17 PHY 12/22 ) SEM-I/II Page 24

26. A glass clad fiber is made with core glass of refractive index 1.5 and cladding is

doped to give a fractional index difference of 0.0005.Determine the cladding index

and numerical aperture. ( 04 Marks)

Given: 𝑛1 = 1.5 ,βˆ† =0.0005 ,π‘›π‘œ = 1 , 𝑛2 = ? , 𝑁𝐴 =?

Using βˆ† =(𝑛1βˆ’π‘›2)

𝑛1

We get 𝑛2 = 𝑛1 βˆ’ βˆ† 𝑛1 = 1.5 – 0.0005π‘₯ 1.5 = 1.49925 (1.499)

Also using NA =βˆšπ‘›1

2βˆ’π‘›22

𝑛0 =

√1.52βˆ’1.499252

1 = 0.0474 (0.054)

@@end@@

Page 73: AND ANSWERS SEM-I/II

REDDAPA C MODULE-4 INTERLINE PUBLISHING.COM

VTU ENGINEERING PHYSICS(17PHY12/22) SEM-I/II Page 1

MODULE-4

CRYSTAL STRUCTURE :

β€’ β€’ β€’ β€’ β€’ β€’ β€’ β€’ β€’ β€’ β€’ β€’ β€’ β€’ β€’

Lattice sites/points

Space lattice :The 3 dimensional periodic array(arrangement) of geometrical points in

Space is called space lattice/crystal lattice. Each point is called lattice point/site which has

identical surroundings.

Basis/Primitive/Translational vectors: A co-ordinate system is used to represent

latticepoints . Three basic vectors , & 𝑐 used to represent lattice points along x,y & z-

axis are called Basis/Primitive Vectors.

Translational vectors are position vector of lattice point in space given by

= 𝑛1+ 𝑛2+ 𝑛3𝑐 , where 𝑛1, 𝑛2 & 𝑛3 π‘Žπ‘Ÿπ‘’ π‘–π‘›π‘‘π‘’π‘”π‘’π‘Ÿπ‘ .

Q: What is Bravais lattice & non-Bravais lattice ?

Bravais lattice –is the lattice in which all the lattice points are equivalent ,

each lattice point represent an identical set of one or more atoms/molecules.

Non-Bravais lattice- is the lattice in which all the lattice points are not equivalent .

Superposition/overlapping of two or more bravais lattice forms non-bravais

lattice.

Q: What is Basis /Pattern?

Unit assembly of identical composition of atoms/molecules is called

Basis/pattern.

Q: What is crystal structure?

Crystal structure is obtained if basis is added to each and every lattice point.

β€’ β€’ β€’ β€’

β€’ β€’ + = β€’ β€’

β€’ β€’ β€’ β€’

Lattice + Basis = Crystal structure

Page 74: AND ANSWERS SEM-I/II

REDDAPA C MODULE-4 INTERLINE PUBLISHING.COM

VTU ENGINEERING PHYSICS(17PHY12/22) SEM-I/II Page 2

Q: What is unit cell ? and explain types of cells.

Unit cell-is the smallest block/geometrical figure from which the crystal can be

built by repetition of it in three dimensions.

Primitive cell-is the unit cell having lattice points only at the

corners /vertices of it or is the unit cell containing only one lattice point in it.

Non-Primitive cell-is the unit cell which have lattice point within it in addition to

lattice points at the corners /vertices.

Note: All primitive cells are unit cells but all unit cells are not primitive cells.

Q: What are lattice parameters ?

The Six quantities that describe the unit cell completely are called lattice parameters.

Three basis vectors are π‘Ž , b & c and three interfacial angles 𝛼, 𝛽& 𝛾 π‘Žπ‘Ÿπ‘’ π‘π‘Žπ‘™π‘™π‘’π‘‘

π‘‘β„Žπ‘’ π‘™π‘Žπ‘‘π‘‘π‘–π‘π‘’ π‘π‘Žπ‘Ÿπ‘Žπ‘šπ‘’π‘‘π‘’π‘Ÿπ‘ .

Q:What are Miller Indices of a plane ?

Miller Indices are the three smallest integers which represent the position & orientation

of the crystal planes having the same ratio as the reciprocals of the intercepts of the plane

on x,y and z axes.

Q:What are Crystallographic direction and Crystallographic plane ?

Crystallographic direction is the line joining origin to a lattice point(s) and

Crystallographic plane is the plane passing through all lattice points in a particular

crystallographic direction.

Z c 𝛽 𝛼 0 b y a 𝛾 x

Page 75: AND ANSWERS SEM-I/II

REDDAPA C MODULE-4 INTERLINE PUBLISHING.COM

VTU ENGINEERING PHYSICS(17PHY12/22) SEM-I/II Page 3

Q: Explain the seven crystal systems.

Based on six lattice parameters all the crystals are classified in to seven crystal systems as

follows:-

1) Cubic 2) Tetragonal

3 ) Orthorhombic 4) Triogonal/Rhombohedral

5 ) Hexagonal 6) Monoclinic

7) Triclinic

π‘Žβ‰ π‘ ≠𝑐 c 𝛼 = 𝛽 = 𝛾=90 Types:SC/BaC/BCC/F 𝛽 0 𝛼 b Ex:KNO3, MgSO3 a 𝛾

π‘Ž=𝑏 =𝑐 a 𝛼 = 𝛽 = 𝛾 β‰ 90 Types:SC 𝛽 𝛼 Ex: As, Sb, 0 a a 𝛾

π‘Ž=𝑏 ≠𝑐 𝛼 = 𝛽 =90 , 𝛾=120 c Types: SC Ex:Zn,Mg, 𝛽 0 𝛼 a a 𝛾

π‘Žβ‰ π‘ ≠𝑐 𝛼 β‰  𝛽 β‰  𝛾 β‰ 90 c

Types:SC Ex: 5H2O CuSO4 𝛽 0 𝛼

a 𝛾 b

π‘Žβ‰ π‘ ≠𝑐 𝛼 = 𝛾 =90 β‰  𝛽 c Types:SC/BaC Ex: CaSO4,2H2O 𝛽 𝛼 b a 0 𝛾

π‘Ž=𝑏 =𝑐 a 𝛼 = 𝛽 = 𝛾=90 Types: SC/BCC/FCC 𝛽 𝛼 a Ex: Copper/Gold a 0 𝛾

π‘Ž=𝑏 ≠𝑐 c 𝛼 = 𝛽 = 𝛾=90 Types: SC/BCC Ex: SnO2,TiO2 𝛽 𝛼

0 a a 𝛾

Page 76: AND ANSWERS SEM-I/II

REDDAPA C MODULE-4 INTERLINE PUBLISHING.COM

VTU ENGINEERING PHYSICS(17PHY12/22) SEM-I/II Page 4

Q:Explain the procedure for finding miller indices.

1, Find the intercepts of the plane along X,Y & Z axes in terms of basis vectors π‘Ž,𝑏 &𝑐.

Ex: Let X,Y &Z axes intercepts are 3π‘Ž,2𝑏 &1𝑐 respectively.

2. Find the coefficients of π‘Ž,b &𝑐.

ie: 3,2 & 1.

3. Find the reciprocals of the coefficients of basis vectors.

ie:The reciprocals are 𝟏

πŸ‘ ,

𝟏

𝟐,

𝟏

𝟏

4. Find the LCM of the denominators .

ie: The LCM of 3,2 &1 is 6.

5. Multiply each reciprocal term by LCM & write the results within the brackets

like(h k l),which gives the miller indices of the plane.

ie:The miller indices are 𝟏

πŸ‘π’™ πŸ” ,

𝟏

πŸπ’™ πŸ”,

𝟏

πŸπ’™ πŸ” = 𝟐, πŸ‘, 𝟏 = (𝟐 πŸ‘ 𝟏)

CHARACTERISTICS OF THREE CUBIC LATTICES

Note: 1. For an intercept at infinity, the miller indices is zero (0)

2. For negative intercepts, the corresponding miller index is negative. ex: h or 2 (Read as

β€˜bar’ h or 2

3. The miller indices ( h k l ) do not represent a single plane but set of parallel planes.

Page 77: AND ANSWERS SEM-I/II

REDDAPA C MODULE-4 INTERLINE PUBLISHING.COM

VTU ENGINEERING PHYSICS(17PHY12/22) SEM-I/II Page 5

Q:Derive an expression for inter-planar distance/spacing in terms of miller indices.

1. Consider the plane ABC of a crystal which intersect the X,Y & Z axes at A,B & C

respectively.

2. Let ( h k l ) be the miller indices of the family of ABC planes.

3. Draw β€˜ON ’ βŠ₯π‘™π‘Ÿ to the plane ABC ,then ON=𝒅 represent inter-planar distance/spacing.

Then OA= π‘Ž

β„Ž , OB=

𝑏

π‘˜ &OC =

𝑐

𝑙 ,where π‘Ž,𝑏& 𝑐 are basis vectors

From right π‘Žπ‘›π‘”π‘™π‘’ βˆ†π‘™π‘’ OAN, Cos πœƒπ‘‹ =𝑂𝑁

𝑂𝐴 =

𝑑

π‘Ž/β„Ž =

β„Žπ‘‘

π‘Ž …….(1)

From βˆ†π‘™π‘’ OBN, Cos πœƒπ‘Œ =𝑂𝑁

𝑂𝐡 =

𝑑

𝑏/π‘˜ =

π‘˜π‘‘

𝑏 …….(2) and

also, from βˆ†π‘™π‘’ OCN, Cos πœƒπ‘ =𝑂𝑁

𝑂𝐢 =

𝑑

𝑐/𝑙 =

𝑙𝑑

𝑐 ……(3)

For orthogonal co-ordinates π‘π‘œπ‘ 2 πœƒπ‘‹+π‘π‘œπ‘ 2 πœƒπ‘Œ + π‘π‘œπ‘ 2 πœƒπ‘ = 1 ……(4)

From eqns 1,2,3 &4 we get, β„Ž2𝑑2

π‘Ž2 +

π‘˜2𝑑2

𝑏2+

𝑙2 𝑑2

𝑐2

= 1

ie: 𝑑2 [ β„Ž2

π‘Ž2 +

π‘˜2

𝑏2+

𝑙2

𝑐2 ] = 1

∴ d = 𝟏

√ π’‰πŸ

π’‚πŸ + π’ŒπŸ

π’ƒπŸ+ π’πŸ

π’„πŸ

For a cubic crystal π‘Ž=𝑏 =𝑐

∴ d= 𝒂

√ π’‰πŸ +π’ŒπŸ +π’πŸ

Z

C

N B Y

πœƒπ‘§ πœƒπ‘¦

0 πœƒπ‘₯

A X

Page 78: AND ANSWERS SEM-I/II

REDDAPA C MODULE-4 INTERLINE PUBLISHING.COM

VTU ENGINEERING PHYSICS(17PHY12/22) SEM-I/II Page 6

Q:Derive an expression for space lattice constant β€˜π‘Žβ€™ for a cubic lattice.

Let 𝜌 𝑏𝑒 π‘‘β„Žπ‘’ 𝑑𝑒𝑛𝑠𝑖𝑑𝑦 & M the molecular weight of a cubic crystal of lattice constantβ€™π‘Žβ€™.

Then, volume of the unit cell = π‘Ž3 and mass of unit cell =𝜌 π‘Ž3 …….(1)

If n be the number of molecules per unit cell and 𝑁𝐴 be the Avagadro’s number.

Then, mass of each molecule = (M/𝑁𝐴) and

mass of molecules in the unit cell=n (M/𝑁𝐴)….(2)

From eqns 1& 2,we get 𝜌 π‘Ž3 = n (M/𝑁𝐴) π’‚πŸ‘ =𝒏𝑴

𝝆𝑡𝑨

Q:Find the relation between atomic radius (R) and lattice constant(π‘Ž) for SC ,BCC & FCC

crystals:-

a) Simple cubic(SC):

If β€˜ r ’ be the rdius and β€˜ π‘Ž β€˜ be the lattice constant/side of the unit cell,

then from the diagram π‘Ž=2R π‘Ž/2 = R

a) Body centered cubic (BCC) :

In a bcc cube AB=BC=CD=π‘Ž and

AC=βˆšπ‘Ž2 + π‘Ž2=√2π‘Ž

Also from βˆ†π‘™π‘’ ACD , 𝐴𝐷2 = 𝐴𝐢2 + 𝐢𝐷2

ie: (4𝑅)2 = (2π‘Ž)2 +π‘Ž2

= 3π‘Ž2

4R = √3π‘Ž

π‘‚π‘Ÿ 𝑅 =√3π‘Ž

4

a

C

a B

D

4R

A

AA A

a

2 R

Page 79: AND ANSWERS SEM-I/II

REDDAPA C MODULE-4 INTERLINE PUBLISHING.COM

VTU ENGINEERING PHYSICS(17PHY12/22) SEM-I/II Page 7

c) Face centered cubic (FCC)

In a fcc cube AB=BC=π‘Ž and AC=4R

Also from βˆ†π‘™π‘’ ABC,

𝐴𝐢2 = 𝐴𝐡2 + 𝐡𝐢2

ie: (4𝑅)2 = (π‘Ž2 +π‘Ž2

= 2π‘Ž2

Or 4R =√2π‘Ž

Or R = √2π‘Ž

4 =

π‘Ž

2√2

Q:Find the number of atoms in unit SC.BCC & FCC cell.

SC : In SC cell each atom is shared by 8 neighbour cells & hence no of atoms per unit cell

due to Corner atoms = 1

8 x 8 =1

BCC: no of atoms per unit bcc cell= 1

8 x 8 corner atoms + one atom at the body = 1

FCC: no of atoms per unit fcc cell = 1

8 π‘₯ 8 π‘π‘œπ‘Ÿπ‘›π‘’π‘Ÿ π‘Žπ‘‘π‘œπ‘šπ‘  +

1

2 x 6 face atoms=1+3=4

Q: What is meant by Co-ordination number ? & mention the co-ordination number of SC,

BCC & FCC cells.

Co-ordination number of any atom in a crystal is the number of nearest equidistant

neighbouring atoms Surrounding that atom.

SC: Co-ordination number of SC =6

BCC: Co-ordination number of bcc = 8

FCC: Co-ordination number of fcc =12

a

B

a

C

4R

A

Page 80: AND ANSWERS SEM-I/II

REDDAPA C MODULE-4 INTERLINE PUBLISHING.COM

VTU ENGINEERING PHYSICS(17PHY12/22) SEM-I/II Page 8

SC BCC FCC

Q:What is meant by Atomic Packing Factor(APF) ?

Atomic packing factor is defined as the fraction of the volume of the unit cell occupied

by the atoms present in the unit cell.

ie: Atomic packing factor = (π‘›π‘œ π‘œπ‘“ π‘Žπ‘‘π‘œπ‘šπ‘  𝑖𝑛 π‘‘β„Žπ‘’ 𝑒𝑛𝑖𝑑 𝑐𝑒𝑙𝑙)π‘₯π‘‰π‘œπ‘™π‘’π‘šπ‘’ π‘œπ‘“ π‘’π‘Žπ‘β„Ž π‘Žπ‘‘π‘œπ‘š

(π‘‰π‘œπ‘™π‘’π‘šπ‘’ π‘œπ‘“ π‘‘β„Žπ‘’ 𝑒𝑛𝑖𝑑 𝑐𝑒𝑙𝑙)

ie: APF = 𝑛

π‘Ž3 π‘₯

4

3 πœ‹π‘…3 where a = Lattice constant & R = Radius of the atom

Q: Calculate the atomic packing factors of SC,BCC & FCC cells.

For SC, n =1, R = 𝒂

𝟐

Using, APF = 𝑛

π‘Ž3 π‘₯ 4

3 πœ‹π‘…3

= 1

π‘Ž3 π‘₯ 4

3 πœ‹(

π‘Ž

2 )3

= 1

π‘Ž3 π‘₯ 4

3 πœ‹

π‘Ž3

8

= πœ‹

6 = 0.52 or 52%

For BCC , n = 2, R = βˆšπŸ‘π’‚

πŸ’

Using, APF = 𝑛

π‘Ž3 π‘₯ 4

3 πœ‹π‘…3

= 2

π‘Ž3 π‘₯ 4

3 πœ‹(

√3π‘Ž

4 )3

= 2

π‘Ž3 π‘₯ 4

3 πœ‹

3√3π‘Ž3

64

= √3πœ‹

8= 0.68 or 68%

For FCC, n = 4, R = π‘Ž

2√2

Using, APF = 𝑛

π‘Ž3 π‘₯ 4

3 πœ‹π‘…3

= 4

π‘Ž3 π‘₯

4

3 πœ‹(

π‘Ž

2√2 )3

= 4

π‘Ž3 π‘₯ 4

3 πœ‹

π‘Ž3

16√2

= πœ‹

3√2 = 0.74 or 74%

β€’

β€’

β€’

Page 81: AND ANSWERS SEM-I/II

REDDAPA C MODULE-4 INTERLINE PUBLISHING.COM

VTU ENGINEERING PHYSICS(17PHY12/22) SEM-I/II Page 9

NOTE: Important data related to SC , BCC & FCC cells.

Sl.

No

Particulars Simple Body

Centere

Face

Centered

1 Volume of a unit cell π‘Ž3 π‘Ž3 π‘Ž3

2 No. of atoms per cell 1 2 4

3 Co-ordination number

(No. of nearest neighbours)

6 8 12

4 Nearest neighbour distance

(2R)

π‘Ž (√3/2)π‘Ž π‘Ž/√2

5 Packing Faction 0.52 0.68 0.74

Q:Explain the structure of diamond crystal.

Note: = 8 corner atoms, = 6 face atoms & = 4 body diagonal atoms

OR

1. Diamond is formed by two intervening FCC sub-lattices one of them is moved by π‘Ž

4

along the body diagonal as shown in the diagram.

2. The origin of one FCC is ( 0,0,0) and the origin of the other FCC is (a/4,a/4,a/4)

3. There are 8 atoms at the corners, 6 atoms at the faces and 4 atoms along the

diagonals.

4. Each diagonal atom form bond with One nearest corner atom and Three nearest face

atoms.

5. The coordination number of diamond is 4.

6. Total atoms in unit diamond cell = 1

8 x 8+

1

2 x 6+4 =8

7. If R be the radius and a the lattice constant ,then we can show that

O 1/2

O

1/2 O 1/2

O 1/2 O

3/4 1/4

1/4 3/4

B 2R A

π‘Ž

4

Page 82: AND ANSWERS SEM-I/II

REDDAPA C MODULE-4 INTERLINE PUBLISHING.COM

VTU ENGINEERING PHYSICS(17PHY12/22) SEM-I/II Page 10

AB=2R=βˆšπ‘Ž2

16 +

π‘Ž2

16+

π‘Ž2

16

= √3π‘Ž2

16

= √3π‘Ž

4 or

R= βˆšπŸ‘π’‚

πŸ–

For Diamond n = 8 & R = βˆšπŸ‘π’‚

πŸ–

∴ Using APF = 𝑛

π‘Ž3 π‘₯

4

3 πœ‹π‘…3

= 8

π‘Ž3 π‘₯ 4

3 πœ‹(

√3π‘Ž

8 )3

= 8

π‘Ž3 π‘₯ 4

3 πœ‹

3√3π‘Ž3

8π‘₯8π‘₯8

= πœ‹βˆš3

16

= 0.34 or 34%

Q: What is perovskite ? explain.

Perovskie is the common name for all the oxides of type ABπ‘ΆπŸ‘,where A & B

are different metals.

Ex: π΅π‘Žπ‘‡π‘–π‘‚3 ( π΅π‘Žπ‘Ÿπ‘–π‘’π‘š π‘‘π‘–π‘‘π‘Žπ‘›π‘Žπ‘‘π‘’) , πΆπ‘Žπ‘‡π‘–π‘‚3( calcium titanate) etc

The common structure of perovskite is as follows:

= 8 Ba/Ca atoms form are at the edges of the cell.,

= 6 𝑂3atoms are at the face of the cell and

= 1 𝑇𝑖 atom at the body centre of the cell.

Perovskites exhibit superconductivity.

Perovskies exhibit both piezo-electric & ferro-electric properties .

Perovskies are used as dielectric in capacitors.

Perovskies are used as piezo-electric in microphone.

Type II superconductors have Perovskite structure.

Page 83: AND ANSWERS SEM-I/II

REDDAPA C MODULE-4 INTERLINE PUBLISHING.COM

VTU ENGINEERING PHYSICS(17PHY12/22) SEM-I/II Page 11

Q: Explain Allotrophy and Polymorphism.

Allotrophy is the property of the material by the virtue of which it can have more

than one type of crystal structures.

All the structures will have the same chemical properties and different physical properties

Carbon,Sulphur ,Phosphorous exhibit allotrophy.

Ex: 1.Diamond and graphite are the allotrophic forms of carbon.

2. Diamond is hard and good insulator .

3. Graphite is soft and good conductor.

Polymorphism is the ability of the materials to crystallize in several solid phases that

posses different lattice structures at different temperatures.

Ex: Iron ( Alpha Iron)exhibit BCC structure from room temperature to 910.

Iron ( Gamma Iron)exhibit FCC structure from 910 to upto 400

Iron ( Delta Iron)exhibit BCC structure from 1400 to 1540( melting point.

All these phases are reversible.

Q:Derive Bragg’s law of diffraction.

Let the incident parallel x-rays of wavelength Ξ», incident on two atomic planes of

Separation d at glancing angles πœƒ and reflect along BC and EF respectively.

Draw BG & BH βŠ₯π‘™π‘Ÿ to DE & EF respectively.

From the βˆ†π‘™π‘’ BGE & BHE we have GE=BE.Sinπœƒ & 𝐻𝐸 = 𝐡𝐸. π‘†π‘–π‘›πœƒ

where BE = d ,interplanar distance.

∴ π‘π‘Žπ‘‘β„Ž π‘‘π‘–π‘“π‘“π‘’π‘Ÿπ‘’π‘›π‘π‘’, 𝑝𝑑 = 𝐺𝐸 + 𝐻𝐸

= 𝑑 π‘†π‘–π‘›πœƒ + 𝑑 π‘†π‘–π‘›πœƒ

= 2𝑑 π‘†π‘–π‘›πœƒ … . (1)

Also for constructive interference pd= nΞ» ……(2)

∴ π‘“π‘Ÿπ‘œπ‘š π‘’π‘žπ‘›π‘  1 &2 , 𝑀𝑒 𝑔𝑒𝑑 nΞ» = πŸπ’… π‘Ίπ’Šπ’πœ½ ,

Incident rays -AD A C reflected rays-CF

Glancing angle

D πœƒ πœƒ F

β‚€ β‚€ B β‚€ β‚€ β‚€

G πœƒ πœƒ H d = Interplanar spacing

β‚€ β‚€ β‚€ E β‚€ β‚€

Page 84: AND ANSWERS SEM-I/II

REDDAPA C MODULE-4 INTERLINE PUBLISHING.COM

VTU ENGINEERING PHYSICS(17PHY12/22) SEM-I/II Page 12

This equation is called the Bragg’s law

Q:Describe the construction & working of Bragg’s X-ray spectrometer.

The labeled schematic diagram of the Bragg’s X-ray Spectrometer is as shown in the

diagram.

To verify Bragg’s law:

1. Collimate the X-rays from X-ray tube by passing them through slits S1 & S2.

2. Allow the collimated X-rays to incident on the surface of a given crystal mounted

vertically at the centre of a turn table at a glancing angle πœƒ

3. The position of the crystal is noted using the vernier V1 and horizontal circular

scale.

4. Adjust the ionization chamber to receive the scattered X-rays and measure the

ionization current (I)which is a measure of intensity of the X-rays using electrometer E.

5. Repeat the experiment as before by increasing the glancing angle 𝜽 gradually and

note the corresponding ionization current each time.

6. Plot a graph of β€˜Iβ€˜v/s β€˜πœƒβ€² π‘Žπ‘›π‘‘ π‘“π‘Ÿπ‘œπ‘š π‘‘β„Žπ‘’ π‘”π‘Ÿπ‘Žπ‘β„Ž note down the glancing angles πœƒ1, πœƒ2& πœƒ3

for the 1st,2nd &3rd order diffraction respectively.

7 Then substituting for πœƒ1, πœƒ2& πœƒ3 in the Bragg’s equation 2d Sinπœƒ = π‘›πœ† ,

It will be found that ,Sin𝜽𝟏: π‘Ίπ’Šπ’πœ½πŸ: π‘Ίπ’Šπ’πœ½πŸ‘ = 𝟏: 𝟐: πŸ‘,which verifies Bragg’s law.

To find the nature of the crystal:

1. Collimate the X-rays from X-ray tube by passing them through slits S1 & S2.

2. Allow the collimated X-rays to incident on the surface of a given crystal mounted

vertically at the centre of a turn table at a glancing angle πœƒ. The position of the

Vernier- 𝑉1 Circular scale

Slits glancing angle-πœƒ Crystal target

𝑆1 𝑆2

X-rays Vernier-𝑉2

Collimated X-rays

𝑆3

𝑆4 E -electrometer

Ionisation chamber (D) Ba

Intensity

0 πœƒ

πœƒ1 πœƒ2 πœƒ3

Page 85: AND ANSWERS SEM-I/II

REDDAPA C MODULE-4 INTERLINE PUBLISHING.COM

VTU ENGINEERING PHYSICS(17PHY12/22) SEM-I/II Page 13

crystal is noted using the vernier V1 and horizontal circular scale

3. Adjust the ionization chamber to receive the scattered X-rays and measure the

ionization current (I) produced by the X-rays using electrometer E.

4. Adjust 𝑑100 plane of the crystal to the incident X-rays and find the glancing angle

πœƒ1 for 1st order Spectrum.

5. Repeat the experiment as before for the crystal surfaces 𝑑110 & 𝑑111 and find

the corresponding 1st order glancing angles πœƒ2 & πœƒ3 .

6. Then using bragg’s equation, 2d Sinπœƒ = π‘›πœ† ,

find π’…πŸπŸŽπŸŽ ∢ π’…πŸπŸπŸŽ ∢ π’…πŸπŸπŸ = 𝟏

π‘Ίπ’Šπ’πœ½πŸ :

𝟏

π‘Ίπ’Šπ’πœ½πŸ:

𝟏

π‘Ίπ’Šπ’πœ½πŸ‘

7. The nature of the crystal is identified from the ratios of 1

π‘†π‘–π‘›πœƒ1 :

1

π‘†π‘–π‘›πœƒ2:

1

π‘†π‘–π‘›πœƒ3

as follows:

For SC; 𝟏

π‘Ίπ’Šπ’πœ½πŸ :

𝟏

π‘Ίπ’Šπ’πœ½πŸ:

𝟏

π‘Ίπ’Šπ’πœ½πŸ‘ = 1:

𝟏

√𝟐 :

𝟏

βˆšπŸ‘

For BCC; 𝟏

π‘Ίπ’Šπ’πœ½πŸ :

𝟏

π‘Ίπ’Šπ’πœ½πŸ:

𝟏

π‘Ίπ’Šπ’πœ½πŸ‘ = 1:

𝟐

√𝟐 :

𝟏

βˆšπŸ‘

For FCC; 𝟏

π‘Ίπ’Šπ’πœ½πŸ :

𝟏

π‘Ίπ’Šπ’πœ½πŸ:

𝟏

π‘Ίπ’Šπ’πœ½πŸ‘ = 1:

𝟏

√𝟐 :

𝟐

βˆšπŸ‘

PROBLEMS SECTION

MODULE-4 : CRYSTAL STRUCTURE

Formulae needed : nΞ» = 2dSinπœƒ ; d= π‘Ž

βˆšβ„Ž2 +π‘˜2 +𝑙2 ;

For SC ,n=1& a = 2R , BCC, n=2 & a = 4

√3 R , FCC , n=4 & a = 2√2 𝑅

Atomic Packing Factor (APF) = 𝑛

π‘Ž3 x

4πœ‹π‘Ÿ3

3

1. Copper has fcc structure with atomic radius 0.127nm.Calculate the inter-planar

spacing for (3 2 1) plane.( JAN2015)

Given: r = 0.127 nm = 0.127x10βˆ’9π‘š , (β„Ž π‘˜ 𝑙 ) = ( 3 2 1),

π‘“π‘œπ‘Ÿ 𝑓𝑐𝑐 π‘Ž = 2√2 π‘Ÿ = 2√2 x 0.127x10βˆ’9π‘š , 𝑑 = ?

Using d = π‘Ž

βˆšβ„Ž2 +π‘˜2 +𝑙2

= 2√2 π‘₯ 0.127π‘₯10βˆ’9

√32 +22 +12

= 9.60 x 10βˆ’11 π‘š

Page 86: AND ANSWERS SEM-I/II

REDDAPA C MODULE-4 INTERLINE PUBLISHING.COM

VTU ENGINEERING PHYSICS(17PHY12/22) SEM-I/II Page 14

2. π·π‘Ÿπ‘Žπ‘€ the following planes in a cubic unit cell: i) (2 0 0) ii) (2 1 0)iii)(1 3 2)

3. The minimum order of Bragg’s reflection occurs at an angle of 20Β° in the plane

(2 1 2 ).Find the wavelength of X-rays if lattice constant is 3.614Γ…. (Dec 2013/Jan2014)

Given: n = 1 ; πœƒ = 20Β° ; ( h k l ) = ( 2 1 2 ) ; a = 3.614Γ… = 3.614x10βˆ’10 m ; Ξ» = ?

Using nΞ» = 2dSinπœƒ & d= π‘Ž

βˆšβ„Ž2 +π‘˜2 +𝑙2

We get , Ξ» = 2π‘Ž π‘†π‘–π‘›πœƒ

π‘›βˆšβ„Ž2 +π‘˜2 +𝑙2

= 2π‘₯3.614π‘₯10βˆ’10 𝑆𝑖𝑛20Β°

1π‘₯√22 +12+22 = 8.240 x 10βˆ’11 m

4. Monochromatic X-rays of wavelength 0.82 Γ… undergo first order reflection from a crystal

of cubic lattice with lattice constant 3 Γ… at a glancing angle of 7.855Β°. Identify the possible

planes which give rise to this reflection in terms of Miller indices. (𝐷𝑒𝑐 11 & 𝐽𝑒𝑛 βˆ’ 𝐽𝑒𝑙16)

Given: n = 1 ; πœƒ = 7.855Β° ; a = 3 Γ… = 3x10βˆ’10 m ; Ξ» = 0.82 Γ… =0.82 x 10βˆ’10 m ; ( h k l ) = ?

Using nΞ» = 2dSinπœƒ & d= π‘Ž

βˆšβ„Ž2 +π‘˜2 +𝑙2

We get , βˆšβ„Ž2 + π‘˜2 + 𝑙2 = 2π‘Ž π‘†π‘–π‘›πœƒ

π‘›πœ†

= 2π‘₯3π‘₯10βˆ’10 𝑆𝑖𝑛(7.855)

1π‘₯0.82 π‘₯ 10βˆ’10 = 1

Thus the possible planes ( h k l ) =( 1 0 0 ) ,(0 1 0 ) ,( 0 0 1)

(2 0 0) 𝐱 =

𝟏

𝟐, 𝐲 = ∞ & 𝑧 = ∞

Z Β½ 0 y X

(2 1 0) 𝐱 = βˆ’πŸ

𝟐, , 𝐲 = 𝟏 & 𝑧 = ∞

Z Β½

0 1 Y

X

(1 3 2 ) 𝐱 = 𝟏, 𝐲 = βˆ’πŸ

πŸ‘ & 𝑧 =

𝟏

𝟐

Z Β½

βˆ’1

3

0 Y 1 X

Page 87: AND ANSWERS SEM-I/II

REDDAPA C MODULE-4 INTERLINE PUBLISHING.COM

VTU ENGINEERING PHYSICS(17PHY12/22) SEM-I/II Page 15

5. Calculate the glancing angle of the (1 1 0) plane of a simple cubic crystal (a=2.814 Γ… )

corresponding to second order diffraction maximum for the x-rays of wavelength

0.710 Γ….

Given: n = 2 ; ( h k l ) = (1 1 0 ) ; a = 2.814Γ… = 2.814x10βˆ’10 m ; Ξ» = 0.710

Γ… =0.710 x 10βˆ’10 m ; πœƒ =?

Using nΞ» = 2dSinπœƒ & d= π‘Ž

βˆšβ„Ž2 +π‘˜2 +𝑙2

We get , πœƒ = π‘ π‘–π‘›βˆ’1( π‘›πœ†βˆšβ„Ž2 +π‘˜2 +𝑙2

2π‘Ž )

= π‘ π‘–π‘›βˆ’1(2π‘₯0.710 π‘₯ 10βˆ’10π‘₯√12 +12 +0

2π‘₯2.814π‘₯10βˆ’10 ) = 20.91Β°

6. Draw the following planes in the unit cube: i) ( 1 0 2 ) ii) ( 1 1 2 ). (𝐷𝑒𝑐 2010)

Ans: i) ( 1 0 2 ) π‘₯ = βˆ’1, 𝑦 = ∞ & 𝑧 =1

2 & ii) (1 1 2 ) π‘₯ = βˆ’1, 𝑦 = 1 & 𝑧 = βˆ’

1

2

7. A monochromatic X-ray beam of wavelength 1.5Γ… undergoes second order Bragg

reflection from the plane (2 1 1) of a cubic crystal, at a glancing angle of 54.38Β°,calculate

the lattice constant. (𝐽𝑒𝑛 2010)

Given: n = 2 ; ( h k l ) = (2 1 1 ) ; Ξ» = 1.5 Γ… =1.5 x 10βˆ’10 m ; πœƒ = 54.38Β° ; π‘Ž = ?

Using nΞ» = 2dSinπœƒ & d= π‘Ž

βˆšβ„Ž2 +π‘˜2 +𝑙2

We get , π‘Ž = π‘›πœ†βˆšβ„Ž2 +π‘˜2 +𝑙2

2π‘†π‘–π‘›πœƒ

= 2π‘₯1.5 π‘₯ 10βˆ’10 π‘₯√22 +12 +12

2π‘₯𝑆𝑖𝑛 54.38

= 4.52 x 10βˆ’10 π‘š

( 1 0 2 ) 𝒙 = βˆ’πŸ, π’š = ∞ & 𝑧 =𝟏

𝟐

Z

Β½ βˆ’1

0 Y

x

(1 1 2 ) 𝒙 = βˆ’πŸ, π’š = 𝟏 & 𝑧 = βˆ’πŸ

𝟐

Z βˆ’1

0 1 Y

X(βˆ’1

2)

Page 88: AND ANSWERS SEM-I/II

REDDAPA C MODULE-4 INTERLINE PUBLISHING.COM

VTU ENGINEERING PHYSICS(17PHY12/22) SEM-I/II Page 16

8. In a calcite crystal, second order Bragg’s reflections occur from the planes with d-spacing

3 Γ…,at a glancing angle of 24Β° .Calculate the path difference between X-rays reflected from

the two adjacent planes. Also calculate the wavelength of x-rays.( 4 marks)

Given: d = 3 Γ… =3x10βˆ’10 π‘š , πœƒ = 24Β° , n = 2 ,Pd = ? and Ξ» = ?

Path difference,Pd = 2d Sinπœƒ =2x3x10βˆ’10 xSin24Β° =2.44 x10βˆ’10 m

Also, Ξ» = 𝑃𝑑

𝑛

= 2π‘‘π‘†π‘–π‘›πœƒ

𝑛

= 2.44 π‘₯10βˆ’10

2

= 1.22 x10βˆ’10 m

9. The atomic radius of gold is 0.144nm.Determine the interplanar distance for ( 1 1 0)

planes assuming that gold belongs to FCC system.( 4 marks)

Given: r = 0.144 nm =0.144x10βˆ’9 m , (h k l)=(1 1 0),

For FCC a =2√2 π‘Ÿ =2√2 π‘₯0.144π‘₯10βˆ’9 π‘š , d = ?

Using d= π‘Ž

√(β„Ž2+π‘˜2+𝑙2 )

= 2√2 π‘₯0.144π‘₯10βˆ’9

√(12+12+02 )

= 2.88 x 10βˆ’10 π‘š

10. Calculate the glancing angle for incidence of of X-rays of wavelength 0.058 nm on the

plane o (1 3 2)of NaCl which results in 2nd order diffraction maxima taking the lattice

spacing as 3.81 Γ….

Given: n = 2 ; ( h k l ) = (1 3 2 ) ; a = 3.81Γ… = 3.81x10βˆ’10 m ; Ξ» = 0.058nm=0.058 x 10βˆ’9 m

; πœƒ =?

Using nΞ» = 2dSinπœƒ & d= π‘Ž

βˆšβ„Ž2 +π‘˜2 +𝑙2

We get , πœƒ = π‘ π‘–π‘›βˆ’1( π‘›πœ†βˆšβ„Ž2 +π‘˜2 +𝑙2

2π‘Ž )

= π‘ π‘–π‘›βˆ’1(2π‘₯0.058 π‘₯ πŸπŸŽβˆ’πŸ—π‘₯√12 +32 +22

2π‘₯3.81π‘₯10βˆ’10 )

= 34.72Β° ( also d = 1.018 X 10βˆ’10 m)

Page 89: AND ANSWERS SEM-I/II

REDDAPA C MODULE-4 INTERLINE PUBLISHING.COM

VTU ENGINEERING PHYSICS(17PHY12/22) SEM-I/II Page 17

11. Draw the following planes in a cubic unit cell.

i) (2 0 0) ii) (Ξ™ Δͺ 0) iii) (Ξ™ 0 2) iV) (Ξ™ Ξ™ 2)

12. An X-ray beam of wavwlength 0.7 Γ… undergoes first order Bragg’s reflection

from the plane ( 302) of a cubic crystal at glancing angle 35Β° , 𝒄𝒂𝒍𝒄𝒖𝒍𝒂𝒕𝒆 𝒕𝒉𝒆

lattice constant. ( 04 Marks)

Given: n = 1 ; ( h k l ) = ( 302 ) ; Ξ» = 0.7 Γ… =0.7 x 10βˆ’10 m ; πœƒ = 35Β° ; π‘Ž = ?

Using nΞ» = 2dSinπœƒ & d= π‘Ž

βˆšβ„Ž2 +π‘˜2 +𝑙2

We get , π‘Ž = π‘›πœ†βˆšβ„Ž2 +π‘˜2 +𝑙2

2π‘†π‘–π‘›πœƒ

= 1π‘₯0.7 π‘₯ 10βˆ’10 π‘₯√32 +02 +22

2π‘₯𝑆𝑖𝑛 35

= 2.200 x 10βˆ’10 π‘š [ Note: d = 6.102x10βˆ’11m]

(102)β‡’X = 1,Y = ∞ & 𝑍 = Β½ Z

Β½

0 Y

X 1

(112)β‡’X = 1,Y = 1 & 𝑍 = Β½

Z

Β½

0 1 Y

X 1

(Ξ™ Δͺ 0)β‡’X = 1 ,Y = βˆ’1 & 𝑍 = ∞

z

0 y

1

x

βˆ’1 0 Y

1

X

(200)β‡’X = Β½ ,Y = ∞ & 𝑍 = ∞ Z

0 Y 1/2

X

Page 90: AND ANSWERS SEM-I/II

REDDAPA C MODULE-4 INTERLINE PUBLISHING.COM

VTU ENGINEERING PHYSICS(17PHY12/22) SEM-I/II Page 18

13.Draw the crystal planes ( 102 ) ( 111 ) ( 011 ) and ( 002 ) in a cubic crystal. ( 04 Marks)

14. Find the Miller indices of a set of parallel planes which make intercepts in th

ratio 3a : 4b and parallel to z-axis and also calculate the interplanar distance of the

planes taking thr lattice to be cubic with a = b = c = 2 Γ…. ( 04 Marks)

Given: Intercepts 3a:4b: βˆžπ‘ , a= 𝑏 = 𝑐 =2Γ…=2 x 10βˆ’10m , (h k l ) =? ,d = ?

Coefficients of a,b and c are 3 ,4, ∞

Reciprocals of the coefficients are 1

3 ,

1

4 ,0

LCM 0f the denominators 3 & 4 is 12

Miller indices are 1

3π‘₯12 = 4 ,

1

4π‘₯12 = 3 , 0 x 12= 0

∴ ( h k l ) = ( 4 3 0 )

π΄π‘™π‘ π‘œ π‘€β„Žπ‘’π‘› π‘Ž = 𝑏 = 𝑐

We have d =π‘Ž

βˆšβ„Ž2+π‘˜2+𝑙2 ∴ d =

2 π‘₯ 10βˆ’10

√42+32+02 = 4 x 10βˆ’11m

(102) X =1/1 =1, Y =1/0 = ∞ and Z =1/2

Z

1/2

0 Y

1

X

(111) X =1/1 =1, Y =1/1 =1, and Z 1/1 =1,

Z 1

1

0 Y

X 1

(002) X =1/0 = ∞, Y =1/0 = ∞ and Z =1/2

Z

Β½

0 Y

X

(011) X =1/0 = ∞, Y =1/1 =1and Z =1/1 = 1

Z 1

0 1 Y

X

Page 91: AND ANSWERS SEM-I/II

REDDAPA C MODULE-4 INTERLINE PUBLISHING.COM

VTU ENGINEERING PHYSICS(17PHY12/22) SEM-I/II Page 19

14. Draw the following planes in a cubic unit cell,

i) ( 0 0 I ) ii) ( I Δ« 0 ) iii) ( I I 2) iV) ( 0 2 0 ) ( 04 Marks)

@@end@@

(112) x =1/1= 1, y=1/1=1 & z =1/2

z

Β½

0 1 y

X 1

(020) x =1/0= ∞, y =1/2 & z = 1/0 = ∞

Z

0 Β½ y

x

( I Δ« 0 ) x =1/1= 1, y =1/-1=βˆ’1 & z =1/0 =∞

z

0 1 y

X 1

(001) x =1/0= ∞, y=1/0=∞ & z =1/1=1

Z 1

0 y

X

Page 92: AND ANSWERS SEM-I/II

REDDAPPA C MODULE-5 INTERLINE PUBLISHING.COM

VTU ENGINEERING PHYSICS(17PHY 12/22) SEM-I/II Page 1

MODULE-5

SHOCK WAVES AND SCIENCE OF NANO MATERIALS:

SHOCK WAVES

Q:What are shock waves ? Explain.

Shock waves are the waves produced due to the sudden release/dissipation of energy.

Shock waves are the waves in which the pressure, density and temperature changes

are large

Ex: Shock waves are produced during the burst of crackers, Explosion of

dynamite/bombs, Volcanic eruptions, etc.

Q:Mention different types of shock waves.

There are FOUR types of shock waves namely :-

Stationary shock waves,

Moving shock waves,

Normal shock waves and

Oblique shock waves,

Q:Mention methods of producing shock waves.

Shock waves can be produced by the following methods ,namely By detonation

of crackers/explosives, by volcanic eruptions, supersonic objects/waves, by

Reddy shock tube in the laboratory.

Q:What are Acoustic waves? Mention the types of acoustic waves.

Acoustic waves are the longitudinal waves which travel with the speed of sound

in a medium (Solid/liquid/gas)

Acoustic waves are classified in to THREE types namely:

1. Infrasonic waves(Infrasonics) are the Acoustic waves of frequency less than 20 Hz.

2. Audible waves are the Acoustic waves of frequency between 20 Hz and 20 kHz.

3. Ultrasonic waves(Ultrasonics) are the Acoustic waves of frequency greater

than 20 kHz. (Elephants detect Infrasonics, Human ear detect Audible waves &

Bats/Dogs detec Ultrasonics)

Page 93: AND ANSWERS SEM-I/II

REDDAPPA C MODULE-5 INTERLINE PUBLISHING.COM

VTU ENGINEERING PHYSICS(17PHY 12/22) SEM-I/II Page 2

Q: Define Mach number, subsonic waves supersonic waves and Mach angle.

1. Mach Number(M) is the ratio of the speed of an object (V) through a fluid to the speed of

sound(a) in the fluid at that point. Mathematically, M = 𝑽

𝒂 , M is dimensionless quantity.

2. Subsonic waves are the mechanical waves whose speed is less than that of sound in the

same medium. mach number of Subsonic waves is less than 1.

Ex: Motor cycle, Bus, Train , aeroplanes etc produce subsonic waves.

3. Supersonic waves are the mechanical waves whose speed is greater than that of sound in

the same medium. for which the mach number of Supersonic waves is greater than 1.

Ex: Fighter planes, Rockets, Missiles,tornedo etc produce supersonic waves

4. Mach angle(𝝁) is the half the angle of cone of sound waves formed and is given by

𝝁 = π‘Ίπ’Šπ’βˆ’πŸ (𝟏

𝑴)

Q: Explain the basic conservation laws and Rankine-Hugonite/Normal shock wave

Relations.

There are three basic conservation laws namely Conservation of mass, Conservation

of momentum and Conservation of energy.

1. Conservation of mass states that the total mass of the system always remains constant as

the mass can neither be created nor destroyed.

Mathematically, πœŒπ‘£ = π‘π‘œπ‘›π‘ π‘‘π‘Žπ‘›π‘‘ or

∴ 𝜌1𝑣1 = 𝜌2𝑣2 , Where 𝜌1, 𝜌2densities & 𝑣1 , 𝑣2 velocities.

2. Conservation of momentum states that the sum total momentum of the system always

remain constant.

Mathematically,𝑃 + πœŒπ‘£2 = π‘π‘œπ‘›π‘ π‘‘π‘Žπ‘›π‘‘ π‘œπ‘Ÿ 𝑃1 + 𝜌1𝑣12 = 𝑃2 + 𝜌2𝑣2

2

Where 𝑃1, 𝑃2 pressures , 𝑣1, 𝑣2 velocities and 𝜌1, 𝜌2densities.

3. Conservation of energy states that the sum total energy of a system is always remains

constant.

Mathematically, β„Ž +𝑣2

2 = π‘π‘œπ‘›π‘ π‘‘π‘Žπ‘›π‘‘ π‘œπ‘Ÿ β„Ž1 +

𝑣12

2= β„Ž2 +

𝑣22

2

Where β„Ž1, β„Ž2 enthalpies and 𝑣1, 𝑣2 velocities.

Page 94: AND ANSWERS SEM-I/II

REDDAPPA C MODULE-5 INTERLINE PUBLISHING.COM

VTU ENGINEERING PHYSICS(17PHY 12/22) SEM-I/II Page 3

Q: RH equations or Normal shock wave equations.

RH equation /Normal shock wave equations are derived from basic conservation laws

,which relate the temperature ratio and density ratio in terms of pressure ratio for normal

shock waves.

The pressure ratio in terms of upstream Mach number M is given by π‘·πŸ

π‘·πŸ =

𝟐𝜸

𝜸+πŸπ‘€2 βˆ’

πœΈβˆ’πŸ

𝜸+𝟏 …….(1)

But temperature ratio, π‘»πŸ

π‘»πŸ = (

𝟏

𝑀2+

πœΈβˆ’πŸ

𝟐) (

𝟐𝜸

πœΈβˆ’πŸ 𝑀2 βˆ’ 1) (

𝟐(πœΈβˆ’πŸ)

(𝜸+𝟏)𝟐) ……..(2)

Substituting for 𝑀2 π‘“π‘Ÿπ‘œπ‘š π‘’π‘žπ‘› 1, 𝑖𝑛 π‘’π‘žπ‘› 2, 𝑀𝑒 𝑔𝑒𝑑

π‘»πŸ

π‘»πŸ =

[𝟏+πœΈβˆ’πŸ

𝜸+𝟏

π‘·πŸπ‘·πŸ

]π‘·πŸπ‘·πŸ

π‘·πŸπ‘·πŸ

+πœΈβˆ’πŸ

𝜸+𝟏

…..(3) ∡ 4ab+(π‘Ž βˆ’ 𝑏)2 = (π‘Ž + 𝑏)2

Also , density ratio, π†πŸ

π†πŸ =

π‘·πŸπ‘·πŸπ‘»πŸπ‘»πŸ

…….(4)

From eqns 1,3 &4,we get,

π†πŸ

π†πŸ

=

π‘·πŸ

π‘·πŸ

[𝟏 +𝜸 βˆ’ 𝟏𝜸 + 𝟏

π‘·πŸ

π‘·πŸ]

π‘·πŸ

π‘·πŸπ‘·πŸ

π‘·πŸ+

𝜸 βˆ’ 𝟏𝜸 + 𝟏

= (

𝜸+𝟏

πœΈβˆ’πŸ

π‘·πŸπ‘·πŸ

+𝟏

𝜸+𝟏

πœΈβˆ’πŸ+

π‘·πŸπ‘·πŸ

) ….(5)

eqns 3 & 5 are called RH relations/NS relations.

Page 95: AND ANSWERS SEM-I/II

REDDAPPA C MODULE-5 INTERLINE PUBLISHING.COM

VTU ENGINEERING PHYSICS(17PHY 12/22) SEM-I/II Page 4

Q:What is a (Reddy)shock tube ? Describe the construction and working of simple

Reddy shock tube .

1. Reddy Shock tube is a device used to produce and study shock waves in the laboratory.

2. Schematic labeled diagram of the original Reddy shock tube is as shown in the diagram.

Construction :

1. RST consists of a steel tube of length 100 cm and diameter 2.9 cm.

2. A diaphragm of thickness 0.1cm divides the tube in to two compartments of length 49 cm

fitted with piston called Driver section filled with driver gas. The other compartment of

length 51 cm is called Driven section filled with driven gas.

3. Sensor S fitted to driver section measures the rupture pressure 𝑃2,temparature𝑇2.

4. Two sensors 𝑆1 & 𝑆2 separated by a distance βˆ†π‘‹ fitted to driven section measures the

pressures 𝑃4,𝑃5 and temperatures 𝑇4, 𝑇5 respectively.

Working :

1. Driver section is filled with gas at high pressure and Driven section is filled with gas of

low pressure 𝑃1 & π‘‘π‘’π‘šπ‘π‘’π‘Ÿπ‘Žπ‘‘π‘’π‘Ÿπ‘’ 𝑇1 .

2. Diaphram is ruptured to produce shock waves by pushing the piston and the rupture

pressure 𝑃2 & temperature is measured using sensor S.

3. The time’𝒕′ taken by the shock wave to travel the distance β€˜x’ is measured using CRO. also

the pressures 𝑃4,𝑃5 and temperatures 𝑇4, 𝑇5 are measured using the sensors 𝑆1 & 𝑆2

respectively.

4. The speed of the shock waves is calculated using V = π‘₯

𝑑 .

5. Then the match number of the shock waves is calculated using M = 𝑉

π‘Ž ,where a is the

speed of sound at temperature 𝑇1.

6. Also the mach number M can be calculated using the RH relations π‘·πŸ

π‘·πŸ =

𝟐𝜸

𝜸+πŸπ‘€2 βˆ’

πœΈβˆ’πŸ

𝜸+𝟏 , by finding 𝑃2, 𝑃1 for the gas of known 𝛾.

Piston Diaphragm S 𝑆1( 𝑃4) π‘ π‘’π‘›π‘ π‘œπ‘Ÿπ‘  𝑆2(𝑃5)

𝑃2 𝑃1 x =7 cm

Driver section/gas Driven section/gas 2.9cm

Plunger

49 cm 51 cm

Page 96: AND ANSWERS SEM-I/II

REDDAPPA C MODULE-5 INTERLINE PUBLISHING.COM

VTU ENGINEERING PHYSICS(17PHY 12/22) SEM-I/II Page 5

Q: Uses of Shock waves.

1. Shock waves (SW)are used in the treatment of kidney stones.

2. SW are used in the pencil industry for softening of pencil wood and painting.

3. Sw are used in the extraction of sandal wood.

4. Sw sre used to rejunevate/activate dried bore wells.

5. Sw are used for needleless drug delivery.

6. Sw are used to push DNA in to the cell.

7. SW are used for the treatment of orthopedic diseases.

SCIENCE OF NANOMATERIALS:

Q:What are nano-particles?

Nano-particles are the material particles whose size is in the range of 1nm to 100 nm

and their properties are size dependant.

Q:What is mesoscopic state?

Mesoscopic state is the size/state of the matter at which its physical properties

changes and becomes size dependent.

Q:Explain different structures(3D,2D,1D & 0D) based on their energy density

graphically.

3D structure/Bulk metal :

1. The 3D structure is as shown in the diagram, which have all the 3 dimensions.

2. The density of states for 3D structure is given by g(E) dE = πŸ–βˆšπŸπ…π’ŽπŸ‘/πŸπ‘¬πŸ/𝟐

π’‰πŸ‘ 𝑑𝐸

Where m = mass, E = energy, h = Planck’s constant

3. The density of states g(E) increases with energy E is as shown in the graph

4. The electrons are not confined to any direction.

g(E)

0 E

Page 97: AND ANSWERS SEM-I/II

REDDAPPA C MODULE-5 INTERLINE PUBLISHING.COM

VTU ENGINEERING PHYSICS(17PHY 12/22) SEM-I/II Page 6

2D structure/Quantum Film/Well:

.

1. Quantum film/well is obtained when 3D structure reduced to nano scale in one dimension

as shown in the diagram.

2. The density of states for quantum well is given by g(E) 𝑑𝐸 = 4πœ‹π‘š

β„Ž2 dE where m = mass,

h =Planck’s constant.

3. For each quantum state the density of states is constant.

4. The variation of g(E) with E is as shown in the graph.

5. The electrons are confined to 1 direction.

1D structure/Quantum Wire :

0

1. Quantum wire is obtained when 3D structure reduced to nano Scale in two dimensions as

shown in the diagram.

2. The density of states for quantum well is given by g(E)dE = 2√2π‘š1/2πΈβˆ’1/2

β„Ž dE

where m = mass, E = energy, h = Planck const.

3. The variation of g(E) with E is as shown in the graph

4. The electrons are confined to 2 directions.

g(E)

0 𝐸1𝐸2𝐸3 𝐸

g(E)

0 𝐸1 𝐸2 𝐸3 𝐸

Page 98: AND ANSWERS SEM-I/II

REDDAPPA C MODULE-5 INTERLINE PUBLISHING.COM

VTU ENGINEERING PHYSICS(17PHY 12/22) SEM-I/II Page 7

0D structure /Quantum Dot :

0

1. Quantum Dot is obtained when 3D structure reduced to nano scale all the 3 dimensions

2. The variation of g(E) with E is as shown in the graph.

3. The electrons are confined to all the 3 directions

4. and g(E) has discrete structure as shown.

Q: Explain the TWO approaches/methods adopted to obtain nanoparticles.

Nanoparticals

TOP DOWN approach

1. In the TOP DOWN approach ,bulk material is crushed in to small pieces.

2. Crushed pieces are further grinded in to smaller and smallers pieces.

3. Crushing and grinding process is continued till nanoparticles are obtained.

BOTTAM UP approach

1. Basic atoms/molecules are grouped to form clusters/globules.

2. The clusters /globules are further grouped in to nanoparticles.

g(E)

0 𝐸1 𝐸2 𝐸4 𝐸

Top Down method

Bulk material

Powder

Nanoparticals

Bottom Up method

Nanoparticals

Clusters /Groups

Atoms/molecules

Page 99: AND ANSWERS SEM-I/II

REDDAPPA C MODULE-5 INTERLINE PUBLISHING.COM

VTU ENGINEERING PHYSICS(17PHY 12/22) SEM-I/II Page 8

Q: Explain Ball Mill method of synthesis/producing nanoparticles.

1. Ball mill method is a mechanical method based on β€œ Top Down ” approach to

synthesis nanoparticles of metals and alloys on large scale.

2. It consists of a steel container filled with the material whose nanoparticles to be produced

along with large number of heavy steel balls.

3. The container is capable of rotation about an axis inclined to the horizontal.

4. The steel balls rotate circularly about the axis and spin about their own axis. Due to these

motions of steel balls ,the material is continuously crushed and powdered.

5. As this process is continued finally we get powdered nanoparticles.

6. Merits : By ball mill method we can produce nanoparticles on large scale economically.

7. Demerits: Nanoparticles produced are irregular in shape and impure.

Q: Explain Sol-Gel method of synthesis/producing nanoparticles.

1. Sol-gel method is a wet chemical process in which bottom-up approach is adopted.

2. The precursor is dissolved in suitable liquid to form Sol.

Steel container Steel Balls

Nanopowder

Stand

Precursor + Solution = Sol + Dehydration = Gel

Spray/dip adding Surfactants Slow heating

Gelled spheres Zero gel

Substrate On Calcination

On calcination On calcination Dense ceramic

Nano film Nano powder

Page 100: AND ANSWERS SEM-I/II

REDDAPPA C MODULE-5 INTERLINE PUBLISHING.COM

VTU ENGINEERING PHYSICS(17PHY 12/22) SEM-I/II Page 9

3. Sol is then dehydrated to get Gel. 4. On slow drying of gel zerogel is obtained. 5. On calcinations zerogel change in to nano dense ceramic.

6. If surfactants are added to Sol, gelled spheres are formed,which on calcinations form nano

powder 7. If Sol sprayed on to substrate by spinning/dipping, gel is formed on the substrate, which

on calcinations form thin nano film. 8. By sol-gel method we can obtain pure nanoparticles.

Q: what are Carbon nano tubes? Mention and explain structures of different types of CNT’s..

1. Carbon nanotube(CNT) structure is imagined to be the cylinder formed by rolling a

hexagonal

graphene sheet of carbon atoms and then closing the ends with fullerene hemispheres.

2. There are three types of CNT structures namely Aramchair CNT,Zigzag CNT and Chiral

CNT.

Types of CNTs:

1.Sigle walled nanotubes(SWNT) consists of single graphene sheet.

2.Multiwalled nanotubes(MWNT) consists of nanotube with in

nanotubes.

Arm chair CNT Zig-Zag CNT Chiral CNT

Axis

Page 101: AND ANSWERS SEM-I/II

REDDAPPA C MODULE-5 INTERLINE PUBLISHING.COM

VTU ENGINEERING PHYSICS(17PHY 12/22) SEM-I/II Page 10

Q:Expain the production of CNTS by Arc method.

1. The schematic diagram of Arc discharge method is as shown in the diagram.

2. It consists of a chamber in to which two graphite electrodes separated by 1 mm and

diameter 5-20 πœ‡π‘š are sealed.

3. Helium gas is circulated through the chamber at a pressure of

500 torrand a voltage of 20-25V can be applied between the electrodes.

4. When a voltage of 20-25 V is applied between the electrodes, graphite evaporates and is

deposited on the cathode.

5. If the anode is coated with catalytic agents like Iron, cobalt or nickel ,SWCNT’s are

produced.

6. MWCNT’s are produced if the anode is not coated with catalytic agents.

7. Pure CNT’s can be obtained by using pure graphite rods.

Q:Expain the production of CNTS by Pyrolysis method.

π‘΅πŸ π‘ͺπŸπ‘―πŸ

1. The schematic diagram of Pyrolisis method is as shown in the diagram.

𝐻𝑒 π‘”π‘Žπ‘  𝑖𝑛𝑙𝑒𝑑 𝐻𝑒 gas outlet

Chamber

Graphite anode

500 torr pressure 5 Ba

Graphite cathode

Furnace

Chamber

C Catalyst

Pressure gauge

π‘΅πŸ π‘ͺπŸπ‘―πŸ

Substrate on graphite sheet

βŠ™ βŠ™ βŠ™ βŠ™ βŠ™

βŠ™

βŠ™ βŠ™ βŠ™ βŠ™ βŠ™

βŠ™

βŠ™

βŠ™

Page 102: AND ANSWERS SEM-I/II

REDDAPPA C MODULE-5 INTERLINE PUBLISHING.COM

VTU ENGINEERING PHYSICS(17PHY 12/22) SEM-I/II Page 11

2. It consists of a chamber in an electric furnace through which a mixture of nitrogen and

acetylene is passed and let out through the out let.

3. A substrate is placed on the quartz sheet inside the chamber.

4. The temperature in the chamber is maintained at about 700-800.

5. Due to high temperature acetylene breaks down in to carbon atoms.

6. When these atoms come near substrate they get attracted and deposited as carbon

nanotubes in the presence of catalysts which are MWCNT’s.

7. SWCNT’

s are obtained when acetylene is replaced with methane or carbon monoxide at 1200

Q: Mention the properties of CNTS’.

1. CNT’s are highly elastic.

2. Young’s modulus of CNTs is about 9 times sronger than that of steel.

3. CNT’s exhibit large strength in tension.

4. CNT’s can be bent without breaking.

5. Electrical properties of CNTs ranges from semiconductor to good conductor.

6. CNT’s have low resistivity and low heat dissipation.

7. Electrical Conductivity of CNTs is maximum along the axis and very less along the

perpendicular direction.

8. CNT’s exhibit magneto-resistance ie:their resistance decreses with increasing mag.field.

9. Thermal conductivity of CNTs is maximum along the axis and very less along the

perpendicular direction

10. CNTs have very high strength to weight ratio and have low density.

11. CNTs are chemically more inert compared to other forms of carbon.

Q: Mention the uses of CNTS’.

1. CNTs can store lithium hence they are used in the manufacture of batteries

2. CNTs can store hydrogen hence they are used in fuel cells.

3. CNTs are used as atomic force microscope probe tips.

4. CNTs are used to produce flat panel display of television and computer monitors .

5. CNTs are used as light weight shield for electromagnetic radiation.

6. Semiconducting CNTs are used to produce field effect transistors used in computers

whose processing capacity is 104 faster than present processors.

7. CNTs are used to produce light weight high strength materials for aircrafts, rockets

automobiles etc

8. CNTs are used as chemical sensors to detect gases.

Page 103: AND ANSWERS SEM-I/II

REDDAPPA C MODULE-5 INTERLINE PUBLISHING.COM

VTU ENGINEERING PHYSICS(17PHY 12/22) SEM-I/II Page 12

Q: Explain the Principle, Construction and Working of SEM

(Scanning Electron Microscope).

Principle:

1. SEM is a device used to produce very high resolving images based on the principle of

wave nature of electrons.

2. The resolving power of SEM is 105 times more than that of a best optical microscope.

3. The schematic diagram of SEM is as shown in the labeled diagram.

Working:

1. The electrons produced from the electron gun are passed through two magnetic

condensing coils 1&2 in a highly evacuated chamber to condense the beam

2. The condensed beam is passed through scanning coil which will scan the entire specimen.

3. The scanning beam is focused on to the specimen by the magnetic objective coil.

4. When the electron beam incident on the specimen a few electrons are reflected back called

back scattered electrons which are detected using the detector 𝐷1.

5. Secondary electrons produced due to interaction with valence electrons are detected by

the detector 𝐷2

6. and X-rays produced due to deep penetration are detected using the detector 𝐷3

7. High resolution 3D image can be seen on the TV monitor to which 𝐷1, 𝐷2& 𝐷3 connected.

8. Biological specimens must be dehydrated and non conducting samples must be coated

with a thin conducting layer.

Electron gun

To Vacuum pump

Magnetic condenser coil -1

Magnetic condenser coil-2

Scanning coil

Objective magnetic coil

𝐷1 back scattered electron detector

𝐷2 Secondary electron detector

𝐷3 X-rays detector

Evacuated chamber Specimen

Base

TV monitor

Page 104: AND ANSWERS SEM-I/II

REDDAPPA C MODULE-5 INTERLINE PUBLISHING.COM

VTU ENGINEERING PHYSICS(17PHY 12/22) SEM-I/II Page 13

Q:Mention the uses of SEM.

1. SEM is used to study reflectivity, roughness of the surfaces .

2. SEM is used to study the composition of a compound and the abundance

of the constituents.

3. SEM is used to study biological specimens like pollen grains, blood cells

tissues, bacteria etc

4. SEM is used to study the structures, corroded layers .

5. SEM is used in forensic science for examining gunshot residues etc

6. SEM is used in textile industry for the evaluation of fabric.

@@@@

PROBLEMS SECTION

MODULE-5 : SHOCK WAVES AND SCIENCE OF NANO MATERIALS

Formulae needed:V = 𝑋

𝑑 ; M =

𝑉

π‘Ž ; Ξ» =

β„Ž

√2π‘šπ‘’π‘‰ or

1.228 𝑋10βˆ’9

βˆšπ‘‰ ( For an electron)

1. The distance between the two pressure sensors in a shock tube is 100 mm.The time taken

by a shock wave to travel this distance is 200 microsecond.If the velocity of sound under

the same conditions is 340 m/s. find the Mach number of the shock wave. ( 4 marks)

Given: x =100 mm=100x10βˆ’3 m,t =200ΞΌS=200π‘₯10βˆ’6 S,a=340 m/s ,M=?

Using, ,V=𝑋

𝑑 and M =

𝑉

π‘Ž

We get, M = 𝑋

π‘Žπ‘‘ =

100π‘₯10βˆ’3

340π‘₯200π‘₯10βˆ’6

=1.471

2. In a scanning electron microscope ,electrons are accelerated by an anode potential

difference of 60 kilo volt. Estimate the wavelength of the electrons in the scanning beam.

( 4 marks)

Given: V=60KV=60x103 V , h=6.63x10βˆ’34 Js; m=9.1x10βˆ’31kg; e=1.6x10βˆ’19 C; Ξ» =?

Using Ξ» = β„Ž

√(2π‘šπ‘’π‘‰)

= 6.63π‘₯10βˆ’34

√(2π‘₯9.1π‘₯10βˆ’31π‘₯1.6π‘₯10βˆ’19 π‘₯60π‘₯103) = 5.016 x 10βˆ’12 m

or Ξ» = 1.228 𝑋10βˆ’9

βˆšπ‘‰

= 1.228 𝑋10βˆ’9

√(60π‘₯103) = 5.013 x 10βˆ’12 m

Page 105: AND ANSWERS SEM-I/II

REDDAPPA C MODULE-5 INTERLINE PUBLISHING.COM

VTU ENGINEERING PHYSICS(17PHY 12/22) SEM-I/II Page 14

3. In a Reddy shock tube,it was found that, the time taken to travel between the two sensors

is 195 ΞΌs.If the distance between the two sensors is 100mm,find the Match number.(CBCS-

Jun/Jul16)

Given: x=100 mm=100x10βˆ’3 m,t =195ΞΌS=195π‘₯10βˆ’6 S,a=?(Not given) ,M=?

Using, ,𝑉𝑆 = 𝑋

𝑑

= 100π‘₯1 0βˆ’3

195π‘₯10βˆ’6

= 512.8 m/s and

Also, M = 𝑉𝑆

π‘Ž

= 512.8

π‘Ž

4. Calculate the wavelength of an electron accelerated under a potential

difference of 100 V in scanning electron microscope.

Given: V=100V , h=6.63x10βˆ’34 Js; m=9.11x10βˆ’31kg; e=1.6x10βˆ’19 C; Ξ» =?

Using Ξ» = β„Ž

√( 2π‘šπ‘’π‘‰)

=6.63π‘₯10βˆ’34

√( 2π‘₯9.11π‘₯10βˆ’31π‘₯1.6π‘₯10βˆ’19 π‘₯100)

= 1.228 x 10βˆ’10 m

or Ξ» = 1.228 𝑋10βˆ’9

βˆšπ‘‰

= 1.228 𝑋10βˆ’9

√(100)

= 1.228 x 10βˆ’10 m

5. The distance between two pressure sensors in a shock tube is 150 mm. The

Time taken by a shock wave to travel this distance is 0.3 ms.If the velocity

of sound under the same condition is 340m/s.Find the Mach number of the

shock wave.

Given: x =150 mm=150x10βˆ’3 m,t = 0.3mS= 0.3π‘₯10βˆ’3 S, a=340 m/s ,M=?

Using, ,V = 𝑋

𝑑 and M =

𝑉

π‘Ž

We get, M = 𝑋

π‘Žπ‘‘

= 150π‘₯10βˆ’3

340π‘₯0.3π‘₯10βˆ’3 = 1.471

Page 106: AND ANSWERS SEM-I/II

REDDAPPA C MODULE-5 INTERLINE PUBLISHING.COM

VTU ENGINEERING PHYSICS(17PHY 12/22) SEM-I/II Page 15

6. Calculate the wavelength of an electron accelerated under a potential

difference of 100 V in scanning electron microscope.

Given: V=100V , h=6.63x10βˆ’34 Js; m=9.11x10βˆ’31kg; e=1.6x10βˆ’19 C; Ξ» =?

Using Ξ» = β„Ž

√(2π‘šπ‘’π‘‰)

= 6.63π‘₯10βˆ’34

√(2π‘₯9.11π‘₯10βˆ’31π‘₯1.6π‘₯10βˆ’19 π‘₯100)

= 1.228 x 10βˆ’10 m

@@end@@

OR Ξ» = 1.228 𝑋10βˆ’9

βˆšπ‘‰

= 1.228 𝑋10βˆ’9

√(100)

= 1.228 x 10βˆ’10 m

Page 107: AND ANSWERS SEM-I/II

IMPORTANT IA TEST & VTU EXAMINATION QUESTIONS

MODULE-3

Lasers and Optical Fibres.

1. Explain : Induced absorption, Spontaneous emission , Stimulated emission and Derive an expression for energy

de sit i ter s of Ei stei ’s coefficie ts. 2. Explain the principle,construction and working of CO2 laser and Semiconductor laser.

3. Explain Recording and Reconstruction of images in Holography.

4. Explain types of Optical fibres with diagrams.

5. Explain point to point communication using optical fibres with block diagram.

6. Define attenuation and explain attenuation mechanisms/losses in optical fibres.

7. Derive an expression for acceptance angle (Theory of optical fibre) and Numerical aperture.

8. Explain the applications of lasers: Measurement of atmospheric pollutants, Welding,

Cutting & Drilling.

9. Explain the Requisites of Laser system.

MODULE-4

Crystal Structure.

1. Explain seven crystal systems with diagrams.

2. What are Miller Indices and explain the steps/method of finding the Miller Indices of a plane.

3. Derive an expression for inter-planar spacing/distance.

4. Define Co-ordination number and (APF)Atomic Packing factor).Also find the APF of SC,BCC&FCC.

5. Write a note on structures of Diamond and Perovskites with diagrams.

6. Deri e Bragg’s la a d e plai ho to erif Bragg’s la a d Ide tif cr stals usi g Bragg’s X-ray

spectrometer(Diffractometer)

7. Explain Polymorphism and Allotrophy.

MODULE-5

Shock Waves and Science of Nano Materials.

1. Explain the terms : Acoustic waves, Infrasonic waves, Audible waves, Ultrasonic waves,

Mach waves, subsonic waves , supersonic waves and Shock waves.

2. State and explain basics of conservation of mass, momentum and energy.

3. Derive/explain Rankine-Hugonite( Normal shock wave) equations.

4. Describe the principle, construction and working of operated Reddy shock tube.

5. Explain density of states with diagrams.

6. Explain the synthesis of nano particles by Ball mill method and Sol-Gel method.

7. Explain the synthesis of CNTs by Arc discharge method and Pyrolysis method.

8. Explain the principle, construction , working and uses of SEM(Scanning Electron Microscope)

9. Methods of producing shock waves and properties and uses of shock waves.

10. Explain different types and structures of CNTs and mention properties & uses of CNTs

Page 108: AND ANSWERS SEM-I/II

HINTS FOR SURE SUCCESS……..

β€œ SLOW AND STEADY WINS THE RACE β€œ

1. Adopt Easy to Difficult approach. First thorough with the

easy questions of the topic and then attempt difficult questions, which you will feel easy.

2. Practice writing diagrams again and again, because no marks are

awarded for derivations without relevant diagrams wherever needed.

3. Memorize all the formulae so that you can solve problems and

score maximum marks with less effort.

4. Always express the given data in terms of simple SI units ,then write

the relevant formula and substitute the data.

5. While answering in the tests/exams, First choose the questions

which you can answer correctly and completely.

6. Attempt all the questions by writing whatever you know about it even

if you are not sure about the answers.

7. Practice the use of calculator by solving the solved problems again

and again, so that you will not find any difficulty in the tests/exams.

8. Take all the tests without fail before the main examination so that

you will know your strengths and weakness .This will help you to

rectify the mistakes and score more marks.

9. Pay attention to the units of the physical quantities so that you can

improve your marks.

10.Write the diagrams neatly and answers legibally.

@@@@@